Sei sulla pagina 1di 149

Home Page Of The IASPrep 2003 Part 2

Ebook
You have download this ebook from the file section of the group
www.groups.yahoo.com/group/IASPrep_2003
It will assist you in your preparation of Civil Service Exam conducted by
UPSC.
If you have any suggestions to improve the ebook please contact the
author at the following email id
mailto:vivek440@yahoo.com OR mailto:vivek_gupta@vsnl.net

This book is a freeware. It can be used without paying anything to the
author.

This is Part 2 of the ebook. To download Part 1 of the ebook visit
www.groups.yahoo.com/group/IASPrep_2003

To know about the new versions of this book join the group
www.groups.yahoo.com/group/IASPrep_2003


NOTE: If you posses any matter, which you think, can be included in this
ebook please send it to me at vivek440@yahoo.com

PS: Please send me feedback. Are you finding this ebook useful? Your
feedback will motivate me to improve this ebook and make it more and
more useful for you.






NP


Time Table For Main Exam - 2001

Date/Day
Forenoon Session
9.30 A.M. to 12.30 P.M
Afternoon Session
2.00 P.M. to 5.00 P.M.
19.10.2001
(Friday)
General Studies Paper-I General Studies Paper-II
20.10.2001
(Saturday)
Essay English
21.10.2001
(Sunday)
Indian Language (Assamese/Bengali/Gujarati/
Hindi/Kannada/Kashmiri/Konkani/Malayalam/
Manipuri/Marathi/Nepali/Oriya/Punjabi/
Sanskrit/Sindhi(Devanagri or Arabic
Script)/Tamil/Telugu/Urdu)
----
29.10.2001
(Monday)
History Paper-I History Paper-II
30.10.2001
(Tuesday)
Mathematics Paper-I/Statistics Paper-I Mathematics Paper-II/ Statistics
Paper-II
31.10.2001
(Wednesday)
Sociology Paper-I/Anthropology Paper-I Sociology Paper-II/
Anthropology Paper-II
02.11.2001
(Friday)
Geography Paper-I Geography Paper-II
03.11.2001
(Saturday)
Civil Engg. Paper-I/ Electrical Engg. Paper-I/
Mechanical Engg. Paper-I /Medical Science
Paper-I/
Civil Engg. Paper-II/ Electrical
Engg. Paper-II/
Mechanical Engg Paper-II /
Medical Science Paper-II
05.11.2001
(Monday)
Political Science & International Relations
Paper-I/Public Administration Paper-I
Political science & International
Relations Paper-II/ Public
Administration Paper-II
06.11.2001
(Tuesday)
Literature of
Assamese/Bengali/Gujarati/Hindi/
Nepali/Kashmiri/Konkani/Marathi/Oriya/
Punjabi/Sanskrit/Pali/Sindhi
(Devanagari/Arabic Script) Malyalam/
Manipuri/Kannada/Tamil/Telugu/
Urdu/Arabic/Persian/German/French/Russian/
Chinese/English Language Paper-I
Literature of Assamese/
Bengali/Gujarati/Hindi/
Nepali/Kashmiri/Konkani/
Marathi/Oriya/Punjabi/
Sanskrit/Pali/Sindhi
(Devanagri/Arabic Script)
Malyalam/Manipuri/
Kannada/Tamil/Telugu/Urdu/
Arabic/ Persian/German/
French/Russain/Chinese/English
Language Paper-II
?


07.11.2001
(Wednesday)
Physics Paper-I Physics Paper-II

08.11.2001
(Thursday)
Commerce & Accountancy Paper-
I/Management Paper-I
Commerce & Accountancy
Paper-II/Management Paper-II
09.11.2001
(Friday)
Zoology Paper-I Zoology Paper-II
10.11.2001
(Saturday)
Economics Paper-I Economics Paper-II
12.11.2001
(Monday)
Botany Paper-I Botany Paper-II
17.11.2001
(Saturday)
Law Paper-I Law Paper-II
19.11.2001
(Monday)
Philosophy Paper-I Philosophy Paper-II
20.11.2001
(Tuesday)
Agriculture Paper-I/Animal Husbandry &
Veterinary Science Paper-I
Agriculture Paper-II/Animal
Husbandry & Veterinary
Science Paper-II
21.11.2001
(Wednesday)
Psychology Paper-I Psychology-II
22.11.2001
(Thursday)
Chemistry Paper-I Chemistry Paper-II
23.11.2001
(Friday)
Geology Paper-I Geology Paper-II


Contact Author At vivek440@yahoo.com

Some Famous Coaching Institute In Delhi

! Raos IAS Study Circle
Address: 44, Dakshineshwar Building, 10, Hailey Road, New Delhi- 110001
Phone: (011) 3355906, 3738906
Fax: 3712795

! Vaids ICS
Address: 58A/2, Kalu Sarai (Sarv Priya Vihar), New Delhi-110016
Phone: 6533839, 6602341


NOTE: As I am living in Delhi, so I have knowledge about
institute in Delhi only. But I want to compile a list of
all good institute in various parts of the country. To
achieve this goal I will need your help. Please send me
the name and address of famous institute in your locality
so that I can add them in my ebook and make this
compilation exhaustive and useful for everybody. Please
send the names of the institute at the following email id
vivek440@yahoo.com OR vivek_gupta@vsnl.net























Saturday, August 03, 2002 1 Q7

1. Name the weapons used by Delhi Police?

2. Name the latest weapons which Delhi Police is acquiring?

3. Birth place of L.K.Advani?

4. Venue of 14
th
international AIDS Conference?

5. Venue of the 1
st
World Ayurveda Congree (held recently)?

6. Nominee of the BJP party for the post of vice president?

7. Give one major difference between President election & Vice President elections?

8. Nominee of the Congress party for the post of vice president?

9. Full form of NIMHANS and where is it located?

10. What does the mental health act in 1987 recommended?

11. Still 2 states dont have a mental health authority. Name them?

12. In which year Punjab National Bank was established?

13. In which country bulls fight Sanfermines is held?

14. Current Guinness Book World Record Holder for having done 819 back hand push ups in one
hour?

15. Name the Present Ghana President & his wife?

16. What is the cause of Japanese B Encephalitis disease & recently due to this disease many
children where killed in which state?

17. Name the company who was awarded National Golden Peacock Training award for 2002?

18. Name the winner of the 10
th
Rajiv Gandhi National Sadbhavana Award for their outstanding
contribution towards communal harmony and national integration?

19. Differentiate between illusion & Hallucination?

20. In soccer there are 11 players in a team. When was this established?

21. Name the first soccer club formed?

22. Full form of FIFA and when was it established?

23. What is Indias present population growth rate?

24. In the year 2002, who was awarded the Ashok Chakra (Highest Peace Time Gallantry Award)?
Saturday, August 03, 2002 2 Q7


25. Who is Jharkhand CM?

26. Why was Jharkhand in news recently?

27. Where is Army Sports Institute located?

28. The blueprint for 2004 olympic games is based on which countrys sports model?

29. In 100 years of Olympic participation 1900-2000, how medals India have won?

30. Other than being the Vice President of the country, he held another constitutional post in the
past. What is it?

31. Name the Chief Minister of Sikkim and to which party does he belong?

32. Who was the CM before the present CM of sikkim and what his party?
HINT: He ruled the state for 14 years

33. Name the first woman speaker of India and to which state she belongs?

34. Who won the Ramon Magsaysay Award 2002 in the emergent leadership category?

35. When is Black Tiger Day is celebrated & what is the reason of its celebration?

36. In which district was the curriculum called Hoshangabad Science Teaching Programme (HSTP)
introduced and why it was recently abolished?

37. Full form of AK 47 and who invented it?

38. Name an Indian who was selected by the Peter Gruber Foundation to receive the 2002 Justice
Prize?

39. Following are the weapons India want to buy from Russia. Tell what are they?
i. Admiral Gorshkov
ii. Amur 1650
iii. Krivak 3

40. What are Falabellas?

41. Name the smallest horse alive?

42. Name the events in which Anjali Vedpathak Bhagwat won the gold medals in the
Commonwealth Games held at Manchester?

43. What is Rhinitis disease?

44. In which year SAARC constituted?


Saturday, August 03, 2002 1 A7

1. Lee Enfield
! Disadvantage: The rifle has to reloaded manually after each shot
SLR 7.62 (SLR stands for self loading rifle)
! Disadvantage: It has large bore, unsuitable for police. Small bore weapons injure the
target whereas large bore weapon kill the target

2. AK-47 & the Heckler and Koch MP-5 sub machine gun
3. Karachi
4. Barcelona
5. Kochi, Kerela
6. Bhairon Singh Shekhawat
7. In the president election, the voters include MPs from both house as well as MLAs of all the
assemblies while only the MPs choose the vice president. However even in the Presidents case,
an MPs vote carries far more weight than an MLAs.
8. Sushil Kimar Shinde
9. National Institue of Mental Health and Neurological Sciences. Located in Banglore
10. The act recommended a mental health authority for every state. Besides the national mental
health programming stipulates a mental health authority for every district
11. Bihar & Orissa
12. 1865
13. Street of Pamplona in northern spain
14. Journalist Bibhuti Nayak
15. Ghana President: John Agyekum Kufuor His wife name: Theresa
16. It is caused when the virus is infected in human beings through mosquitoes which have bitten
infected pigs. State: Assam
17. Institute of Oil and Gas Production Technology of Oil and Natural Gas Commission(ONGC)
18. Harsh Mander (Retired IAS officer) & Teesta Setalvad(Journalist)
19. When a person has an illusion, there is something present that stimulates the eyes or sense.It
can be verified because other persons can see it as well. Whereas when a person has
hallucination nothing is there. Mostly when a person is disturbed he hallucinate. Certain drugs
also cause hallucination.
20. In1870, 11 player team format was established
21. In 1857, Sheffield, England
22. Federation International Football Association, in 1904
23. 1.9%
24. Subedar Surinder Singh. He gunned down 4 pakistani mercenaries in an encounter near Poonch
25. Babulal Marandi
26. Because of its new domicile policy
27. Ghorpuri in Pune
28. It is based on Cuban sports model where, in every province, there is a sports initiation school
that gives sports as well as educational training to 15,000 students every year
29. 8 gold, 3 silver & 4 bronze medals = Total 15 Medals
! Breakup Of Medals
i. Hockey 8golds, 1 Silver, 1 Bronze
ii. Sprinter Norman Pritchard 2 Silver
iii. Wrestler K.D. Jadhav 1 Bronze
iv. Tennis Player Leander Paes 1 Bronze
v. Weight Lifter Karnam Malleswari 1 Bronze
30. He was governor of Andhra Pradesh for more than 7 years
31. Pawan Kumar Chamling, Sikkim Democratic Front
Saturday, August 03, 2002 2 A7

32. Nar Bahadur Bhandari, Sikkim Sangram Parishad
33. Kalabati Speaker, Sikkim
34. Sandeep Pandey, lives in Lucknow and is involved in social work
35. July 5, celebrate in the remembrance of the first suicide mission of the LTTE in 1987. On this
day, Captain Millar, who drove a lorry loaded with explosives into the military camp at Nelliady
Maha Vidyalaya in 1987 and killed 200 soldiers
36. Hoshangabad district in Madhya Pradesh. The government assessment report pointed that
even after 30 years HSTP has not shown any significant increases in the learning outcomes of
the children when measured by common standards
37. Avtomat Kalashnikova and the man behind the gun was Sergent Mikhail Kalashnikov (Russian
Sergent)
38. Senior Supreme Court lawyer and Rajya Sabha MP Fali S Nariman
39. They are as follows
a. Aircraft Carrier
b. Submarines
c. Talwar class stealth frigates

40. Falabellas are miniature horses that grow up to just 2 and a half feet
41. Name: Sugar Dumpling (Vital statistics: 20 inches and 13.63 kilos) bred by Walter Smith
McCoy of the US
42. Womens 10m air rifle (Singles), Womens 50m rifle 3 positions (Singles), Womens 10m air
rifle (pairs) with Suma Shirur & Womens 50m rifle 3 positions (Pairs) with Raj Kumari
43. Inflammation of the mucus membranes in the nose, its prolonged neglect can cause asthma
44. 1985












Brahmos: Missile Being Developed by India & Russia

! It is supersonic cruise missile with a range of 290 km

! It is 8 m long

! It was recently tested from the Interim test range near balasore

!


Changes In Civil Procedure Code
The union government has notified the Civil procedure code (Amendment) act, 2002 on July 1.
Following changes where introduced in this amendment

! A time limit of one year for the settlement of civil cases was fixed

! The act allows the use of fax, email and courier for serving summons.

! The situation of refusal to accept summons by a party has been made almost impossible by
providing that the person to whom it was issued shall be presumed to have received it even if the
summons are returned with an endorsement that the party refuses to accept

! The general power of courts to extend the time prescribed in the CPC is restricted to 30 days
now.

! On filing of revision application against the order of a subordinate court, its record shall not be
called unless the high court orders to do so specifically cases for conciliation and arbitration

! The act now limits the number of adjournments in a civil case to 3

! The judges too have been made accountable. They will have to give their ruling within 60 days
of the completion of hearing of a civil case

! The existing provision of a second appeal in financial suits where the value did not exceed Rs
25, 000 has also been abolished

!


East Timor: Newest Country
! 192
nd
independent country on May 19 after 450 years of oppressive foreign rule


Fact File Of Sikkim
! Capital: Gangtok

! 22
nd
state of India

! Date Of Accession: 26 April, 1975

! Official Language: English

! Phunstock, the great great grand son of Guru Tashi, born in 1604, became the first king or
Chogyal of Sikkim. The dynasty ended when the state merged with India in 1975 during during
the reign of Miwang Chogyal Chempo Palden Thondup Namgyal




Indias Future Potential Players For Olympics

! Ali Qamar
o He is in the light flyweight (48 Kg) division
o He first came into the limelight when he won gold at the 1999 SAF games in
Kathmandu

! Diwakar Prasad
o He is in boxing

! Pankaj Advani
o He is good in billiard

! Seema Antil
o She is into disc throwing

! Manan Chandra
o He is into Snooker
o He is the youngest cueist to represent India in the 1988 Bangkok Asian Games and
has already been selected for the 2002 Games

! Abhinav Bindra
o He is in Air Rifle

! Anjali Vedpathak Bhagwat
o World number one in the womens air rifle event
o When she qualified for the final of the 10m womens air rifle event in the 2000
sydney Olympics she was the first Indian to do source




Industrial Sickness In India
! Causes
1. Stiff competition from MNCs

2. Competition from unorganized sector which can offer cheaper products due to lower taxes &
other charges

3. Inadequate power supply

4. Payment by heavy excise duties by Indian companies while imports were available at cheaper
rates due to drastic cut in custom duties. Because of cheaper products from China etc the
indigenous manufacturers have found it very tough to market their products

5. Many promoters of these sick units establish them with the sole purpose of getting finances
and using the same for purposes other than investing in that unit. And this is the reason why we
have many sick industries but not even a single sick industrialist

! Solutions
1. Short term loan can be converted into long term so that financial charges are spread over a long
period

2. Financial reconstruction with or without change in management. New partners can also be
induced who will bring in capital to revive the company

3. Merger of the company with any other healthy company of the same group, source that profit of
the healthy company can be used to solve the liquidity problem of the sick company

4. Sale of lease of the units to any other party which could bring in the required interest free funds
to revive the company.

5. If the companys non-operating cost is too high because of fixed financial charges, a settlement
with the creditors should be made considering the projected cash flows, for the repayment of
their dues in a phased manner

6. the operating agency, which monitors revival programme, should also be made responsible in
case, the revival scheme does not succeed

7. In case of a multi product company, the company should sell out the loss making products






Steps Taken By Government To Improve
Countrys Performance In Olympics

! First Step: In 1998, IOA, the sports commission was formed under the chairmanship of
K.P.Singh Deo with the motto, Eight years for gold

! This commission made the following recommendation
o There was an urgent need for convergence of financial, administrative and
managerial resources of government, public and private organizations and institues
for excellence in Olympic sport
o Industry must become a partner and get a 100% tax rebate
o The private sector should help develop and run infrastructure




General Studies - 1997 (Main) (Paper - I)
Time Allowed : Three Hours Maximum Marks : 300
Candidates should attempt questions from all the Sections I, II and III
SECTION I

1. How did Sardar Vallabhbhai Patel succeed in avoiding the evil consequences of the doctrine of
lapse of paramountcy and in integrating Indian States?
OR
Examine the causes and nature of extremism in Indian politics in the early part of the present
century. (About 250 words)
2. Answer any two of the following:
(a) What were the salient features of Gandhi-Irwin Pact?
(b) Discuss Mahatama Gandhis concept of Basic Education. How far was it a departure from
conventional system of education?
(c) What were the circumstances that led to the partition of India in 1947?
(About 250 words on each)
3. Answer any three of the following :
(a) What do you understand about Drain of wealth during British rule? Examine its effects on
Indian economy.
(b) The name of RajaRam Mohan Roy stands foremost in the field of religious and social
reforms. Elucidate.
(c) The Preamble to the Constitution is aimed to embody the fundamental values and the
philosophy on which the Constitution is based Elucidate.
(d) What are the provisions regarding the protection of Linguistic minorities in the Constitution?
(About 75 words on each)
4. (a) Write about the chief features of the following :
(i) Chaitya (ii) Vijayanagara (iii) Yaksha Gana (iv) Khayal
(b) What do you know about the following?
(v) Lalit Kala Academy (vi) Poona Sarvajanik Sabha
(vii) Rowlatt Act (viii) August Declaration,1917
(c) Where are the following located and with which event are they associated?
(ix) Surat (x) Vaikom (xi) Midnapur
(d) Where are the following become famous?
(xii) Shymji Krishnavarma (xiii) Rani Gaidinliu
(xiv) Saifuddin Kitchlew (xv) Dr. Nivedita Bhasin (About 20 words on each)
SECTION II
5. Where and when was the meeting between the Prime Ministers of India and Pakistan held in


1997? What were the main decisions taken and what follow-up action has been taken?
OR
Describe the recent significant developments in Sino-Russian relations. How have the Russain
people reacted to these developments? (About 250 words)
6. Attempt any three of the following:
(a) What is the Taliban Movement? How has it influenced the geopolitical equations in Asia?
(b) What circumstances led to the victory of Socialists in the parliamentary elections in France?
(c) What was the Pathfinder by Mission? What were its main findings?
(d) What do you understand by Heavens Gate? Why was it in the news recently?
(About 100 words on each)
7. Attempt any three of the following:
(a) What fallout can the Mir troubles have on the future of US-Russian cooperation in space?
(b) Why has there been opposition from the North-eastern States to the Supreme Court ban on all
activities inside forests?
(c) What is VDIS? What does it aim at?
(d) What was the fallout of the Plenary Session of Indian National Congress in Calcutta in August
1997?
(e) On what grounds did India not sign the CTBT?
(f) What is likely to be the impact on Hong Kongs economy after its transfer to China?
(g) What was the controversy about Prithvi? (About 50 words on each)
8. (a) Name the authors of the following books?
(i) The God of Small Things (ii) A Suitable Boy
(iii) Nice Guys Finish Second
(b) Who are the following? Why have they been in the news?
(iv) Mahashweta Devi (v) Sivaji Ganeshan (vi) Sanjay Ghosh
(vii) Sanath Jayasurya (viii) Martina Hingis (ix) Iva Majoli
(x) Mike Tyson (xi) Mother Teresa
(c) Name the directors of the following films:
(xii) Train to Pakistan (xiii) Sardari Begam (xiv) Saaz
(xv) Batman and Robin (About 20 words on each)
SECTION III
9. In1985, out of a total 2000 students in a college, 1400 were for graduation and the rest for
post-graduation. Out of 1400 graduate students, 100 were girls; however in all there were 600girls
in the college.
In 1990, the number of graduate students increased to 1700 out of which 250 were girls, but the
number of post-graduate students fell to 500, of which only 50 were boys.
In 1995, out of 800girl students, 650 were for graduation, whereas the total number of graduate
students was 200. The number of boys and girls in graduate classes was equal.


Represent the above information in a tabular form. Also calculate the percentage increase in the
number of graduate students in the college in 1995 as compared to 1985.
10. (a) In one of the Five Year Plans of India, of the total of Rs. 7250 crore, the plan outlays for
Agriculture & Irrigation, Transport & Communication, Community Development, Power, and
Industry & Minerals had been rupees 1275 crore, 1450 crore, 400 crore, 925 crore, 1500 crore
respectively. The remainder was under the head Miscellaneous.
Represent the data by a pie diagram showing the relative percentages and corresponding angles in a
tabular form.
(b) The mean of marks obtained in an examination by a group of 100 students was found to be
49.98. The mean of the marks obtained in the same examination by another group of 200 students
was 52.32. Find the mean of the marks obtained by both the groups taken together.
11. (a) Mention, in each of the following cases, the most appropriate diagram to represent the data.
(i) Number of refrigerators produced by a certain factory during the last 10 yeas.
(ii) The expenditure on different major heads by a family during a given month.
(iii) Consumer price index number of working class people in two cities for the period 1985 to
1994.
(iv) The daily sales (in Rs.) in a shop for the last one year period.
(b) The following table shows the distribution of men according to their heights:
Height in cm. No. of men
less then 140 16
less then 150 40
less then 160 72
less then 170 93
less then 180 106
less then 190 125
less then 200 150
Find : (i) the percentage of men having height between 150 cm. and 180 cm. and
(ii) the percentage of men having height between 155 cm. and 170 cm.
12. (a) Examine the correctness of the following data regarding consumers liking for two products
A and B.
Out of a total of 200 consumers, 100 liked both A and B, 70 liked A but not B and 50 liked B but
nor A.
(b) Two firms A and b respectively pay 80,000 and Rs.60,000 per month as salary to its employees.
From this information, can we concludes that firm A is paying higher salary to its employees as
compared to the firm B ? State reason.
(c) The following is a series of index numbers using 1980 as base year. Reconstruct the series
using 1983 as base year.
Year 1980 1981 1982 1983 1984


Index 100 110 175 250 300




General Studies - 1997 (Main) (Paper - II)
Time Allowed : Three Hours Maximum Marks : 300

Candidates should attempt questions from all the Section I, II, and III.

SECTION I
1. (a) How are the President and the Vice President of India elected? What are the constitutional
issues involved in their election? (About 250 words)
OR
(b) What is Social Justice? How can reservation of seats for women in Parliament contribute to the
establishment of a socially just society in India? (About 250 words)

2. (a) 20th century was century of oil, 21st century would be the century of natural gas.
Comment.
OR
(b) Give your assessment on land reforms in India. (About 250 words)
3. (a) Why is Agni Project important for India? (About 250 words)
(b) How has Indias Nuclear Programme contributed to the economic and defence requirements of
the country? (About 250 words)
SECTION-II
4. Answer any two of the following :
(Answer to each question should be in about 150 words)

(a) Present your views for and against the creation of an All India Judicial Service.

(b) Discuss the factors responsible for the rise of regionalism in India. How does it influence the
political system?
(c) What, according to the Supreme Court, constituted The Basic Features which it upheld in
cases known as
(i) Keshavanand Bharati v/s State of Kerala (1990) and
(ii) Minerva Mills v/s Union of India (1990)
(d) How does the Parliament control the financial system in India?
5. Answer any two of the following :
(Answer to each question should be in about 150 words)
(a) Discuss the steps taken by the government in recent years to control inflation.
(b) What measures has the government taken to make women economically independent and self-
reliant during the last decade?
(c) Examine the important changes in Indias Industrial Policy since the year 1991.
(d) Discuss the advantages of Alternative Scheme of Devolution suggested by Tenth Finance
Commission of India.


6. Answer any two of the following :
(Answer to each question should be in about 150 words)

(a) Discuss the origin of Monsoon in India.
(b) What measures has the Government of India taken for the development of Tribal areas in India?
(c) Why is there a demand for making Uttarakhand a Separate Hill State in India?
(d) Why has the Narmada River Valley Project attracted the attention of the common people?
7. Answer any two of the following :
(Answer to each question should be in about 150 words)
(a) What is cryogenic engine? Explain its importance to our country.
(b) What is Madcow disease?
(c) What are fullerenes? Why are they important?
(d) What is Polio? Name its few features and how it can be prevented.
SECTION - III
8. Answer four from Group A, three from Group B and three from Group C.
(Answer to each question should be in about 25 words)
GROUP-A
(a) Why was HODVAT Scheme introduced in the year 1986?
(b) Why was the rationale for Mid-day Meal Scheme?
(c) How has the Census (1991) defined the Urban Areas?
(d) Describe the use of Command Area Development in India.
(e) What are the objectives of Social Forestry?
(f) What are the main objective of Ninth Five Year Plan?
GROUP B
(a) What do Rule 184 and 193 in Parliamentry Procedure signify?
(b) What is meant by Gujral Doctrine? Write its specific principles.
(c) Give a brief account of C.M.P. (Common Minimum Programme) of the united Front
Government at the Centre.
(d) Write any four fundamental duties prescribed in the Constitution of India.
(e) What specific provisions exists in the Constitution of India about child labour?
(f) What is Article 356 in the Indian Constitution? Comment.
GROUP-C
(a) What is the safe level of noise for the human ear? Give the noise level of electric bell.
(b) What is the difference between a comet and a meteor?


(c) What is E-mail?
(d) What do the following stand for?
(i) TIFR (ii) GRDO (iii) NEERI
(e) What is tissue culture?
(f) What are Quasars?





General Studies - 1998 (Main) (Paper - I)
Time Allowed : Three Hours Maximum Marks : 300

Candidates should attempt questions from all the Section I, II, and III. The number of marks
carried by each question is indicated at the end of the question.

SECTION- I

1. Did Jawaharlal Nehru really 'speak' the 'language' of Gandhi? Locate the points of their
agreements and departures.
OR
How did economic nationalism mirror the work of the early nationalist leadership in India?
(About 250 words) 35
2. Answer any two of the following : 15 x 2 =30
(About 150 words on each)
(a) Why did the moderates lose appeal with the Indians and failed to elicit desired response from
the British?
(b) How did the Policy of free trade hurt Indian textile industry and crafts in the latter half of the
19th century?
(c) Trace the origins of the R.N.I. Mutiny and evaluate its impact on the political situation in India.
3. Answer any three of the following : 10 x 3 =30
(About 75 words on each)
(a) Examine the ideas of Rabindranath Tagore on democracy.
(b) How did the Swadeshi Movement in Bengal influence the nationalist politics?
(c) What was the significance of the Prajamandal movements in the Indian States in the decade
preceding India's independence?
(d) Assess the role of C. Rajagopalachari during the pre-partition years of Indian public life.
4. (a) What do you know about the following : 2 x 15 =30
(About 20 words on each)
(i) Nil Darpan (ii) Sarda Sadan
(iii) Sabarmati Ashram (iv) Hunter Commission
(v) Bandi Jiwan
(b) Why have the following become famous?
(vi) Khan Abdul Gaffar Khan
(vii) Seth Jamanlal Bajaj (viii) S. Satyamurti
(ix) Udham Singh (x) Sarojini Naidu
(c) Write about the chief fatures of the following :
(xi) Kulu School (xii) Gopuram
(xiii) Wahabi Movement


(xiv) Bharat Dharma Mahamandal
(xv) Communal Award
SECTION II
5. Examine the reasons for the non-introduction of Women's Reservation Bill in the LokSabha.
(About 250 words) 35
OR
Discuss the implications of Israel's expansion plan. How have the major western powers reacted to
it ? (About 250 words)
6. Attempt any three of the following : 10 x 3 =30
(About 100 words on each)
(a) Examine the fall out of population explosion in India at the close of twentieth century.
(b) Discuss the justification for the creation of new states in U.P. and Bihir.
(c) What are the reasons for the slow progress of SAPTA?
(d) Describe the problem of KOSOVO.
(e) What is ASEAN PLUS? What is its nature and duty?
7. Attempt any five of following : 6 x 5 =30
(About 50 words on each)
(a) What were the circumstances leading to the promulgation of Prasar Bharati ordinance in August
1998?
(b) Who has claimed the responsibility for the recent bomb attacks on American embassies at
Nairobi and Dar Es Salam? What are their demands?
(c) What is the significance of Pope John Paul II's visit to Cuba?
(d) What are the significant features of Lokpal Bill recently introduced in the Lok Sabha?
(e) Why was a variant of Basmati rice in news recently?
(f) What solution has been arrived at recently in the Cauvery water dispute?
(g) Bring out the issues involved in the appointments and transfer of judges of the Supreme Court
and high Courts in India.
8. (a) Explain the following : 2 x 15 =30
(About 20 words on each)
(i) Netizens (ii) Shehab-3 (iii) Planet-B
(iv) W.H.O. (v) Greenpeace
(b) What do you know about the following?
(vi) Chattisgarh (vii) Article 356 (viii) Bentota
(ix) Narmada Sagar dam (x) Full monty
(xi) Pamirs (xii) Nelson Mandela
(xiii) Bacharuddin Jusul Habibie
(xiv) E.M.S. Namboodripad. (xv) Jnanpith Award


SECTION III
9. The total population of a country is 357 million of which 62 million live in urban areas. Of these
who live in rural areas, 234 million are agriculturists including 124 million earning members.
Among the urban population, 12 million are agriculturists of which 7 million are earning members.
Among the non-agriculturist rural population 43 million are earning are earning members whereas
among non-agriculturist urban people 14 million are non-earning members.
Represent the above information in a tabular form. 12
10. (a) The following table shows annual profits (Rs. in crores) of two investment companies A and
B during the given 5 years :
1993 1994 1995 1996 1997
A : 70 60 70 60 70
B : 60 50 60 80 80
If someone plans to invest Rs. 50,000, which of the two companies should be preferred and
why ? 8
(b) An investor bought shares of a company at a premium of Rs. 20. If the par value of the share is
Rs. 120 and the company declares 21% dividend in a given year, find the rate of interest received
by the investor.
11. (a) Mention, in each of the following cases, the most appropriate diagram to represent the data
: 5
(i) Number of AIDS patients in a country detected during the last 10 years.
(ii) Monthly production of eggs in a poultry farm in a given yaer.
(iii) Yearly export and import (in dollars) of a multinational firm during the period from 1990 to
1997.
(iv) Distribution of weights of 500 school children in the age group 15-18 years.
(v) Political party-wise percentage of votes cast during a parliamentary election.
(b) The following table shows the IQ scores of adolescents aged 15-17 years:
Interval Frequency
121 and above 15
116-120 35
111-115 50
106-110 60
101-105 100
96-100 75
91-95 70
86-90 60
81-85 25
80and below 10


Find :
(i) the cut-off point above which 25% most intelligent adolescents fall.
(ii) the cut-off point below which the 25% least intelligent adolescents fall. 8
12. (a) State whether the following statements are true or false.
Give reasons. 6
(i) A person's monthly income first increases by 20% and then decreases by 20% . There is no net
change in his income.
(ii) All facts expressed numerically pertain to statistics.
(iii) If A stands 50 % change of getting through Test I and Test II independently, then A stands at
least 50 % chance of getting through both the tests.

(b) The following is the series of index numbers taking 1992 as base year :
Year :- 1992 1993 1994 1995 1996 1997
Index :- 100 120 135 170 180 210
How would the index numbers be changed if 1993 is taken as base year? 7




General Studies - 1998 (Main) (Paper - II)
Time Allowed : Three Hours Maximum Marks : 300
Candidates should attempt questions from all the Section I, II, and III. The number of marks
carried by each question is indicated at the end of the question.
SECTION- I

1. (a) Briefly state the stages through which the present position of the Directive Principles
vis--vis the Fundamental Rights has emerged. (About 250 words) 40
OR
(b) Explain the concept of Prime Ministerial Government and account for its decline in recent
times in India. (About 250 words) 40
2. (a) The past 50 years of development in India were characterised by planning but the next 50
years of development would be led by the market. Comment. (About 250 words) 40
OR
(b) How is absolute poverty line measured? What are the important measures taken by the
Government to eradicate rural poverty in India? (About 250 words) 40
3. (a) What is remote sensing? What are its uses especially in the Indian context?
(About 250 words) 40
OR
(b) What is genetic engineering? Why is it getting increasingly important these days ?
(About 250 words) 40
4. Answer any TWO of the following :-
(Answer to each question should be in about 150 words) 20+20
(a) Highlight the significance of the Seventy-third Amendment to the Constitution of India.
(b) How are the States formed in India? Why have the demands of separate States like those of
Vidarbha, Telangana etc. not been considered by the Government recently?
(c) On what grounds the Legislative Councils are justified? How is it created or abolished in a
State?
(d) Differentiate and state the significance of general election, mid-term election and by-election.
5. Answer any TWO of the following :-
(Answer to each question should be in about 150 words) 20+20
(a) What are the main reasons for industrial sickness in India? How can it be overcome?
(b) Infrastructure bottlenecks continue to stifle the economic growth in India. Comment.
(c) Comment on the view that Monetary Policy in India is used more as a stabilisation device rather
than as a development tool.
(d) What has been the role of NRIs in the economic development of India in the recent past?


6. Answer any TWO of the following :-
(Answer to each question should be in about 150 words) 15+15
(a) What are the geo-economic causes of underdevelopment of various regions in India?
(b) Which parts of India have been identified as draughtprone? Mention the norms for such
identification.
(c) Describe the changes that have taken place in the direction of international trade of India since
independence.
(d) How do the Andaman and Nicobar Islands and the Lakshadweep differ in the geological
ecvolution and topographical conditions?
7. Answer any TWO of the following :-
(Answer to each question should be in about 150 words) 20+20
(a) Describe the development of Super-computers in India.
(b) How does the development of Pinaka benefit India?
(c) What are optical fibres? What are their advantages?
(d) What are enzymes? What is their importance?
8. Answer FOUR questions from Group A, THREE from Group B and THREE from Group C :-
(Answer to each question should be in about 25 words) 10x3=30
GROUP-A
(a) What is meant by Mutual Fund?
(b) Explain per capita income as a measure of economic growth.
(c) What is meant by Quality of life?
(d) What constitutes a Minor Irrigation Project?
(e) What is meant by Environmental Impact Assessment?
(f) What is the difference between Gross National Product and Net National Product?
GROUP-B
(a) What is meant by Protem Speaker?
(b) Point out the constitution and functions of the Central Vigilance Commission.
(c) State the amplitude of Art. 21 of the Constitution.
(d) Which of the cases regarding disqualifications for Membership of either House of Parliament
are decided by the President?
(e) Differentiate between Parliamentary Secretary and Lok Sabha Secretary.
(f) What is a Privilege Motion?
GROUP-C
(a) Why are transgenic organisms important?


(b) State the importance of Antrix Corporation.
(c) Why is Website getting popular?
(d) What do the following stand for?
(i) C-CAD (ii)ICMR (iii) TRAI
(e) Give the functions of interferons.
(f) What is the difference between E-mail and Fax?




General Studies - 1999 (Main) (Paper - I)
Time Allowed : Three Hours Maximum Marks : 300
Candidates should attempt all questions from Section I, II and III.

SECTION I

1. How did Communalism manifest in Indian political scene? Explain the background of the
passing of the momentous Pakistan Resolution. (About 250 words) 35
OR
What were the proposals of the Cabinet Mission ? Analyse there actions of the Congress and the
League to the proposals. (About 250 words) 35
2. Answer any two of the following : (About 150 words on each)
(a) Trace the formation of the Swaraj Party. What were its demands.
(b) 'What began as a flight for religion ended as a war of independence, for there is not the slightest
doubt that the rebels wanted to get rid of the alien government and restore the old order of which
the king of Delhi was the rightful representative.' Do you support this viewpoint?
(c) How far Curzon's policy towards Tibet was influenced by strategic consideration.
15x2=30
3. Answer any three of the following : (About 75 words on each)
(a) Assess the contributions of Ishwarchandra Vidyasagar to the making of modern India.
(b) In what way did Ramakrishna infuse a new vigour and dynamism into Hinduism ?
(c) 'Tagore's poetry is a written record of his religious experience.' Elucidate.
(d) How did Nehru's plan for modernization make rapid stride during the decade 1951-61?
10x3=30
4. (a) Write about the chief features of the following : (About 20 words on each)
(i) Chola architecture (ii) Baisakhi
(iii) Neo-art movement
(b) What do you know about the following ?
(iv) Munda Movement
(v) Indian Association for the Cultivation of Science
(vi) Ilbert Bill (vii) Sarda Act
(viii) Education Despatch of 1854
(c) Why have the following become famous ?
(ix) Rammanohar Lohia (x) C. Y. Chintamani
(xi) Henry Cotton (xii) Tej Bahadur Sapru
(xiii) Veeresalingam P.K. (xiv) Bhulabhai Desai
(xv) Kamladevi Chattopadhyay 2 x 15 =30
SECTION II
5. What are the highlights of the draft nuclear doctrine formulated by the National Security


Advisory Board ? What shortcomings have been pointed out in media reports relating to the same?
(About 250 words) 35
OR
What are the reasons for the recent diplomatic confrontation between the U.S.E. and China? What
specific measures have been taken by the U.S.E. to defuse the tension building up the Taiwan
strait? (About 250 words) 35
6. Attempt any three of the following : (About 100 words on each)
(a) What new type of satellite is proposed to be inducted for bolstering surveillance on Indias
boarder? How does this type of satellite compare with existing IRS satellites?
(b) Explain in brief the observations made by the leaders of the Group of Eight countries in their
meeting in Cologne on Kargil intrusion by Pakistan.
(c) What are the major provisions of the New Telecom Policy approved by the Government? What
has been the decision of the Delhi High Court in this respect?
(d) Analyse the recent French response to Indias diplomatic efforts for promoting Indo-French
friendship. 10 x 3 =30
7. Attempt any five of the following : (About 50 words on each)
(a) What was the rationale for the recourse to air strikes by the Indian Air Force during Operation
Vijay ?
(b) What is Deep-water Oil Exploration? What are its prospects in India?
(c) Has India the capability to make a Neutron Bomb?
(d) What percentage of GDP India spends of defence? To what extent defence expenditure should
be increased ?
(e) What is Chandra ? For what purpose it has been launched?
(f) What is the most important provision of the Wye River Accord?
(g) What is Al Khalid ? What are its features? 6 x 5=30
8. (a) Who are the following ? Why have they been in the news ? (About 20 words on each)
(i) Dr. Sam Nujoma (ii) Mahendra Chaudhry
(iii) Colonel Eileen Collins (iv) Dr. Neelan Thiruchelvam
(v) David Ogilvy (vi) Jyotirmoyee Sikdar
(b) What do you know about the following?
(vii) Marthasvineyard (viii) Mianmari Creek
(ix) Karabash
(c) Name the authors of the following books ?
(x) Three Horsemen of the New Apocalypse
(xi) My Childhood Days
(d) Answer the following :
(xii) Who became the highest wicket taker in the World Cup 1999 ?
(xiii) Which film won the Best film award at the Cannes International Film Festival this


year? Name the directors.
(xiv) Which film won the Special Jury Award for Experimental Cinema at the 46
th
National
Film Festival? Name the director.
(xv) Who won the 1999 French open Doubles title? 2 x
15=30
SECTION III
9. Enrolments in different faculties, in the Universities of a state, for the period 1993-94 to 1995-96
are described below :
Enrolment in Arts faculty during the three sessions were 1,20,605; 1,35,100; and 1,50,000
respectively. In Commerce, Science, Law and Education faculties the enrolment in the session
1993-94 were 90,000; 60,500; 50,500 and 45,600 respectively. During the next two sessions the
enrolment figures in the Science faculty were 65,210 and 70,300 while in Law and Education
faculties the corresponding figures were 55,000; 60,000 and 48,000; 50,000respectively. In
Commerce faculty the enrolment during 1994-95 and 1995-96 were 1,15,000 and 1,22,250.
(a) Represent the information in tabular form. 10
(b) From your table calculate the increase in total enrolment from 1993-94 to 1995-96. 2
10.(a) A pie chart is to be constructed to represent the distribution of total budget of a government.
The total budget is Rs. 7200 crores. Out oj this Rs. 360 crores is budgeted for Agriculture, Rs. 90
crores for irrigation, Rs. 90 crores for Education, Rs. 540 crores for Defence, Rs. 72 crores for
Health and Rs. 108 crores for Industry. The remaining amount is budgeted for other
(miscellaneous) heads. Provide the percentages and angle measures of all the above heads in a
tabular form. 7
(b) In a housing complex there are 20 families. A family having annual income of Rs. 1.8 lacs
leaves and another with annual income of Rs. 3.0 lacs moves in. What change is caused to the
average monthly income of the housing complex due to this replacement? 5
11. (a) Given below are the percentages of words of different lengths used by an author in his entire
work :
Number of letters in a
word
Percentage
1 4.5
2 16.0
3 21.5
4 24.0
5 12.0
6 7.0
7 5.0
8 4.0
9 3.0


10 2.5
11 0.4
12 0.1
(i) What is the total percentage of words of length 8 or more? 3 (ii) What
is the maximum number of letters contained the shortest 78% of the words? 4
(b) Name the most appropriate diagram to be used for representing the information in each of the
following cases :
(i) Scientific manpower in four consecutive years in four different fields of science.
(ii) Distribution of the number of children belonging to a school by their ages.
(iii) The percentages of persons serving in different branches of the army such as infantry,
artillary, signals, etc.
(iv) The production of wheat in India in each year from 1955 to 1984.
(v) The heights and weights of the 50 children in a high school class. 5
12. (a) A group of freedom fighters met in 1998 and their average age was 81.7 years. The
survivors of this group met again in 1999 and their average age was 81.0 Explain why has the
average decreased when everyone in the second group is one year older than what they were in
1998. 4
(b) Below is given the gross corporate sector output in four selected years at current prices and also
the inflation factor for these years. Calculate the gross output for these years adjusted for
inflation. 6
Year 1980-81 1985-86 1990-
91
1995-96
Out put at
current prices
(thousands of
crores)
28.6 49.1 126.6 305.64
Inflation factor 1.0 1.39 1.98 3.05
(c) In report it is quoted that 65% of the women and 30% of the men in a locality were against the
opening of a liquor store in their locality. Thus it is argued that, an overwhelming 95% of the total
population is against the opening of the liquor store. Is this argument justifiable? Give reasons.
2
(d) Six lots of twenty guinea-pigs each were administered carcinogenic drugs in an experiment.
After 6 weeks the percentage of survivors in the six lots were :
Lot No 1 2 3 4 5 6
Percentage
of
survivors
53 86 48 27 8 24

Is this data prima ffacie plausible? Give reasons for your answer. 2






General Studies - 1999 (Main) (Paper - II)
Candidates should attempt all ques. from Section I, II and III.
SECTION I

1. (a) In what ways is the RAJYA SABHA expected to play a special role in today's changing
political scenario ? (About 250 words) 40
OR
(b) On what grounds does Article 15 of the Indian Constitution prohibit discrimination ? Indicate
the way the concept of 'Special protection' has qualified this prohibition, and contributed to social
change. (About 250 words) 40

2. (a) What is 'new' in India's New Industrial Policy of 1991 ? (About 250 words) 40
OR
(b) What is Multilateral Agreement on Investment? How will it affect the economy of a developing
country like India ? (About 250 words) 40
3. (a) Discuss the role of modern technological inputs on agricultural development in India. How
would it be helpful for food security during 21st century? (About 250 words) 40
OR
(b) What is a fast breeder reactor? Comment on its suitability in the Indian context.
(About 250 words) 40
4. Answer any TWO of the following : 20+20 =40
(Answer to each question should be in about 150 words)

(a) What is the importance of Directive Principles of State Policy ? Mention which Directive
Principles of State Policy have got primacy over the Fundamental Rights.

(b) Discuss the composition and functions of the National Security Council.
(c) Highlight the significance of the Twenty Forth Amendment to the Constitution of India ?
(d) Access the importance of the role played by the Public Accounts Committee.
SECTION II
5. Answer any TWO of the following :- 20+20 =40
(Answer to each question should be in about 150 words) :
(a) What does buy-back of shares mean ?
(b) When and why was SEBI established?
(c) What is SIDBI ? What are its main functions?
(d) What is Tobin tax ? What are its main features?
6. Answer any TWO of the following :- 15+15 =30
(Answer to each question should be in about 150 words) :
(a) How is deforestation of Himalayas disturbing the ecological balance of north India?


(b) What is dry farming? Discuss its relevance in augmenting the food supply in India.
(c) Name the mega cities of India and mention their specific problems.
(d) What is the importance of Indian Ocean for India?
7. Answer any TWO of the following :- 20+20 =40
(Answer to each question should be in about 150 words) :
(a) What is the Human Genome Project? Discuss briefly its importance.
(b) What is carbon dating? Describe its applications in archaeology.
(c) What is ozone hole? How is it caused and what are its implications for life on Earth ?
(d) What are biosensors? Describe their uses.
SECTION III
8. Answer any FOUR of the following :- 3 x 4 =12
(about 25 words each) :
(a) What is 'Swatch of no ground' ?
(b) From where was the most advanced commercial satellite of India Insat 2E launched?
(c) What is Global Positioning System (G.P.S.) ?
(d) What is Cash Reserve Ratio?
(e) How is revenue deficit different from budgetary deficit?
(f) What is Plan Holiday?
9. Answer any THREE of the following :- 3 x 3 = 9
(Answer to each question should be in about 25 words):
(a) Who presides over the Joint-Session of the two Houses of the Indian Parliament over a non-
memory bill?
(b) Is there any provision to impeach the Governor of a State?
(c) In case of any dispute whether a bill is a money bill or not, whose dicision is final?
(d) How is the vice-President of India elected?
(e) What is the status of the Right to Property in the Indian Constitution?
(f) What is the maximum gap between two sessions of the Indian Parliament?
10. Answer any THREE of the following :- 3 x3 =9
(About 25 words each):
(a) What is a modem? What is it used for?
(b) What is CNG ? How can it reduce air pollution?
(c) What is a computer virus? How does it infect systems?


(d) What do the following stand for?
(i) NISSAT (ii) GMRT (iii) INMAS

(e) Where are the following located?
(i) Centre for Cellular and Molecular Biology
(ii) Indira Gandhi Centre for Atomic Research
(iii) Vikram Sarabhai Space Centre

(f) How do identical twins differ from each other genetically?




General Studies - 2000 (Main) (Paper - 2)

Time Allowed : Three Hours Maximum Marks : 300
INSTRUCTIONS
Each Question is printed both in Hindi and in English.
Answers must be written in the medium specified in the Admission Certificate issued to you, which
must be stated clearly on the cover of the answer-book in the space provided for the purpose. No
mark will be given for answers written in a medium other than that specified in the Admission
Certificate. Candidates should attempt ALL questions strictly in accordance with the instructions
given under each question. The number of marks carried by each question is indicated at the end of
the question.
Q. 1 . Answer any two of the following. (Answer to each question should be in about 150
words) : - 15 x 2 = 30
(a) Discuss Indian-Sri Lankan relations in recent years.
(b) India's Kargil victory rested on its successful effort at combining diplomacy with the use of
force. Discuss.
(c) Differentiate and state the significance of Pokharan I and Pokharan II.
Q. 2. Answer the following (Answer to each question should be in about 20 words)
2 x 5 = 10
(a) What does track two diplomacy stand for ?
(b) Non-alignment is a need, not a creed, Elaborate.
(c) Differentiate between Peace-making and Peace-keeping.
(d) What is deterrence ?
(e) Why has the indigenous development of Cryogenic Enging become necessary for India ?
Q. 3. Answer the following. (Answer to each question should be in about 20 words)
2 x 5 = 10
(a) What does the Global organisation of people of Indian origin stand for ?
(b) What is the impact of economic sanctions against India on NRIs ?
(c) Why is Fiji crisis of May 19, 2000 a specific concern for India ?
(d) Examine the role of NRIs in the liberalisation process of India.
(e) Mention few NRIs who have brought name and fame for India.
Q. 4. Answer anyone of the following. (Answer should be in about 250 words) 30
(a) India is rapidly emerging as an Information Technology (IT) Superpower. Discuss some aspects
of the growth of this Sector in the Indian economy. What role can public policy play in further
enhancing growth prospects in this Sector ?
(b) Control over growth of population in India is an essential condition for the country's rapid
economic development. Discuss.
Q. 5. Answer any two of the following. (Answer to each question should be in about 150 words)
: 2 x 15 = 30
(a) Discuss the reasons for the failure of the Seattle Millennium talks on the WTO (World Trade
Organisation). Discuss some implications of this failure for the Indian economy.
(b) What is (Revised) Targeted Public Distribution System ? What are its main features ?
(c) Discuss the economic effects of Black money (Parallel economy) in Indian economy.


Q. 6. Answer the following. (Answer to each question should be in about 20 words)
2 x 15 = 30
(a) What are the main objectives of the 9th Five-Year Plan of the Government of India ?
(b) Write a note on MODVAT Scheme of 1986.
(c) Explain per capita income as a measurement of economic growth.
(d) What are the objectives of Social Security ?
(e) What do you mean by 'Parallel Economy' ?
(f) Describe the use of 'Command Area Development' in India.
(g) What is ICOR (Incremental Capital Output Ratio) ?
(h) What are the main objectives of NABARD ?
(i) What do you know about 11th Finance Commission ?
(j) What do you mean by revenue deficit in the Central Government's Budget ?
(k) How has the Census (1991) defined the Urban Areas ?
(1) What are the objectives of New Economic Policy of the Government of India ?
(m) Write a note on Rao-Manmohan model of development.
(n) What is the rational for 'Mid-day Meal' Scheme ?
(o) What has been the impact of the recent economic reforms programme on the incidence of
poverty in India ?
Q.7. Answer any two of the following. (Answer to each questions should be in about 150 words)
: - 15 x 2 = 30
(a) Define Globalisation and differentiate it with Internationalism.
(b) There is a wide-spread concern about the degradation of environment and the problems that go
with it. Assess the international response.
(c) How can India accommodate her foreign policy principle of 'non-intervention' with the
emerging right to 'intervention'
Q. 8. Answer the following. (Answer to each question should be in about 20 words)
2 x 5 = 10
(a) What are the prospects of CTBT ?
(b) Differentiate between Collective security and Cooperative security.
(c) What do the following stand for ?
(i) MARV
(ii) MIRV
(iii) ICBM
(d) Differentiate between Explosion and Implosion.
(e) What do you know about Siachin Dispute ?
Q. 9. Answer the following (Answer to each question should be in about 20 words)
2 x 5 = 10
(a) What do the following stand for ?
(i) SAARC
(ii) ASEAN
(iii) OIC
(b) What is meant by SDI ?
(c) Differentiate between SALT and START.
(d) What do you know about G-77 ?
(e) What is Global Environmental Facility (GEF) ?
Q.10. Answer any one of the following. (Answer should be in about 250 words) 30
(a) What is the significance of the geostationary orbit ?What would happen if it becomes too


crowded ?
(b) Why is the HIV considered so dangerous ? Why has the spread of the virus proved so difficult
to contain, especially in the poorer countries of the world ?
Q. 11. Answer any two of the following : (Answer to each question should be in about 150
words) : 15 x 2 = 30
(a) How are transgenic plants different from hybrid plants and what is their relevance in modern
agriculture ? Elaborate.
(b) What are greenhouse gases ? What impact do they have on the Earth's climate and with what
consequences ? Elaborate.
(c) What is water harvesting ? Describe its relevance in the Indian context.
Q. 12. Answer the following : (Answer to each question should be in about 20 words)
2 x 5 = 10
(a) What does the 'clock speed' of a computer signify ?
(b) What do the following stand for ?
(i) http
(ii) CPU
(iii) ROM
(vi) BIOS
(c) What is a modem and what is it used for ?
(d) What is spamming ?
(e) How does a computer virus destroy data ?
Q. 13. Answer the following :
(a) The table below shows the number of students in different faculties of a University :
Year Number of Students Total
Humanities Science Commerce
1975-76 2810 890 540 4240
1976-77 3542 1363 471 5376
1977-78 4301 1662 652 6615
1978-79 5362 2071 895 8328
1979-80 6593 2752 1173 10518

Represent the data by a suitable diagram. 8
(b) The pie chart below gives the annual production of certain commodities of a State.

If the production of sugar is 3000 tons, find the total production of rice and wheat. 8


Q. 14. Answer the following :
(a) The number of overtime hours worked by 1000 employees of a company during the last six
months is as follows : 4 x 2 = 8
Overtime
(in hours)
Number of
Employees
0-20 401
20-40 226
40-60 182
60-80 74
80-100 43
100-120 38
120-140 20
140-160 9
160-180 5
180-200 2
(i) What is the percentage of employees having worked less than 40 overtime hours during these
months ?
(ii) What is the percentage of employees having worked for at least 40 overtime hours but less than
100 overtime hours ?
(b) A certain company offered its shares, of par-value Rs 120, at a premium of Rs 20. In the
following year a dividend of 21% was declared by the company. Find the rate of interest at the
hands of the investor. 8
Q. 15. Answer the following : 2 x 4 = 8
(a) A farmer has a square plot for cultivation. If the length of each side is increased by 10%, then
the yield from the plot will increase by 20%. Is it true ? Give reason.
(b) A company increased its production 8 times in a period of 3 years. What is the percentage
increase per year ?
(c) The price of a commodity fell by 25% and then rose by 33.33%. Can we conclude that the
current price is 8.33% more of the original price ? Give reason.
(d) A group of 150 workers have Rs 72.60. When both the groups are combined, what be the
average daily wage ?




Indian Polity Section Mains Analysis

! Year 2001 Percentage: 33.3% (100/300)






! Year 2000 Percentage: 33.3% (100/300)
Questions Appeared
Q. 6. Answer any ONE of the following (About 250 words):- 30
(a) Examine the need for the review of the Indian Constitution.
(b) Examine the demand for greater State-antonomy and also its impact on the smooth
functioning of the Indian polity.
Q. 7. Answer any ONE of the following (About 250 words) :- 30
(a) How does Parliament control the Union Executive ? How effective is its control ?
(b) What constitutes the doctrine of 'basic features' as introduced into the Constitution of
India by the Judiciary ?
Q. 8. Answer any TWO of the following (About 150 words each) :- 15 x 2 = 30
(a) Identify, the major electoral reforms which are necessary in the Indian Political System.
(b) Examine the role of Estimates Committee.
(c) Discuss the major extra-constitutional factors influencing the working of federal polity
in India.
Q. 9. Answer the following (About 20 words each) :- 5 x 2 = 10
(a) What is vote on account ?
(b) What is a Caretaker Government ?
(c) Do you justify the Prime Minister's entry into Parliament through the Rajya Sabha ?
(d) What is a Privilege Motion ?
(e) What is Contempt of Parliament ?


! Year 1999 Percentage: 30% (89/100)
Questions Appeared
1. (a) In what ways is the RAJYA SABHA expected to play a special role in today's
changing political scenario ? (About 250 words) 40
OR
(b) On what grounds does Article 15 of the Indian Constitution prohibit discrimination ?
Indicate the way the concept of 'Special protection' has qualified this prohibition, and
contributed to social change. (About 250 words)

4. Answer any TWO of the following : 20+20 =40
(Answer to each question should be in about 150 words)
(a) What is the importance of Directive Principles of State Policy ? Mention which
Directive Principles of State Policy have got primacy over the Fundamental Rights.
(b) Discuss the composition and functions of the National Security Council.
(c) Highlight the significance of the Twenty Forth Amendment to the Constitution of India ?
(d) Access the importance of the role played by the Public Accounts Committee.
9. Answer any THREE of the following :- 3 x 3 = 9
(Answer to each question should be in about 25 words):
(a) Who presides over the Joint-Session of the two Houses of the Indian Parliament over a
non-memory bill?


(b) Is there any provision to impeach the Governor of a State?
(c) In case of any dispute whether a bill is a money bill or not, whose dicision is final?
(d) How is the vice-President of India elected?
(e) What is the status of the Right to Property in the Indian Constitution?
(f) What is the maximum gap between two sessions of the Indian Parliament?




! Year 1998 Percentage: 30% (90/300)
Questions Appeared
1. (a) Briefly state the stages through which the present position of the Directive Principles
vis--vis the Fundamental Rights has emerged. (About 250 words) 40
OR
(b) Explain the concept of Prime Ministerial Government and account for its decline in
recent times in India. (About 250 words)

4. Answer any TWO of the following :-
(Answer to each question should be in about 150 words) 20+20
(a) Highlight the significance of the Seventy-third Amendment to the Constitution of India.
(b) How are the States formed in India? Why have the demands of separate States like those
of Vidarbha, Telangana etc. not been considered by the Government recently?
(c) On what grounds the Legislative Councils are justified? How is it created or abolished in
a State?
(d) Differentiate and state the significance of general election, mid-term election and by-
election.

8. Answer THREE question 10
(Answer to each question should be in about 25 words)
(a) What is meant by Protem Speaker?
(b) Point out the constitution and functions of the Central Vigilance Commission.
(c) State the amplitude of Art. 21 of the Constitution.
(d) Which of the cases regarding disqualifications for Membership of either House of
Parliament are decided by the President?
(e) Differentiate between Parliamentary Secretary and Lok Sabha Secretary.
(f) What is a Privilege Motion?






General Studies - 2000 (Main) (Paper - I)
Time Allowed : Three Hours Maximum Marks : 300
INSTRUCTIONS
Each Question is printed both in Hindi and in English.
Answers must be written in the medium specified in the Admission Certificate issued to you, which
must be stated clearly on the cover of the answer-book in the space provided for the purpose. No
mark will be given for answers written in a medium other than that specified in the Admission
Certificate. Candidates should attempt ALL questions strictly in accordance with the instructions
given under each question. The number of marks carried by each question is indicated at the end of
the question.
Q.1. Answer any ONE of the following (About 250 words) : - 30
(a) Trace the origin of the Swadeshi Movement. How did it involve the masses
(b) What was Mountbatten Plan ? Discuss the reactions of Gandhi and Azad to the Plan.
Q. 2. Answer any TWO of the following (About 150 words each):- 15 x 2 = 30
(a) Who established the Arya Samaj ? What was its goal ?
(b) How did the terrorist movement gather strength in countries other than India?
(c) Was Jawaharlal Nehru justified in adopting the principle of non-alignment as the corner-stone
of India's foreign policy ?
Q. 3. Answer the following (About 20 words each) : - 2 x 15 = 30
(i) Brahmagupta
(ii) Amir Khusro
(iii) William Jones
(iv) C.F. Andrews
(v) Narayan Guru
(vi) Tantia Tope
(vii) Sayyid Ahmad
(viii) Margaret Noble
(ix) Sangam Literature
(x) Gandhara School of art
(xi) Granth Sahib
(xii) Fort William College
(xiii) Epsom
(xiv) Lala Amarnath
(xv) Tiger Woods.
Q. 4. Answer any TWO of the following (About 125 words each):- 2 x 10 = 20
(a) Mention the agro-climatic regions of India stating the basis of classification.
(b) Explain the major racial groups of India.
(c) What is waste land ? Write a note on prospects of waste land development in India.
Q. 5. Write notes on the following (About 20 words each) :- 5 x 2 = 10
(i) Linguistic regions of India
(ii) Nagarjunasagar Project
(iii) Mango Showers
(iv) Multi-level planning in India
(v) India's Laterite soils.


Q. 6. Answer any ONE of the following (About 250 words):- 30
(a) Examine the need for the review of the Indian Constitution.
(b) Examine the demand for greater State-antonomy and also its impact on the smooth functioning
of the Indian polity.
Q. 7. Answer any ONE of the following (About 250 words) :- 30
(a) How does Parliament control the Union Executive ? How effective is its control ?
(b) What constitutes the doctrine of 'basic features' as introduced into the Constitution of India by
the Judiciary ?
Q. 8. Answer any TWO of the following (About 150 words each) :- 15 x 2 = 30
(a) Identify, the major electoral reforms which are necessary in the Indian Political System.
(b) Examine the role of Estimates Committee.
(c) Discuss the major extra-constitutional factors influencing the working of federal polity in India.
Q. 9. Answer the following (About 20 words each) :- 5 x 2 = 10
(a) What is vote on account ?
(b) What is a Caretaker Government ?
(c) Do you justify the Prime Minister's entry into Parliament through the Rajya Sabha ?
(d) What is a Privilege Motion ?
(e) What is Contempt of Parliament ?
Q.10. Answer any ONE of the following (About 250 words) :- 30
(a) Discuss the provisions of the Human Rights Protection ACt (1993) relating to the following :-
(i) Definition of human rights.
(ii) Composition of the National Human Rights Commission.
(iii) Functions of the Commission.
(iv) What suggestions have been made for amending the Act for making the role of the NHRC
more effective ?
(b) Discuss the propositions laid down by the Supreme Court of India in the context of Torture in
Prisons and Human Dignity.
Q. 11. Write on any TWO of the following (About 125 words each) :- 2 x 10 = 20
(a) Explain Public Interest Litigation. Who can file it and on what basis can it be rejected by the
Court of Law ?
(b) Discuss Human Genome.
(c) What are the features of the Uttar Pradesh Regulation of Public Religious Buildings and Places
Bill, 2000, that have caused widespread protests from minorities ?
Q. 12. Write notes on any TWO of the following (About 125 words each) : 2 x 10 = 20
(a) Universal Postal Union
(b) Economic and Social Council
(c) Florence Nightingale.
Q.13. Write about the following (About 20 words each) :- 2 x 5 = 10
(a) CRY
(b) Baba Amte
(c) Ali Sardar Jafri
(d) 'Water'
(e) Aruna Roy.




General Studies - 1997 (Prelims)
Time Allowed : Two Hours Maximum Marks : 150

1. About 50% of the world population is concentrated between the latitudes of
(a) 5N and 20N
(b) 20N and 40N
(c) 40N and 60N
(d) 20S and 40S

2. A rectangular plot of lawn shown in the figure has dimensions `x' and `y' and is surrounded by a
gravel pathway of width 2m. What is the total area of the pathway?

(a) 2x + 2y + 4
(b) 2x + 2y + 8
(c) 4x+4y+8
(d) 4x+4y+16

3. After consolidating his power, Balban assumed the grand title of
(a) Tute-i-Hind (b) Kaisr-i-Hind (c) Zil-i-Ilahi (d) Din-i-Ilahi
4. Match List I with List II and select the correct answer:
List I List II
(Climatic conditions) (Reasons)
A. Madras is warmer than Calcutta 1. North-East Monsoon
B. Snowfall in Himalayas 2. Altitude
C. Rainfall decreases from 3. Western depressions
West Bengal to Punjab
D. Sutlej-Ganga Plain gets 4. Distance from sea
some rain in winter 5. Latitude
A B C D A B C D
(a) 1 2 4 5 (b) 4 5 1 3
(c) 5 2 4 3 (d) 5 1 3 4
5. The economic and monetary union of 15 European countries is proposed to be made by 1999.
But the currencies of two countries, franc, have already the same value and circulate freely in both
the countries. The countries are
(a) France and Switzerland
(b) Switzerland and Luxembourg
(c) Luxembourg and Belgium
(d) France and Belgium


6. The average rate of domestic savings (gross) for the Indian economy is currently estimated to be
in the range of
(a) 15 to 20 percent (b) 20 to 25 percent
(c) 25 to 30 percent (d) 30 to 35 percent

7. The tendency of a liquid drop to contract and occupy minimum area is due to
(a) surface tension (b) viscosity
(c) density (d) vapour pressure

8. The pituitary gland by virtue of its tropic hormones controls the secretary activity of other
endocrine glands. Which one of the following endocrine glands can function independent of the
pituitary gland ?
(a) Thyroid (b) Gonads
(c) Adrenals (d) Parathyroid
9. The average monthly income of a person in a certain family of 5 is Rs. 1000/-. What will be the
monthly average income of a person in the same family if the income of one person increased by
Rs. 12000/- per year ?
(a) 1200/- (b) 1600/-
(c) 2000/- (d) 3400/-
10. Match List I with List II and select the correct answer:
List I List II
A. Butler Committee 1. Jallianwala Bagh
Report massacre
B. Hurtog Committee 2. Relationship between
Report the Indian States and Paramount Power
C. Hunter Inquiry 3. Working of Dyarchy as laid down in the Mont
Committee Report - agu-Chelmsford Reforms
D. Muddiman 4. The growth of education in British India and potenti
Committee Report -alities of its further progress
A B C D A B C D
(a) 3 2 1 4 (b) 1 4 2 3
(c) 2 1 3 4 (d) 2 4 1 3
11. Match List I with List II and select the correct answer:
List I List II
A. Deccan Traps 1. Late cenozoic
B. Western Ghats 2. Pre-cambrian
C. Arayalli 3. Cretaceous Eocene
D. Narmada-Tapi 4. Cambrian
alluvial deposits 5.Pleistocene
A B C D A B C D
(a) 3 5 1 4 (b) 3 1 2 5
(c) 2 1 3 4 (d) 1 4 2 5

12. The number of industries for which industrial licensing is required has now been reduced to
(a) 15 (b) 6 (c) 35 (d) 20
13. Which one of the following is a vector quantity ?
(a) Momentum (b) Pressure


(c) Energy (d) Work

14. Oxygen transportation in a human body takes place through
1. Blood. 2. Lungs. 3. Tissue.
The correct sequence of transportation is
(a) 1, 2, 3 (b) 3, 1, 2 (c) 2, 1, 3 (d) 1, 3, 2
15. The characteristic ordour of garlic is due to
(a) a chloro compound
(b) a sulpher compound
(c) a fluorine compound
(d) acetic acid

16. The head of the military department under the recognized central machinery of administration
during Akbar's reign was
(a) Diwan (b) Mir Bakshi
(c) Mir Saman (d) Bakshi

17. The January isotherm taken as a basis for dividing India into tropical and subtropical zones is
(a) 21C (b) 18C (c) 12C (d) 15C
18. In the given figure, if QRS is an equilateral triangle and QTS is an isosceles triangle and x =
47. then the value (in degrees) of y will be
(a) 13 (b) 23 (c) 33 (d) 43


19. "..... They are people of yellow complexion, oblique eyes, high check bones, sparse hair and
medium height." The reference here is to
(a) Nordic Aryans (b) Austrics (c) Negroids (d) Mongoloids
20. What is the correct sequence of the descending order of the following States in respect of
female literacy rates as per the 1991 census ?
1. Mizoram 2. Kerala 3. Goa 4. Nagaland
Choose the correct answer using the codes given below:
(a) 2, 3, 1, 4 (b) 2, 1, 3, 4 (c) 2, 3, 4, 1 (d) 1, 2, 4, 3
21. The tail of a comet is directed away from the sun because
(a) as the comet rotates around the sun, the lighter mass of the comet is pushed away due to the
centrifugal force alone
(b) as the comet rotates, the lighter mass of the comet is attracted by some star situated in the
direction of its tail
(c) the radiation emitted by the sun 'exerts a radial pressure on the comet throwing its tail away
from the sun


(d) the tail of the comet always exists in the same orientation

22. Corpus lutcum is a mass of cells found in
(a) brain (b) ovary (c) pancreas (d) spleen
23. Given below are two statements, one labelled as Assertion (A) and the other labelled as Reason
(R).
Assertion (A) : The sponsor and the most prominent figure of the Chisti order of Sufis in India is
Khwaja Moinuddin Chisti.
Reason (R) : The Chisti order takes its name from a village Chisti in Ajmer.
In the context of the above two statements, which one of the following is correct ?
(a) Both A and Rare true and R is the correct explanation ' of A
(b) Both A and R are true but R is not the correct explanation of A
(c) A is true but R is false
(d) A is false but R is true
24. Match List I with List II and select the correct answer:
List I List II
(Minerals) (Typical areas of occurence)
A. Coal 1. Bhandara
B. Gold 2. Karanpura
C. Mica 3. Hutti
D. Manganese 4. Nellore
A B C D A B C D
(a) 1 3 2 4 (b) 2 3 4 1
(c) 3 4 2 1 (d) 2 1 4 3
25. In which one of the following countries will the no confidence motion to bring down the
government passed by the legislature be valid only when the legislature is able to find
simultaneously a majority to elect successor government ?
(a) France (b) Germany (c) Italy (d) Portugal
26. In the series POQ, SRT, VUW,------ , the blank space refers to
(a) XYZ (b) XZY (c) YXZ (d) YZX
27. The sum of which of the following constitutes Broad Money in India ?
1. Currency with the public.
2. Demand deposits with banks.
3. Time deposits with banks.
4. Other deposits with RBI.
Choose the correct answer using the codes given below:
(a) 1 and 2 (b) 1, 2 and 3
(c) 1, 2, 3 and 4 (d) 1, 2 and 4
28. A smooth inclined at an angle ! with the horizontal as shown in the above figure. A body starts
from rest and slides down the inclined surface. The time taken by the body to reach the bottom is






29. "A graduate at 18, professor and associate editor of the Sudharak at 20, Secretary of the
Sarvajanik Sabha and of the Provincial Conference at 25, Secretary of the National Congress at 29,
leading witness before an important Royal Commission at 31, Provincial legislator at 34, Imperial
legislator at 36, President of the Indian National Congress at 39 a patriot whom Mahatma Gandhi
himself regarded as his master". This is how a biographer describes
(a) Pandit Madan Mohan Malaviya
(b) Mahadev Govind Ranade
(c) Gopal Krishna Gokhale
(d) Bal Gangadhar Tilak

30. Which one of the following is an active component of oil of clove ?
(a) Menthol (b) Eugenol
(c) Methanol (d) Benzaldehyde

31. Consider the map given below
The river shown on the map is
(a) Irrawaddy (b) Mekong (c) Chao Phraya. (d) Salween


32. Which one of the following was NOT proposed by the 73rd constitutional amendment in the
area of Panchayati Raj ?
(a) Thirty per cent seats in all elected rural local bodies will be reserved for women candidates at all
levels
(b) The States will constitute their Finance Commissions to allocate resources to Panchayati Raj
institutions
(c) The Panchayati Raj elected functionaries will be disqualified to hold their offices if they have


more than two children
(d) The elections will be held in six months time if Panchayati Raj bodies are superceded or
dissolved by the State government

33. According to the latest census figures sex ratio in India is
(a) declining (b) stable (c) increasing (d) fluctuating
34. The most reactive among the halogens is
(a) fluorine (b) chlorine (c) bromine (d) iodine
35. Which one of the following organisms is likely to show the highest concentration of DDT once
it has been introduced into the ecosystem ?
(a) Grasshopper (b) Toad (c) Snake (d) Cattle
36. There are three drawers in a table. One contains two golds coins., another two silver coins, and
the third, a silver coin and a gold coin. One of the drawers is pulled out and a coin is taken out. It
turns out to be a silver coin What is the probability of drawing a gold coin, if one of the other two
drawers is pulled out next and one of the coins in it is drawn at random ?
(a) 37.5% ( b) 50% (c) 62,5% (d) 75%
37. Who among the following was associated with supression if Thugs ?
(a) General Henry Prendergast (b) Captain Sleeman
(c) Alexander Burnes (d) Captain Robert Pemberton
38. The Alamatti is on the river
(a) Godavari (b) Cauvery (c) Krishna (d) Mahanadi
39. Which one of the following States of India has passed a legislation (in 1996) making the
maintenance of one's parents mandatory ?
(a) Kerala (b) West Bengal (c) Maharashtra (d) Himachal Pradesh
40. Match List I with List II and select the correct answer:
List I (Events) List II (Results)
A. Morley Minto Reforms 1. Country-wise agitation
B. Simon commission 2. Withdrawal of a movement
C. The Chauri-Chaura incident 3. Communal Electorates
D. The Dandi March 4. Communal outbreaks
5. Illegal manufacture of salt
A B C D A B C D
(a) 3 4 5 2 (b) 4 1 2 3
(c) 2 3 4 5 (d) 3 1 2 5
41. In the following figure





P is 30 km eastward of O and Q is 400 km north of O. R is exactly in the middle of Q and P. The
distance between Q and R is
(a) 250 km (b) 300 km (c) 350 km (d) 250 X (km)
1/2


42. The Poona Pact which was signed between the British Government and Mahatma Gandhi in
1934 Provided for
(a) the creation of dominion status for India (b) separate electorates for the Muslims
(c) separate electorate for the Harijans (d) joint electorate with reservation for Harijans

43. Which one of the following was used as a chemical weapon in the First World War ?
(a) Carbon monoxide (b) Hydrogen cyanide (c) Mustard gas (d) Water gas

44. Proportional representation is NOT necessary in a country where
(a) there are no reserved constituencies
(b) a two-party system has developed
(c) the first-past-post system prevails
(d) there is a fusion of presidential and Parliamentary forms of government

45. Match List I with List II and select the correct answer
List I (Committees) List II (Chaired by)
A. Disinvestment of shares in Public Sector Enterprises 1. Rajah Chelliah
B. Industrial Sickness 2. Onkar Goswami
C. Tax Reforms 3. R. N. Malhotra
D. Reforms in Insurance Sector 4. C. Rangarajan
A B C D A B C D
(a) 1 4 2 3 (b) 4 2 1 3
(c) 4 1 2 3 (d) 1 3 4 2
46. Alpha-keratin is a protein present in
(a) blood (b) skin (c) wool (d) eggs
47. Which one of the following is paramagnetic in nature?
(a) Iron (b) Hydrogen (c) Oxygen (d) Nitrogen

48. When three coins are tossed together the probability that all coins have the same face up is
(a) 3 (b) 6 (c) 8 (d) 12
49. Which one of the following factors is responsible for the change in the regular direction of the
ocean currents in the Indian Ocean ?
(a) Indian Ocean is half an ocean (b) Indian Ocean has Monsoon drift
(c) Indian Ocean is a land-locked ocean (d) Indian Ocean has greater variation in salinity

50. Match List I with List II and select the correct answer:
List I List II
A. Surat Split 1. 1929
B. Communal Award 2. 1928
C. All Party Convention 3. 1932
D. Poorna Swaraj Resolution 4. 1907
5. 1905
A B C D A B C D


(a) 4 3 1 5 (b) 4 3 2 1
(c) 2 5 4 1 (d) 1 4 2 3
51. Consider the map given below

The predominant languages spoken in the areas marked A B, C and D are respectively
(a) Brajbhasha, Bhojpuri, Maithili and Awadhi
(b) Awadhi, Maithili, Bhojpuri and Brajbhasha
(c) Brajbhasha, Awadhi, Bhojpuri and Maithili
(d) Maithili, Bhojpuri, Brajbhasha and Awadhi

52. The number of students in two sections, A and B having different heights is shown in the
following Table
Number of students with that
height
Height
(in metres)
In Section A in Section B
1.55
1.60
1.62
1.65
1.68
1.71
1.75
3
7
12
15
8
6
3
2
6
14
14
9
5
4
The ration of the number of students of a particular height in Section A to that in Section B is the
maximum for the height of
(a) 1.55 m (b) 1.60 m (c) 1.65 m (d) 1.71 m
53. The canal-carrying capacity of Farrakka is
(a) 75,000 Cusecs (b) 70,000 Cusecs
(c) 40,000 Cusecs (d) 35,000 Cusecs

54. The Prime Minister of which one of the following countries is chosen by the ruling prince from
a slate of three candidates put up to him by the President of France ?
(a) San Marius (b) Liechtenstein (c) Malta (d) Monaco

55. One of the important agreements reached in the 1996 Ministrial Conference of WTO refers to
(a) commerce in Information Technology


(b) Multilateral Agreement on Investment
(c) Multi-fibre Agreement
(d) exchange of technical personal

56. Which one of the following has the highest fuel value?
(a) Hydrogen (b) Charco (c) Natural gas (d) Gasoline
57. Match List I with List II and select the correct answer:
List I List II
A. Chittagaon Armoury raid 1. Lala Hardayal
B. Kakori Conspiracy 2. Jatin Das
C. Lahore Conspiracy 3. Surya Sen
D. Ghadr Party 4. Ram Prasad Bismil
5. Vasudeo Phadke
A B C D A B C D
(a) 3 4 1 5 (b) 4 3 2 5
(c) 3 4 2 1 (d) 2 4 3 1

58. Which one of the following pairs is correctly matched?
(a) Teak : Jammu and Kashmir
(b) Deodar : Madhya Pradesh
(c) Sandalwood : Kerala
(d) Sundari : West Bengal

59. Which one of the following is cultivated by transplanting seedlings ?
(a) Maize (b) Sorghum (c) Onion (d) Soyabeen
60. Consider the man given below


The places marked A, B, C and D in the map are respectively
(a) Rift valley region, Chattisgarh plain, Rain shadow region and Chotanagpur plateau
(b) Chattisgarh plain, Chotanagpur plateau, Rift valley region, and Rain shadow region
(c) Rift valley region, Chattisgarh plain, Chotanag-pur plateau and Rain shadow region
(d) Chattisgarh plain, Rain shadow region, Chota-nagpur plateau and Rift valley region



61. Match List I with List II and select the correct answer:
List I (Crops) List II (Geographical conditions)
A. Barley 1. Hot and dry climate with poor soil
B. Rice 2. Cool climate with poorer soil
C. Millets 3. Warm and Moist climate with high attitude
D. Tea 4. Hot and Moist climate with rich soil
A B C D A B C D
(a) 2 4 1 3 (b) 3 4 1 2
(c) 2 1 4 3 (d) 3 2 4 1

62. The world's fastest computer has been able to perform (as of Dec. 1996)
(a) 10
6
operations per second (b) 10
9
operations per second
(c) 10
12
operations per second (d) 10
15
operations per second
63. The Basque separatist organization is active in
(a) Russia (b) Cyprus (c) Portugal (d) Spain

64. The following table shows the per cent change in the amount of sales (in rupees) at different
retail stores in a given neighborhood market in the period 1993 to 1995:
Percent Change Retail Store
1993 to 1994 1994 to 1995
Anshu
Borna
Calpo
Dilip
Elegant
+10
-20
+5
-7
+17
-10
+9
+12
-15
-8
If the sales at Anshu store amounted to Rs. 8 Lakhs in 1993, then the amount of sales (in lakhs of
rupees) at that store in 1995 was
(a) 7.92 (b) 8.00 (c) 8.80 (d) 9.68

65. The planet mercury is revolving in an elliptical orbit around the sun as shown in the given
figure. The kinetic energy of mercury is greatest at the point labelled

(a) A (b) B (c) C (d) D

66. Match List I with List II and select the correct answer:
List I List II
A. Gupta 1. Badami
B. Chandella 2. Panamalai
C. Chalukya 3. Khajuraho
D. Pallava 4. Deogarh


A B C D A B C D
(a) 4 3 1 2 (b) 4 2 3 1
(c) 2 3 4 1 (d) 3 4 1 2

67. Consider the map given below

The places marked A, B, C and D in the map are respectively known for the cultivation of
(a) groundnut, ragi, tobacco and sugarcane
(b) groundnut, sugarcane, ragi and tobacco
(c) ragi, sugarcane, groundnut and tobacco
(d) ragi, groundnut, sugarcane and tobacco
68. The film The Making of the Mahatma has been directed by
(a) Peter Ustinov (b) Richard Attenborough
(c) Shyam Benegal (d) Mira Nair
69. "From Aceh in the far northwest to Torres Strait in the east is 5000 miles, almost as far as from
London to Baghdad. The archipelago has 14,000 islands, some mere equatorial rocks, others some
of the largest in the world." This description best fits
(a) West Indies (b) Japan (c) Philippines (d) Indonesia
70. Human Development Index comprises literacy rates, life. expectancy at birth and
(a) Gross Domestic Product per head in the US dollars
(b) Groos Domestic Product per head at real purchasing power
(c) Gross National Product in US dollars
(d) National Income per head in US dollars
71. A major health mission carried out in 1996 was
(a) war against leprosy (b) small-pox eradication
(c) pulse polio immunization (d) popularisation of oral rehydration therapy
72. If the numbers representing volume and surface area of a cube are equal, then the length of the
edge of the cube in terms of the unit of measurement will be
(a) 3 (b) 4 (c) 5 (d) 6
73. Which one of the following statements regarding Asokan stone pillars is incorrect ?
(a) These are highly polished
(b) These are monolithic


(c) The shaft of pillars is tapering in shape
(d) These are parts of architectural structures
74. State funding of elections takes place in
(a) U.S.A. and Canada (b) Britain and Switzerland
(c) France and Italy (d) Germany and Austria
75. Which of the following places are known for paper manufacturing industry ?
1. Yamunanagar 2. Guwahati 3. Shahabad 4. Ballarpur
Chose the correct answer using the codes given below:
(a) 1, 2 and 3 (b) 1, 2 and 4 (c) 1, 3 and 4 (d) 2, 3, and 4
76. Consider the following graphs


The behaviour of a fictitious stock market index [comprising a weighted average of the market
prices of a selected list of companies including some multinational corporations (MNC's) over a 15
day period is shown in the graph. The behaviour of the MNC's in the same period is also shown in
the second graph. Which one of the following is a valid conclusion ?
(a) MC's fell steeper in the period depicted
(b) Share prices of every non-MNC company soared by over 5% on the 12th day
(c) Government announced a policy disfavouring MNC's on the 11th day
(d) Whatever be the reason favouring market revival on the 12th day, it appears to be relevant only
to non-MNC companies

77. Which one of the following pairs of composers in different languages and their works on the
Mahabharata theme is correctly matched ?
(a) Sarladasa Bengali
(b) Kasirama Oriya
(c) Tikkana Marathi
(d) Pampa Kannada
78. Match List I with List II and select the correct answer:
List I List II
(Commodities exported from India) (Countries of destination)
A. Iron Ore 1. Russia
B. Leather goods 2. U.S.A.
C. Tea 3. Japan
D. Cotton fabrics 4. U.K.
5. Canada
A B C D A B C D
(a) 5 1 2 3 (b) 3 1 4 2
(c) 1 5 4 3 (d) 3 4 1 2


79. The concept of Public Interest Litigation originated in
(a) the United Kingdom (b) Australia (c) the United States (d) Canada

80. The length, breadth and height of a room are `l, `b' and `h' respectively. The perimeter of the
ceiling expressed as a percentage of the total area of the four walls is
(a) 100 h (b) 100/h (c) h (d) h/100
81. What is the average fat content of buffalo milk ?
(a) 7.2% (b) 4.5% (c) 9.0% (d) 10.0%

82. If the Prime Minister of India belonged to the Upper House of Parliament
(a) he will not be able to vote in his favour in the event of a no-confidence motiom
(b) he will not be able to speakon the budget in the Lower House
(c) he can make statements only in the Upper House
(d) he has to become a member of the Lower Hose within six months after being sworn in as the
Prime Minister

83. M. C. Setalvad, B. N. Rao and Alladi Krishnaswamy Iyer were distinguished members of the
(a) Swaraj Party
(b) All India National Liberal Federation
(c) Madras Labour Union
(d) Servants of India Society
84. The tribal population in Andaman and Nicobar Islands belongs to the
(a) Australoid race (b) Caucasoid race
(c) Mongoloid race (d) Negroid race
85. Given below are two statements, one labelled as Assertion (A) and the other labelled as Reason
(R)
Assertion (A) : The reservation of thirty-three per cent of seats for women in Parliament and State
Legislatures does not require Constitutional amendment.
Reason (R) : Political parties contesting elections can allocate thirty-three per cent of seats they
contest to women candidates without any Constitutional amendment.
In the context of the above two statements which one of the following is correct ?
(a) Both A and R are true and R is the correct explanation of A
(b) Both A and R are true but R is not the correct explanation of A
(c) A is true but R is false
(d) A is false but R is true
86. Which one of the following dances involves solo performance ?
(a) Bharatanatyam (b) Kuchipudi
(c) Mohiniattam (d) Odissi
87. What is the correct chronological sequence of the following ?
1. Wood's Education Despatch.
2. Maeauley's minute on education.
3. The Sargent Education Report.
many votes as there are multiples of one thousand in the quotient obtained by dividing the
population of the State by the total number of the elected members of the Assembly. As at present
(1997) the expression "population" here means the population
as ascertained by the,
(a) 1991 Census (b) 1981 Census (c) 1971 Census (d) 1961 Census


88. The group of small pieces of rock revolving round the sun between the orbits of Mars and
Jupiter are called
(a) meteors (b) comets (c) meteorites (d) asteroids
89. Which one of the following is used as an anti-freeze for the automobile engines ?
(a) Propyl alcohol (b) Ethanol (c) Methanol (d) Ethylene glycol

90. Seven persons P, Q, R, S, T, U and V participate in and finish all the events of a series of
swimming races. There are no ties at the finish of any of the events. V always finishes somewhere
ahead of P. P always finishes somewhere ahead of Q. Either R finishes first and T finishes last or S
finishes first and U or Q finishes last. If in a particular race V finished fifth, then which one of the
following would be true ?
(a) S finishes firsts
(b) R finishes second
(c) T finishes third
(d) R finishes fourth

91. Which one of the following scripts of ancient India was written from right to left ?
(a) Brahmi (b) Nandnagari
(c) Sharada (d) Kharoshti
92. Which one of the following techniques can be used to establish the paternity of a child ?
(a) Protein analysis
(b) Chromosome counting
(c) Quantitative analysis of DNA
(d) DNA finger printing
93. In the earth's direction of rotation is reversed , what would be the IST when it is noon at the
International Date Line?
(a) 06.30 hrs (b) 05.30 hrs (c) 18.30 hrs (d) 17.30 hrs
94. Bishop Carlos Felipe Ximenes Belo and Jos Remos Horta who shared the 1996 Nobel Peace
Prize are known for their work for the cause of
(a) East Timor (b) Gautemala (c) Bosnia (d) Barunda
95. In the Presidential election in India, every elected member of the Legislative Assembly of a
State shall have as many votes as there are multiples of one thousand in the quotient obtained by
dividing the population of the State by the total number of the elected members of the Assembly.
As at present (1997) the expression "population" here means the population as ascertained by the,
(a) 1991 Census (b) 1981 Census (c) 1971 Census (d) 1961 Census
96. In India, inflation is measured by the
(a) Wholesale Price Index number
(b) Consumers Price Index for urban non-manual workers
(c) Consumers Price Index for agricultural workers
(d) National Income Deflation
97. Which one of the following stars is nearest to the Earth?
(a) Polaris (b) Alpha Centauri (c) Sun (d) Sirius
98. Consider the following
1. Saxena, David, Jain and Kumar were District Collectors at places P, Q, R and S respectively in


1970
2. In 1972 they were transferred. Saxena and Jain interchanged places. Kumar and David also
interchanged places.
3. One year later in 1973 they were again transferred such that David and Jain interchanged places
and Saxena and Kumar were also interchanged.
What should be the next round of transfers so that all the four persons could have been posted at all
the .four places ?
(a) Interchange Saxena and David as well as Jain and Kumar
(b) Interchange Saxena and Kumar as well as David and Jain
(c) Interchange David and Kumar as well as Saxena and Jain
(d) It is not possible for all the four persons to have been posted at all the four places

99. The medieval Indian writer who refers to the discovery of America is
(a) Malik Muhammad Jayasi (b) Amir Khusrau
(c) Raskhan (d) Abul Fazl
100. Which one of the following conditions is most relevant for the presence of life on Mars ?
(a) Atmospheric composition
(b) Thermal conditions
(c) Occurrence of ice caps and frozen water
(d) Occurrence of ozone
101. Which of the following are/is stated in the Constitution of India ?
1. The President shall not be a member of either House of Parliament.
2. The Parliament shall consist of the President and two Houses.
Choose the correct answer from the codes given below :
(a) Neither 1 nor 2 (b) Both 1 and 2
(c) 1 alone (d) 2 alone
102. The following are the major oilseeds produced in India
1. Sesamum 2. Mustard. 3. Groundnut 4. Soyabeen
Which one of the following is the correct sequence of the descending order of the quantity of their
production ?
(a) 1, 2, 3, 4 (b) 3, 2, 4, 1 (c) 2, 4, 3, 1 (d) 3, 4, 2, 1
103. A girl is swimming on a swing in sitting position. When the same girl stands up, the period of
swing will
(a) be shorter
(b) be longer
(c) depend on the height of the girl
(d) not change
104. The famous dialogue between Nachiketa and Yama is mentioned in the
(a) Chhandogyopanishad (b) Mundakopanishad
(c) Kathopanishad (d) Kenopanishad


105. Consider the geographical details given in the following figure


The point marked by A in the above figure indicates a country in
(a) North America (b) South America
(c) Europe (d) Asia
106. Match List I with List II and select the correct answer:
List I List II
(Functionaries) (Oaths or affirmations)
A. President of India 1. Secrecy of Information
B. Judges of 2. Faithful Discharge of Supreme Court Duties
C. Members of 3. Faith and Allegiance to Parliament the Constitution of India
D. Minister for the Union 4. Upholding the Constitutetion and the law
A B C D A B C D
(a) 3 4 1 2 (b) 4 3 2 1
(c) 3 4 2 1 (d) 4 3 1 2
107. In the map shown in the given figure, rivers labelled as 1, 2, 3 and 4 respectively


(a) Kosi, Gomati, Ghaghara and Gandak
(b) Kosi, Ganga, Gomati and Ghaghara
(c) Gandak, Ganga, Gomati and Ghaghara
(d) Teesta, Gomati, Ghaghara and Kosi

108. The major component of honey is
(a) glucose (b) sucrose (c) maltose (d) frustose
109. Arteries supplying blood to the heart are called
(a) carotid arteries (b) hepatic arteries
(c) coronary arteries (d) Pulmonary arteries
110. Milindapanho is in the form of a dialogue between the king Menander and the Buddhist monk
(a) Nagasena (b) Nagarjuna
(c) Nagabhatta (d) Kumarilabhatta



l l l. In which one of the following countries are 75 percent of seats in both Houses of Parliament
filed on the basis of first past-the post system and 25 per cent on the basis of Proportional
Representation system of elections ?
(a) Germany (b) Italy (c) France (d) Russia
112. Match List I with List II and select the correct answer:
List I List II
A. Moplah revolt 1. Kerala
B. Pabna revolt 2. Bihar
C. Eka Movement 3. Bengal
D. Birsa Munda revolt 4. Awadh
A B C D A B C D
(a)1 3 4 2 (b) 2 3 4 1
(c)1 2 3 4 (d) 3 4 1 2
113. The Sixth and the Eighth Five Year Plans covered the period 1980-1985 and 1992-1997
respectively. The Seventh Five Year Plan covered the period
(a) 1987-1992 (b) 1986-1901
(c) 1985-1990 (d) 1988-1994
114. Recommended daily intake of proteins for a moderately active women is
(a) 30 g (b) 37 g (c) 40 g (d) 46 g
115. In a group of travelling in a bus, 6 persons can speak Tamil, 15 can speak Hindi and 6 can
speak Gujrati. In that group none can speak any other language. If 2 persons in the group
can speak two languages and one person can speak all the three languages, then how many persons
are there in the group?
(a) 21 (b) 22 (c) 23 (d) 24

116. Which one of the following edicts mentions the personal name of Asoka ?
(a) Kalsi (b) Rummindei
(c) Special Kslinga Edict (d) Maski
117. Consider the map given below


Of the four shaded areas in the map, that which is characterised by hot dry summers, mild and
moist winters and seasonal reversal of winds is the area labelled
(a) 1 (b) 2 (c) 3 (d) 4


118. In the following quotation,
"WE THE PEOPLE OF INDIA,
having solemnly resolved to
constitute India into a Sovereign Socialist Secular
Democratic Republic and to secure to all its citizens
JUSTICE, social, economic and political ;
LIBERTY of thought, expression, belief faith and worship;
EQUALITY of status and of opportunity ; and to promote among them all;
FRATERNITY assuring the dignity of the individual and the unity and the integrity of
the Nation ;
In our Constituent Assembly this `X'
do hereby adopt, enact and
give to ourselves this Constitution.",
`X' stands for
(a) twenty-sixth day of January, 1950
(b) twenty-sixth day of November, 1949
(c) twenty-sixth day of January, 1949
(d) None of the above

119. Examine the following statements regarding a set of balls
1. All balls are black. 2. All balls are white.
3. Only some balls are black 4. No balls are black.
Assuming that the balls can only be black or white, which two of the statements given above can
both be true, but cannot both be false ? Choose the correct answer from the codes given
below :
(a) 1 and 4 (b) 1 and 3
(c) 2 and 3 (d) 2 and 4

120. Match List I with List II and select the correct answer:
List I List II
A. Rashtriya Mahila Kosh 1. Empowerment of women
B. Mahila Samriddhi 2. Education for women's equality
Yojna
C. Indira Mahila Yojana 3. Promotion of savings among rural women
D. Mahila Samakhya 4. Meeting credit needs of the poor women
Programme
A B C D A B C D
(a) 3 2 1 4 (b) 1 3 4 2
(c) 4 3 2 1 (d) 4 1 2 3

121. The following map shows four of the sixteen mahajanapadas that existed in ancient India





The places marked A, B, C and D respectively are
(a) Matsya,Cedi, Kosala, Anga
(b) Surasena, Avanti, Vatsa, Magadha
(c) Matsya, Avanti, Vatsa, Anga
(d) Surasena, Cedi, Kosala, Magadha
122. A boy standing at the point `O' in the given diagram, throws a ball three times with the same
force, but projecting it along different inclinations from the ground. The results of the throws have
been plotted in the diagram. Which one of the following is a valid conclusion ?



(a) The Larger the initial inclination, the longer the throw
(b) The Larger the height reached, the longer the throw
(c) The Larger the height reached, the shorter the throw
(d) The larger the initial inclination, the greater the height reached

123. In the Mahayana Buddhism, the Bodhisatva Avalokitesvar was also known as
(a) Vajrapani (b) Manjusri (c) Padmapani (d) Maitreya
124. Daily weather map showing isobars is an example of
(a) Choropleth map (b) Isopleth map (c) Chorochromatic map (d) Choroschematic map
125. Given below are two statements, one labelled as Assertion (A) and the other labelled as
Reason (R).
Assertion (A) : Wilful disobedience or non-compliance of Court orders and use of derogatory
language about judicial behaviour amount to Contempt of Court.
Reason (R) : Judicial activism cannot be practised without arming the judiciary with punitive
powers to punish contemptuous behaviour.
In the context of the above two statements which one of the following is correct
(a) Both A and Rare true and R is the correct explanation of A


(b) Both A and R are true but R is not the correct explanation of A
(c) A is true but R is false
(d) A is false but R is true
126. Schemes of (i) Urban Micro-Enterprises, (ii) Urban Wage Employment and (iii) Housing and
Shelter Upgradation are parts of
(a) Integrated Rural Development Programme
(b) Nehru Rojgar Yojana
(c) Jawahar Rojgar Yojana
(d) Prime Minister's Rojgar Yojana
127. A boat which has a speed of 5 km/hr in still water crosses a river of width 1 km along the
shortest possible path in 15 minutes. The velocity of the river water in km/hr is
(a) 1 (b) 3 (c) 4 (d) (41)
1/2

128. A survey was conducted on a sample of 1000 persons with reference to their knowledge of
English, French and German. The results of the survey are presented in the given
Venn diagram. The ratio of the number of the persons who do not know any of the three languages
to those who know all the three languages is


(a) 1/27 (b) 1/25 (c) 1/550 (d) 175/1000
129.The silver coins issued by the Guptas were called
(a) rupaka (b) karshapana (c) dinara (d) pang
130.Which one of the following countries is the largest producer of fuel wood in the world ?
(a) Indonesia (b) Russia (c) India (d) China
131.Given below are two statements, one labelled as Assertion (A) and the other labelled as Reason
(R)
Assertion (A) : The emergence of economic globalism does not imply the decline of socialist
ideology.
Reason (R) : The ideology of socialism believes in Universalism and globalism.
In the context of the above two statements which one of the following is correct ?
(a) Both A and Rare true and R is the correct explanation of A
(b) Both A and R are true but R is not the correct explanation of A
(c) A is true but R is false
(d) A is false but R is true
132.The contribution of India's small scale sector to the gross turnover in the manufacturing sector
since 1992 has been of the order of


(a) 40% (b) 34% (c) 30% (d) 45%

133.The working principle of a washing machine
(a) centrifugation (b) dialysis (c) reverse osmosis (d) diffusion

134.Consider the figures given below

In the given sequence of figures, the figure that would logically appear at Sl. No. 5 is

135. Match List I with List II and select the correct answer:
List I (Author) List II (Text)
A. Varahamihira 1. Prabandha Chintamani
B. Visakhadatta 2. Mrchchha-katikam
C. Sudraka 3. Brhat-Samhita
D. Bilhana 4. Devi-Chandraguptam
5. Vikramankadeva-charita
A B C D A B C D
(a) 3 4 5 2 (b) 3 4 2 5
(c) 5 3 4 1 (d) 1 3 5 2

136. One will NOT have to pass through the Suez Canal while going from Bombay to
(a) Alexandria (b) Suez (c) Port Said (d) Benghazi

137. The Dinesh Goswami Committee recommended
(a) the constitution of state level election commission
(b) List system of election to the Lok Sabha
(c) governmental funding of parliamentary elections
(d) a ban on the candidature of independent candidates in the parliamentary elections

138. Which of the following come under Non-plan expenditure ?
1. Subsidies
2. Interest payments
3. Defence expenditure
4. Maintenance expenditure for the infrastructure created in the previous plans
Choose the correct answer using the codes given below :
(a) 1 and 2 (b) 1 and 3 (c) 2 and 4 (d) 1, 2, 3 and 4

139. The sequencing of the entire genome (the totality of all genes) of an organism was completed
in 1996. The organism was
(a) albino mouse (b) yeast (c) human being (d) plasmodium vivax

140. Which one of the following was a corporation of merchants in ancient India ?


(a) Chaturvedimangalam (b) Parishad
(c) Ashtadikgaja (d) Manigrama
141. Match List I with List II and select the correct answer:
List I List II
A. Malaria 1. Bone marrow
B. Filaria 2. Brain
C. Encephalitis 3. Muscle
D. Leukaemia 4. Lymph node
5.Blood cells
A B C D A B C D
(a) 5 3 2 1 (b) 5 4 2 1
(c) 4 3 5 1 (d) 5 4 1 2
142. National Income is the
(a) Net National Product at market price
(b) Net National Product at factor cost
(c) Net Domestic Product at market price
(d) Net Domestic Product at factor cost

143.Which one of the following is NOT a principle of "Panch-sheel" ?
(a) Non-alignment
(b) Peaceful Co-existence
(c) Mutual respect for each other's territorial integrity and sovereignty
(d) Mutual non-interference in each other's internal affairs

144. During a flight from Delhi to Tokyo the following are the landing airports
1. Hongkong 2. Hanoi 3. Taipei. 4. Bangkok.
The correct sequence of the landing at these airports during an onward journey is
(a) 1, 2, 3, 4 (b) 4, 2, 1, 3 (c) 3, 4, 1, 2 (d) 4, 1, 2, 3
145. Antigen is a substance which
(a) lowers body temperature
(b) destroys harmful bacteria
(c) triggers the immune system
(d) is used as an antidote to poison
146. The number of times in a day the Hour-hand and the Minute-hand of a clock are at right angles
is
(a) 44 (b) 48 (c) 24 (d) 12
147. The Badami rock inscription of Pulakesin I is dated in the Saka year 465. If the same were to
be dated in Vikrama Samvat, the year would be
(a) 601 (b) 300 (c) 330 (d) 407
148. The Minimum Alternative Tax (MAT) was introduced in the Budget of the Government of
India for the year
(a) 1991-92 (b) 1992-93
(c) 1995-96 (d) 1996-97
149. Which one of the following is present in the largest amount in terms of per cent by mass in the
earth's crust ?


(a) Silicon (b) Oxygen
(c) Carbon (d) Calcium
150. Match List I with List II and select the correct answer:
List I List II
(Hazardous industries using (Located at)
child labour leading to the filing
of a public interest petition in the
Supreme Court)
A. Glass Industry 1. Moradabad
B. Brassware Industry 2. Marakpur
C. Slate Industry 3. Ferozabad
D. Handmade carpet Industry 4. Mirzapur
A B C D A B C D
(a) 3 1 2 4 (b) 1 3 4 2
(c) 3 1 4 2 (d) 1 3 2 4




General Studies - 1998 (Prelims)
Time Allowed : Two Hours Maximum Marks : 150

1. The water pollution in river is measured by the dissolved amount of
(a) Chlorine (b) Ozone (c) Nitrogen (d) Oxygen

2. Some time back, the Government of India, decided to delicense `white goods' industry. `White
goods' include
(a) stainless steel and alumunium utensils
(b) milk and milk products
(c) items purchased for conspicuous consumption
(d) soaps, detergents and other mass consumption goods

3. Examine the following three statements
1. Processed meat is a perishable food.
2. All perishable foods are packed in sealed tins.
3. Sealed tins sometimes do not contain processed meat.
Which one of the following inferences can be drawn from the above statements ?
(a) Sealed tins always contain perishable food
(b) Processed meat is sometimes not packed in sealed tins.
(c) Processed meat is always packed in sealed tins. (d) Non-perishable foods are never packed in
sealed tins.

4. What is the correct sequence of the following Indian states in descending order of their length of
surface roads per 100 km
2
of their area?
1. Haryana 2. Maharashtra 3. Punjab 4. Tamil Nadu
(a) 4, 3, 2, 1 (b) 4, 3, 1, 2 (c) 3, 4, 1, 2 (d) 3, 4, 2, 1

5. Match List I with List II and select the correct answer
List I List II
A. Potassium bromide 1. Fertiliser
B. Potassium nitrate 2. Photography
C. Potassium sulphate 3. Bakery
D. Monopotassium tartarate 4. Gunpowder
A &sp; B C D A B C D
(a) 2 4 1 3 (b) 2 4 3 1
(c) 4 2 3 1 (d) 4 2 1 3
6. The four railway junctions shown by numerals 1, 2, 3, 4 on the rough outline map of Gujarat are
respectively
(a) Patanpur, Mahesana, Ahmedabad and Vadodara
(b) Mahesana, Surendranagar, Rajkot and Junagarh
(c) Palanpur, Kanda, Bhuj and Okla
(d) Ahmedabad, Vadodara, Bhavnagar and Broach



7. The damage to the Spektr Module of the Russian Space Station Mir was due to
(a) collision with a Soyuz cargo ship
(b) faulty material design of the Spektr Module
(c) explosion inside the space station
(d) collision with an asteroid

8. The misery index is the sum of a country's unemployment and inflation rate. The higher the
index, the more miserable a is the country to live in. The figure given below is the Misery Index for
various countries in Europe


Which of the following conclusions can be drawn from the misery index given above?
1. Britain is the most miserable country to live in.
2. The inflation rate in Spain is less than than in Belgium and Britain.
3. Italy and France seem to have almost identical unemployment rate.
4. The higher the misery index the higher the inflation rate.
Select the correct answer using the codes given below
(a) 1 alone (b) 2 and 3
(c) 1, 2, 3 and 4 (d) None

9. The meeting of G-15 countries held in Malayasia in 1997 was attended by
(a) 14 member countries
(b) 15 member countries
(c) 16 member countries
(d) 17 member countries

10. Olympics 2004 is to be held in
(a) Johannesberg (b) Capetown (c) Rome (d) Sydney

11. Which one of the following elements is essential for the construction of nuclear reactors?
(a) Cobalt (b) Nickel (c) Zirconium (d) Tungsten

12. The educated middle class in India
(a) opposed the revolt of 1857
(b) supported the revolt of 1857
(c) remained neutral to the revolt of 1857
(d) fought against native rulers



13. Which one of the following regions of the world supplies the maximum of our imported
commodities (in terms of rupee value)?
(a) Africa (b) America (c) Asia and Oceania (d) Europe

14. The currency of the proposed European Monetary Union will be
(a) Dollar (b) Euro (c) Guilder (d) Mark
15. Which one of the following was the venue for the preliminary talks between the Sri Lankan
Government and representatives of Tamil United Liberation Front and other militant groups ?
(a) New Delhi (b) Colombo (c) Thimpu (d) Madras

16. Consider the following statements regarding asteroids
1. Asteroids are rocky debris of varying sizes orbiting the Sun.
2. Most of the asteroids are small but some have diameter as large as 1000 km
3. The orbit of asteroids lies between orbits of Jupiter and Saturn.
Of these statements
(a) 1, 2 and 3 are correct (b) 2 and 3 are correct
(c) 1 and 2 are correct (d) 1 and 3 are correct

17. 'MERCOSUR' consists of group of countries of
(a) Africa (b) Asia (c) Latin America (d) South East Asia
18. Which one of the following countries has replaced Italy as the major importer of bauxite from
India ?
(a) Canada (b) Greece (c) Ukraine (d) United Arab Emirates

19. Lord Mountbatten came to India as Viceroy along with specific instruction to
(a) balkanize the Indian sub-continent
(b) keep India united if possible
(c) accept Jinnah's demand for Pakistan
(d) persuade the Congress to accept partition

20. Which one of the following is the correct sequence of the states (labeled 1, 2, 3 and 4) of India
shown on the map in descending order in terms of their available ground water-resources for
irrigation?
(a) 3, 4, 1, 2
(b) 3, 4, 2, 1
(c) 4, 3, 1, 2
(d) 4, 3, 2, 1

21. Consider the following statements
The price of any currency in international market is decided by the


1. World Bank.
2. demand for goods/services provided by the country concerned.
3. stability of the government of the concerned country.
4. economic potential of the country in question.
Of these statements
(a) 1, 2, 3 and 4 are correct (b) 2 and 3 are correct
(c) 3 and 4 are correct (d) 1 and 4 are correct

22. Consider the following statements
Coke is one of the materials of the charge added to blast furnace for the production of steel/iron. Its
function is to
1. act as a reducing agent.
2. remove silica associated with the iron ore
3. function as fuel, to supply heat.
4. act as an oxidizing agent.
Of these statements
(a) 1 and 2 are correct (b) 2 and 4 are correct
(c) 1 and 3 are correct (d) 3 and 4 are correct

23. A consumer is said to be in equilibrium, if
(a) he is able to fulfill his need with a given level of income
(b) he is able to live in full comforts with a given level of income
(c) he can fulfill his needs without consumption of certain items
(d) he is able to locate new sources of income

24. Production of Rice and Wheat
(In millions of Tonnes)
Year Rice Wheat Percentage of Wheat to Rice
1950-51 20.58 6.46 31.4
1960-61 34.58 11.00 31.8
1970-71 42.22 23.83 56.4
1980-81 53.63 36.31 67.7
1990-91 74.29 55.14 74.2
1994-95 81.81 65.77 80.4
1995-96 79.62 62.62 78.6
The above table indicates the performance in India in rice and wheat production from 1950-51 to
1995-96. Which of the following conclusions arrived at from the above table would be valid?
1. Record production of rice as well as wheat has been in 1994-95
2. The ratio of wheat to rice production seems to have steadily increased over 16 years
3. Wheat has not been popular among the Indian population before 1980.
4. India became self-sufficient in rice and wheat only after 1990.
Select the correct answer using the codes given below
(a) 1 and 2 (b) 1, 2, 3 and 4
(c) 3 and 4 (d) None

25. Among the Indian states shown labeled 1, 2, 3 and 4 in the rough outline map given, the correct
sequence of descending order of percent of scheduled tribe population to their total population is :


(a) 1, 3, 2, 4 (b) 3, 1, 2, 4 (c) 3, 1, 4, 2 (d) 1, 3, 4, 2

26. Which one of the following sets of states stands to benefit the most from the Konkan Railway?
(a) Goa, Karnataka, Maharashtra, Kerala
(b) Madhya Pradesh, Maharashtra, Tamil Nadu, Kerala
(c) Tamil Nadu, Kerala, Goa, Maharashtra
(d) Gujarat, Maharashtra, Goa, Tamil Nadu

27. The supply-side economics lays greater emphasis on the point of view of
(a) producer (b) global economy (c) consumer (d) middle-man

28. Which one of the following languages belongs to the Austric group ?
(a) Maratlti (b) Ladakhi (c) Khasi (d) Tamil

29. Which one of the following metals does not form amalgams?
(a) Zinc (b) Copper (c) Magnesium (d) Iron
30. Indonesian forest fire in 1997 was caused by
(a) greenhouse effect (b) depletion of ozone laver
(c) EL Nino effect (d) None of the above

31. LMNOP, is semicircle with center at R and diameter LP, LSRand RQP are also semicircles
with centers at T and U and diameters LR= RP =1 /2 LP. The ratio of perimeters of LMNOP and
LSRQP us
(a) 0.75: 1
(b) 1:1
(c) 1 : 0.75
(d) 1.25: 1


32. Match the following research institutes A, B, C and D with their respective location labelled as
1 to 6 in the given rough outline map :



Select the correct answer using the codes given below
A. Central Drug Research Institute
B. National Atlas and Thematic Mapping Organisation
C. National Institute of Ocean Technology
D. Temperate Forest Research Centre
A B C D
(a) 4 3 2 6
(b) 4 5 1 6
(c) 2 3 4 5
(d) 1 6 2 3

33. Bharat Ratna was awarded in 1997 to
(a) Dr. Homi Bhabha (Posthumous)
(b) Former President R. Venkataraman
(c) Satyajit Ray (Posthumous)
(d) Dr. Abdul Kalam

34. Which of the following statements are true about the Indian born woman astronaut Kalpana
Chawla ?
1. She was born in Karnal.
2. She flew on board the shuttle flight STS - 87.
3. She was trained at the Kennedy Space Centre.
4. She did a space-walk to retrieve the Spartan Satellite.
Select the correct answer using the codes given below
(a) 1 and 2
(b) 2 and 3
(c) 1, 2 and 3
(d) 1, 2 and 4

35. A man purchases two clocks A and B at a total cost of Rs. 650. He sells A with 20% profit and
B at a loss of 25% and gets the same selling price for both the clocks. What are the purchasing price
of A and B respectively
(a) Rs. 225 ; Rs. 425
(b) Rs. 250 ; Rs. 400
(c) Rs. 275 ; Rs. 375
(d) Rs. 300 ; Rs. 350



36. A fuse is used in main electric supply as a safety device. Which one of the following statements
about the fuse is correct ?
(a) It is connected in parallel with the main switch
(b) It is made mainly from silver alloys
(c) It must have a low melting point
(d) It must have a very high resistance
Directions :- The following fourteen items consist of two statements, one labelled as 'Assertion A
and the other labelled as 'Reason R'. You are to examine these two statements carefully and decide
if the Assertion A and the Reason R are individually true and if so, whether the Reason is a correct
explanation of the Assertion. Select the your answers to these items using the codes given below
and mark your answer sheet accordingly.
Codes:
(a) Both A and Rare true and R is the correct explanation of A
(b) Both A and R are true but R is NOT the correct explanation of A
(c) A is true but R is false
(d) A is false but R is true

37. Assertion A : The Gandhara School of Art bears the mark of Hellenistic influence.
Reason R : Hinayana form was influenced by that art.
38. Assertion A : Formic acid is a stronger acid than acetic acid.
Reason R : Formic acid is an organic acid.
39. Assertion A : At first the Turkish administration in India was essentially military.
Reason R : The country was parcelled outas'Iqtas' among leading military leaders.
40. Assertion A : According to Asoka's edicts social harmony among the people was more
important than religious devotion.
Reason R : He spread ideas of equity instead of promotion of religion.
41. Assertion A : The temperature of a metal wire rises when an electric current is passed
through it.
Reason R : Collision of metal atoms with each other releases heat energy.
42. Assertion A : The Khilafat movement did bring the urban Muslims into the fold of the.
National Movement.
Reason R : There was a predominant elements of anti-imperialism in both the National
and Khilafat Movement.
43. Assertion A : Phenyl is used as a household germicide. Phenyl is a phenol derivative and
phenol is an effective germicide.
Reason R : Phenyl is a phenol derivative and phenol is an effective germicide
44. Assertion A : Partition of Bengal in 1905 brought to an end the Moderates role in the Indian
freedom movement.
Reason R : The Surat session of Indian National Congress separated the Extremists from
the Moderates.
45. Assertion A : The first ever Bill to make primary education compulsory in India was rejected
in 1911
Reason R : Discontent would have increased every cultivator could read


46. Assertion A : Sodium metal is stored under kerosene
Reason R: Metallic sodium melts when exposed to air
47. Assertion A : The cngress rejected the cripps proposals
Reason R : The cripps Mission consisted solely of whites
48. Assertion A : The United States of America has threatened to ask the World Trade
Organisation (WTO) to apply sanctions against the developing countries for the non-obsergance of
I.L.O. conventions
Reason R : The United States of America itself has adopted and implemented those I.L.O.
conventions.

49. Assertion A :During the reign of Shahjahan, Dara Sikoh was sent on expedition to Balkha,
Badakhshan and Qandahar.
Reason R :The expedition sent by Shahjahan to the Middle-East was a marvellous success
50. Assertion A : Gandhi stopped the Non-Co-operation Movement in 1922.
Reason R : Violence at Chauri Chaura led him to stop the movement.

51. The correct sequence of different layers of the atmosphere from the surface of the Earth
upwards is
(a) Troposphere, Stratosphere, Lonosphere, Mesosphere
(b) Stratosphere, Troposphere, Lonosphere, Mesosphere
(c) Troposphere, Stratosphere, Mesosphere, Lonosphere
(d) Stratosphere, Troposphere, Mesosphere, Lonosphere

52. If 15 pumps of equal capacity can fill a tank in 7 days, then how many extra pumps will be
required to fill the tank in 5 days'?
(a) 6
(b) 7
(c) 14
(d) 21

53. Which of the following pairs are correctly matched'?
1. Lothal Ancient dockyard
2. Sarnath First Sermon of Buddha
3. Rajgir Lion capital of Ashoka
4. Nalanda Great seat of Buddhist learning
Select the correct answer using the codes given below
(a) 1, 2, 3 and 4
(b) 3 and 4
(c) 1, 2 and 4
(d) 1 and 2

54. Which one of the following pairs of cities has recently been connected through a 6-lane express
way ?
(a) Ahmedabad : Vadodra
(b) Dhaka : Chittagong
(c) Islamabad : Lahore
(d) Mumbai : Pune


55. Match List I with List II and select the correct answer
List-I List II
A. Ringgit 1. Indonesia
B. Baht 2.Siuth Korea
C. Rupiah 3.Thailand
D. Won 4.Malaysia
A B C D
(a) 1 3 4 2
(b) 4 3 1 2
(c) 1 2 4 3
(d) 4 2 1 3
56. Consumption offish is considered to be healthy when compared to flesh of other animals
because fish contains
(a) polyunsaturated fatty acids
(b) saturated fatty acids
(c) essential vitamins
(d) more carbohydrates and proteins

57. Consider the following statements Ahadis were those troopers who
1. offered their services singly.
2. did not attach themselves to any chief.
3. has the emperor as their immediate colonel.
4. attached themselves to Mirzas.
Of these statements
(a) 1, 3 and 4 are correct
(b) 1, 2 and 3 are correct
(c) 2 and 3 are correct
(d) 1 and 4 are correct

58. Out of three annual examinations, each with a total of 500 marks, a student secured average
marks of 45% and 55% in the first and second annual examinations. To have an overall average of
60%, how many marks does the student need to secure in the third annual examination ?
(a) 450
(b) 400
(c) 350
(d) 300

59. When the Indian Muslim League was inducted into the interim government in 1946, Liyaqat Ali
Khan was assigned the portfolio of
(a) Foreign affairs
(b) Home
(c) Finance
(d) Defence

60. Which one of the following ancient Indian records is the earliest royal order to preserve food-
grains to be utilised during the crises in the country ?
(a) Sohagaura Copper-plate
(b) Rummindei pillar-edict of Asoka
(c) Prayaga-Prasasti


(d) Mehrauli Pillar inscription of Chandra

61. According to Meadows(1972), if the present trends in world population industrialization,
pollution, food production and resource depletion continue unchanged, the "Limits to Growth" on
our planet will be reached in the next
(a) 50 years (b) 100 years (c) 150 years (d) 200 years

62. Human Poverty Index was introduced in the Human Development Report of the year
(a) 1994 (b) 1995 (c) 1996 (d) 1997

63. Match List-I with List-II and select the correct answer
List-I (Disease) List II (Organism)
A. Malaria 1. Fungi
B. Poliomyelitis 2. Bacteria
C. Tuberculosis 3. Virus
D. Ringworm 4. Protozoan
A B C D A B C D
(a) 4 3 2 1 (b) 4 3 1 2
(b) 3 4 1 2 (c) 1 2 3 4

64. Consider the following
1. Tughlaquabad fort. 2. Lodi Garden
3. Qutab Minar. 4.Fatehpur Sikri.
The correct chronological order in which they were built is
(a) 3, 1, 4, 2 (b) 3, 1, 2, 4 (c) 1, 3, 2, 4 (d) 1, 3, 4, 2

65. Which one of the following satisfied the relationship Dda : aDD :: Rrb: ?
(a) D D A (b) R R R (c) b R R (d) B B r

66. The concept of Eight-fold path forms the theme of
(a) Dipavamsa (b) Divyavadana
(c) Mahaparirubban Sutta (d) Dharma Chakara Pravartana Sutra .

67. The recent Land Mines Conference to sign the historic treaty was held in the Capital city of
(a) Canada (b) Japan (c) Sweden (d) Zimbabwe

68. Which of the following pairs are correctly matched
1. Dow Jones New York
2. Hang Seng Seoul
3. FTSE- 100 London
Select the correct answer using the codes given below
(a) 1, 2 and 3 (b) 2 and 3 (c) 1 and 2 (d) 1 and 3

69. The Indian National Congress agreed in 1947 to the partition of the country mainly because
(a) the principle of two-Nation theory was then acceptable to them
(b) it was imposed by the British Government and the Congress was helpless in this regard
(c) they wanted to avoid large-scale communal riots
(d) India would have otherwise lost the opportunity to attain freedom



70. Haemophilia is a genetic disorder which leads to
(a) decrease in haemoglobin level (b) rheumatic heart disease
(c) decrease in WBC (d) non-clotting of blood

71. Match List-I with List-II and select the correct answer
List-I List-II
A. 1556 1. Battle of Haldi Ghati
B. 1600 2. Nadir Shah's capture of Delhi
C. 1686 3. Death of Shivaji
D. 1739 4. Grant of Charter to East India Company
5. Accession of Akbar
A B C D A B C D
(a) 3 4 2 1 (b) 5 4 3 2
(c) 5 2 1 4 (d) 1 5 3 2
72. A square pond has 2 in sides and is 1 m deep. If it is to be enlarged, the depth remaining the
same, into a circular pond with the diagonal of the square its diameter its shown in the fixture, then
what would be the volume of earth to be removed ?


(a) (2 - 4) m
3
(b) (4 - 4) m
3

(c) (4 - 2) m
3
(d) (2 - 2) m
3

73. Match river labelled A, B, C and D on the given map with their names given in the list and
select the correct answer
List-I
1. St. Lawrence
2. Orinoco
3. Mackenzie
4. Amazon
5. Yukon

A B C D
(a) 4 3 2 1
(b) 5 3 1 2
(c) 5 4 1 3
(d) 3 1 4 2


74. Consider the following statements about acetylene
1. It is used in welding industry.
2. It is a raw material for preparing plastics.
3. It is easily obtained by mixing silicon carbide and water.
Of these statements


(a) 1 and 2 are correct (b) 1 and 3 are correct
(c) 2 and 3 are correct (d) 1, 2 and 3 are correct

75. At the time of India's Independence, Mahatma Gandhi was
(a) a member of Congress Working Committee (b) not a member of the Congress
(c) the President of the Congress (d) the General Secretary of the Congress

76. A ball is dropped from the top of a high building with a constant acceleration of9.8 m/s7. What
will be its velocity after 3 seconds'?
(a) 9.8 m/s (b) 19.6 m/s (c) 29.4 m/s (d) 39.2 m/s

77. The economist who was associated with the WTO draft document is
(a) A. K. Sen (b) T. N. Srinivasan
(c) J. N. Bhagwati (d) Avinash Dixit

78. One local and another express train were proceeding in the same direction on parallel tracks at
29km/hourand65 km/hour respectively. The driver of the former notices that it look exactly 16
seconds for the faster train to pass by him. What is the length of the faster train?
(a) 60 m (b) 120 m (c) 160 m (d) 240 m

79. In the given map, the shaded part represents Akbar's empire at a certain empire : 'A' stands for
an independent country and B' marks the site of a city. Which one of the following alternatives
given all correct information

(a) Akbar in 1557 : (A) Golkunda, (B) Lahore
(b) Akbar in 1557 : (A) Khandesh (B) Multan
(c) Akbar in 1605 : (A) Gondwana, (B) Multan
(d) Akbar in 1605 : (A) Gondwana, (B) Lahore
80. Estuaries possess distinct blooms of excessive growth of a pigmented dinoflagellates. These
blooms are called
(a) red tides (b) sea tides (c) black tides (d) sea flowers
81. Consider the following climatic conditions (northern hemisphere) :
Temperature Rainfall
C Cm.
J 3.9 4.7


F 4.4 5.7
M 8.3 8.2
A 14.4 9.2
M 20.0 9.2
J 23.3 17.7
J 27.8 14.5
b 27.8 14.0
S 22.3 12.7
O 18.3 7.0
N 12.2 5.0
D 6.7 3.5
These are most likely to be found in the natural regions of
(a) China type (b) Equatorial type
(c) Hot desert type (d) Monsoon type

82. A, B, C, D, E and F, not necessarily in that order are sitting on six chairs regularly placed
around a round table. It is observed that
A is between D and F, C is opposite D, and D and E are not on neighbouring chairs.
Which one of the following pairs must be sitting on neighbouring chairs ?
(a) A and B (b) C and E (c) B and F (d) A and C

83. The number of economically active women (excluding students and those doing domestic
duties in their homes) as a percentage of all women of working age (generally those aged 15-64
years) was the highest in 1996 in
(a) USA (b) China (c) Russia (d) S. Korea

84. If in a certain code SAND is VDOG and BIRD is ELUG, then what is the code for LOVE
(a) PRYG (b) ORTG (c) NPUH (d) ORYH

85. When there is noon at I.S.T. meridian people on another place of the Earth are taking their at 6
O'clock morning tea. The longitude of the place is
(a) 17 30' E (b) 7 30' W (c) 172 30' E (d) 90 W

86. Match List-I with List-II and select the correct answer
List I List II
(Special characteristic) (Name of planet)
A. Smallest planet of the 1. Mercury solar system
B. Largest planet of the 2. Venus solar system
C. Planet second from the 3. Jupiter Sun in the solar system
D. Planet nearest to the 4. Pluto Sun
5. Saturn
A B C D A B C D
(a) 2 3 5 1 (b) 3 5 1 2
(c) 4 1 2 3 (d) 4 3 2 1
87. The given map relates to :




(a) Kaniska at the time of his death
(b) Samudragupta after the close of his South Indian campaign
(c) Asoka towards close of his reign
(d) Empire of Thaneswar on the eve of Harsha's accession

88. In a family, a couple has a son and a daughter. The age of the father is three times that of his
daughter and the age of the son is half of his mother. The wife is nine years younger to her husband
and the brother is seven years older than his sister. What is the age of the mother'?
(a) 40 years (b) 45 years (c) 50 years (d) 60 years

89. The missing fraction in the series given below is
4/ 9, 9 / 20, .... 39 / 86
(a) 17 / 40 (b) 19 / 42 (c) 20 / 45 (d) 29 / 53

90. Which one of the following schedules of the Constitution of India contains provisions regarding
anti-defection Act '1
(a) Second Schedule (b) Fifth Schedule (c) Eighth Schedule (d) Tenth Schedule

91. Which one of the following types of micro organisms is most widely used in industries ?
(a) Bacteria (b) Bacteria and Fungi
(c) Bacteria and algae (d) Bacteria, microalgae and fungi

92. Many of the Greeks, Kushanas and Shakas embraced Buddhism rather than Hinduism because
(a) Buddhism was in the ascendant at that time
(b) they had renounced the policy of war and violence
(c) caste-ridden Hinduism did not attract them
(d) Buddhism provided easier access to Indian society

93. There are 50 students admitted to a nursery class. Some students can speak only English and
some can speak only Hindi. 10 students can speak both English and Hindi. If the number of
students who can speak English is 21, then how many students can speak only Hindi, how many
can speak only Hindi and how many can speak only English?
(a) 21, 11 and 29 respectively (b) 28, 18 and 22 respectively
(c) 37, 27 and 13 respectively (d) 39, 29 and 11 respectively

94. Match the names of outstanding Indian scientists given in List-I with area of their specialized


work given in List-II and select the correct answer using the codes given below the lists
List-I List II
A. Dr. Raja Ramanna 1.Plant Chemistry
B. Dr. M. S. Swaminathan 2.Nuclear Physics
C. Prof U.R. Rao 3.Thermodynamics
D. Prof Meghnad Saha 4.Space Research
5.Agricultural sciences
A B C D A B C D
(a) 3 5 2 1 (b) 2 1 4 3
(c) 2 5 4 3 (d) 3 1 4 2

95. The satellites of which one of the following countries have helped in the preparation of a detai-
-led and complete map of Antarctica?
(a) Canada (b) France (c) Russia (d) U.S.A.

96. The Indian parliamentary system is different from the British parliamentary system in that India
has
(a) both a real and a nominal executive (b) a system of collective responsibility
(c) bicameral legislature (d) the system of judicial review

97. Match List-I with List-II and select the correct answer
List-I List II
A. Fruit 1.Ovule
B. Seed 2.Leaf
C. Wood 3.Stem
D. Starch 4.Ovary
A B C D A B C D
(a) 2 1 3 4 (b) 4 1 3 2
(c) 2 3 1 4 (d) 4 3 1 2
98. The member of Shivaji's Astha Pradhana who-looked after foreign affairs was
(a) Peshwa (b) Sachiv (c) Pandit Rao (d) Sumant

99. PanchayatRajwasfirstintroducedinIndiainOctober,1959 in
(a) Rajasthan (b) Tamil Nadu
(c) Kerala (d) Karnataka

100. Some people in Manipur live in houses built on floating is lands of weeds and decaying''.
vegetation held together by suspended silt. These islands are called
(a) Tipis (b) Burkhans
(c) Phoomdis (d) lzba

101. The Ashokan major rock edicts which tell us about the Sangam Kingdom include rock edicts
(a) I and X (b) I and XI
(c) II and XIII (d) II and XI V

102. Which one of the following east flowing rivers of India rift valley due to down warping ?
(a) Damodar (b) Mahanadi
(c) Son (d) Yamuna



103. According to the World Development Report, low income economies are those for which the
per capita GNP in 1994 was
(a) US $ 925 or less (b) US $ 825 or less
(c) US $ 725 or less (d) US $ 525 or less

104. Match List-I with List-II and select the correct answer
List-I List II
A. Theory of Mutation 1. Beadk and Tatum
B. Theory of Evolution 2. Jacob and Monod
C. One gene one enzyme 3. Darwin
hypothesis
D. Operon concept 4. DeVries
A B C D A B C D
(a) 3 4 1 2 (b) 4 3 1 2
(c) 4 3 2 1 (d) 3 4 2 1
105. What is the correct sequence of the following events ?
1. Tilak's Home Rule League.
2. Kamagatamaru Incident.
3. Mahatma Gandhi's arrival in India.
Select the correct answer using the codes given below the lists:
(a) 1, 2, 3 (c) 2, 1, 3 (b) 3, 2, 1 (d) 2, 3, 1
106. An accurate clock shows the time as 3.00. After hour hand has move 135, the time would be
(a) 7.30 (b) 6.30 (c) 8.00 (d) 9.30

107. The loss of Qandhar was a big blow to the Mughal empire from the view point of
(a) natural resources
(b) buffer territory
(c) communication
(d) strategic stronghold

108. A tree species in Mauritius failed to reproduce because of the extinction of a fruit-eating bird.
Which one of the following was that bird ?
(a) Dove (b) Dodo
(c) Condor (d) Skua

109. Which of the following pairs are correctly matched?
1. Mrichchakatikam Shudraka
2. Buddhacharita Vasuvandhu
3. Mudrarakshasha Vishakhadatta
4. Hurshacharita Banabhatta
Select the correct answer using the codes given below
(a) 1, 2, 3 and 4 (b) 1, 3 and 4
(c) 1 and 4 (d) 2 and 3

110. Forest areas have been labelled as 1, 2, 3 and 4 in the rough outline map given :





Among these, those which were threatened in 1997 by a serious epidemic include
(a) teak forests of 3 and 4
(b) oaks forests of 1 and sal forests of 2
(c) sal forests of 3
(d) sandalwood forests of 4

111. Simon Commission of 1927 was boycotted because
(a) there was no Indian Member in the Commission
(b) it supported the Muslim League
(c) Congress felt that the people of India are entitled to Swaraj
(d) there were differences among the members

112. The current price index (base 1960) is nearly 330. This means that
(a) all items cost 3-3 times more than what they did in 1960
(b) the prices of certain selecteditemshavegoneupto3-3times
(c) weighted mean of prices of certain items has increased 3-3 times
(d) gold price has gone up 3-3 times

113.



In the above set of figures (I) to (IV), some parts are shown to change their position in a regular
direction. Following the same sequence, which one of the following will appear at the fifth stage?






114. The Indian Muslims in general, were not attracted to the Extremist movement because of the
(a) influence of the Sir Sayid Ahmed khan
(b) anti-Muslim attitude of Extremist leaders
(c) indifference shown to Muslim aspirations
(d) Extremists' policy of harping on Hindu past

115. Which one of the following statements regarding starch and cellulose is NOT correct ?
(a) Both of them are of plant origin
(b) Both of them are polymers
(c) Both of them give colour with iodine
(d) Both of them are made up of glucose molecules

116. Commercial production of mineral oil has started recently in which one of the areas of India,
labelled 1, 2, 3 and 4 in the rough map given below
(a) 1 (b) 2 (c) 3 (d) 4

117. Examine the following statements
1. All members of Mohan's family are
2. Some members of Mohan's family are not employed.
3. Some employed persons are not honest.
4. Some honest persons are not employed.
Which one of the following inferences can be drawn from the above statements ?
(a) All members of Mohan's family are employed
(b) The employed members of Mohan's family are honest
(c) The honest members of Mohan's family are not employed
(d) The employed members of Mohan's family are not honest

118. The banks are required to maintain a certain ratio between their cash in hand and total assets.
This is called


(a) SBR (Statutory Bank Ratio)
(b) SLR (Statutory Liquid Ratio)
(c) CBR (Central Bank Reserve)
(d) CLR (Central Liquid Reserve)

119. Ergotism is due to consumption of
(a) contaminated grains
(b) rotting vegetables
(c) contaminated water
(d) stale cooked food

120. What is the correct chronological order in which the following appeared in India ?
1. Gold coins.
3. Iron plough.
2. Punch-marked silver coins.
4. Urban culture.
Select the correct answer using the codes given below
(a) 3, 4, 1, 2 (b) 3, 4, 2, 1
(c) 4, 3, 1, 2 (d) 4, 3, 2, 1

121. Match List-I with List-II and select the correct answer
List-I List II
(Minerals) (Mining area)
A. Graphite 1. Bellary
B. Lead 2. Didwana
C. Salt 3. Rampa
D. Silver 4. Zawar
A B C D A B C D
(a) 3 4 1 2 (b) 1 4 2 3
(c) 3 1 4 2 (d) 2 3 1 4
122. In a dinner party both fish and meat were served. Some took only fish and some only meat.
There were some vegetarians who did not accept either. The rest accepted both fish and meat.
Which one of the following logic diagrams correctly reflects the above situation ?



123. Which one of the following events, was characterized by Montague as 'Preventive Murder? (a)
Killing of INA activists (b) Massacre of Jallianwalla Bagh
(c) Shooting of the Mahatma (d) Shooting of Curzon-Wythe


124. Fawazil in the Sultanate period means
(a) extra payment made to the nobles
(b) revenue assigned in lieu of salary
(c) excess amount paid to the exchequer by the iqtadars
(d) illegal exactions extracted from the peasants

125. The discovery of Oak flora in 1966 added a new chapter to the history of Indian Sericulture.
Which one of the following states is the leading producer of Oak tasar silk ?
(a) Assam (b) Bihar (c) Manipur (d) Orissa

126. What is the correct sequence of the following events ?
1. The August offer.
2. The I.N.A. trial.
3. The Quit India Movement.
4. The Royal Indian Naval Ratings Revolt.
Select the correct answer using the codes given below
(a) 1, 3, 2, 4 (b) 3, 1, 2, 4 (c) 1, 3, 4, 2 (d) 3, 1, 4, 2

127. Nobel Prize in Economics for the year 1997 was awarded for contribution in the area of
(a) International Economics (b) Financial Economics
(c) Public Economics (d) Development Economics
128. Match List-I with List-II and select the correct answer
List-I List- II
(Places) (Industries)
A. Jamnagar 1.Aluminium
B. Hospet 2.Wollen Textile
C. Korba 3. Fertilizers
D. Haldia 4. Cement
5. Iron and Steel
A B C D A B C D
(a)4 3 1 2 (b) 2 5 1 3
(c)4 5 2 1 (d) 2 1 4 3
129. The complete conversion of glucose in the presence of oxygen into carbon dioxide and water
with release of energy is called
(a) aerobic respiration (b) anaerobic respiration
(c) glycolysis (d) hydrolysis

130. Which one of the following defines extremist ideology during the early phase of Indian
freedom movement'?
(a) Stimulating the production of indigenous articles by giving them preference over imported
commodities
(b) Obtaining self-government by aggressive means in place of petitions and constitutional ways
(c) Providing national education according to the requirements of the country
(d) Organizing coups against the British empire through military revolt

131. In the vicinity of Mumbai, a number of specialized towns have been developed. Match the lists
of specialization with towns and select the correct answer using the codes given below the
lists:
List-I List- II


(Towns) (Specialization)
A. Alibag 1. Fishing Centre
B. Balapur 2. Holiday Resort
C. Nhava Sheva 3. Petro-Chemical complex
D. Ratnagiri 4. Port
A B C D A B C D
(a) 1 3 2 4 (b) 2 3 4 1
(c) 3 4 2 1 (d) 2 1 4 3

132. The accounting year of the Reserve Bank of India is
(a) April-March (b) July-June
(c) October-September (d) January-December

133. The Sultan of Delhi who is reputed to have built the biggest network of canals in India was (a)
Iltutmish (b) Ghiyasuddin Tughlaq
(c) Feroze Shah Tughlaq (d) Sikandar Lodi
134. The major chemical compound found in human kidney stone is
(a) Uric acid (b) Calcium carbonate
(c) Calcium oxalate (d) Calcium sulphate

135. Which of the following pairs are correctly matched?
1. Theodore Beck : Mohammadan Anglo-Oriental College, Aligarh
2. Ilbert Bill : Ripon
3. Pherozeshah Mehta : Indian National Congress.
4. Badruddin Tyabji : Muslim League
Select the correct answer using the codes given below
(a) 1, 2, 3 and 4 (b) 2 and 4 (c) 1, 3 and 4 (d) 1, 2 and 3

136. Match List-I with List-II and select the correct answer
List-I List -II
(Agricultural products) (Foremost Producer)
A. Cotton 1. Madhya Pradesh
B. Gram 2. Gujarat
C. Black pepper 3. West Bengal
D. Pineapple 4. Kearla
A B C D A B C D
(a) 2 1 4 3 (b) 2 1 3 4
(c) 1 2 4 3 (d) 1 2 3 4
137. Which one of the following is the correct sequence of decreasing order of the given currencies
in terms of their value in Indian Rupees ?
(a) US dollar, Canadian dollar, New Zealand dollar, Hong Kong dollar
(b) US dollar, New Zealand dollar, Canadian dollar, Hong Kong dollar
(c) US dollar, Hong Kong dollar, New Zealand dollar, Canadian dollar
(d) US dollar, Hong Kong dollar, Canadian dollar, New Zealand dollar

138. 'Eco mark' is given to the Indian products that are
(a) pure and unadulterated
(b) rich in proteins
(c) environment friendly


(d) economically viable

139.A, B, C, D, E, F and G are members of a family consisting of 4 adults and 3 children, two of
whom, F and G are girls. A and D are brothers and A is a doctor. E is an engineer married to one of
the brothers and has two children. B is married to D and G is their child. Who is C?
(a) G's brother (b) F's father (c) E's daughter (d) A's son

140. Capital Account Convertibility of the Indian Rupee implies
(a) that the Indian Rupee can be exchanged by the authorized dealers for travel
(b) that the Indian Rupee can be exchanged for any major currency for the purpose of trade in
goods and services
(c) that the Indian Rupee can be exchanged for any major currency for the purpose of trading
financial assets
(d) none of the above

141. What are the official languages of the U. N. O ?
(a) English, French and Russian
(b) English, French, German and Russian
(c) English, French, Russian, Chinese and Hindi
(d) English, French, Chinese, Russian, Arabic and Spanish

142. The rough outline map given shows centres if cement industry labelled 1, 2, 3 and 4. Match
these centres with the following

A B C D A B C D
(a) 3 4 2 1 (b) 2 4 1 3
(c) 1 2 4 3 (d) 2 3 1 4

143. The Congress is tottering to its fall and one of my great ambitions while in India, is to assist it
to a peaceful demise." This statement is attributed to
(a) Lord Duferin (b) Lord Curzon (c) Lord Lytton (d) None of the above

144. Economic Survey in India is published officially, every year by the
(a) Reserve Bank of India
(b) Planning Commission of India
(c) Ministry of Finance, Govt. of India
(d) Ministry of Industries, Govt. of India

145. One Astronomical Unit is the average distance between
(a) Earth and the Sun
(b) Earth and the Moon


(c) Jupiter and the Sun
(d) Pluto and the Sun
146. Which one of the following ports shown on the rough outline map of India is a riverine port'?

(a) 1 (b) 2 (c) 3 (d) 4
147. Who was the leader of the Ghaddar Party'?
(a) Bhagat Singh
(b) Lala Hardayal
(c) Bal Gangadhar Tilak
(d) V.D. Savarkar

148. Which of the following Parties were not a part of the United Front which was in power during
'96-97' '?
1. Bahujana Samaj Party.
2. Samata Party:
3. Haryana Vikas Patty.
4. Asom Gana Parishad.
Select the correct answer using the codes given below
(a) 1, 2, 3 and 4 (b) 1, 2 and 3
(c) 3 and 4 (d) 1 and 2

149. World Environment Conference to discuss global warming was held in 1997 in
(a) Stockholm
(b) Riodejaneiro
(c) Paris
(d) Kyoto

150. Match List-I with List-II and select the correct answer
List-I List-II
A. Blue Vitriol 1. Sodium Biocarbonate
B. Epsom salt 2. Sodium Hydrooxide
C. Baking soda 3. Magnesium Sulphate
D. Caustic soda 4. Copper Sulphate
A B C D A B C D
(a) 3 4 2 1 (b) 4 3 2 1
(c) 3 4 1 2 (d) 4 3 1 2




General Studies - 1999 (Prelims)
Time Allowed : Two Hours Maximum Marks : 150

1. One consistent feature found in the history of southern India was the growth of small regional
kingdoms rather than large empires because of
(a) the absence of minerals like iron
(b) too many divisions in the social structure
(c) the absence of vast areas of fertile land
(d) the scarcity of manpower

2. Consider the following statements
An ordinary light bulb has a rather short life because the
1. filament wire is not uniform.
2. bulb cannot be evacuated completely.
3. wires supporting the filament melt at high temperature.
Which of the above statements are correct ?
(a) 1 and 3 (b) 2 and 3
(c) 1 and 2 (d) 1, 2 and 3

3. From the balance sheet of a company, it is possible to
(a) judge the extent of profitability of the company
(b) assess the profitability and size of the company
(c) determine the size and composition of the assets and liabilities of the company
(d) determine the market share, debts and assets of the company
4. In eye donation, which one of the following parts of donor's eyes is utilized ?
(a) Iris (b) Lens
(c) Cornea (d) Retina

5. If is is 10.00 a.m. I.S.T, then what would be the local time at Shillong on 92 E longitude ?
(a) 9.38 a.m. (b) 10.38 a.m.
(c) 10.22 a.m. (d) 9.22 a.m.

6. The Constitution of India recognises
(a) only religious minorities
(b) only linguistic minorities
(c) religious and linguistic minorities
(d) religious, linguistic and ethnic minorities

7. In the sequence of numbers
5, 8, 11, X, 34, 55, 89, ....... the value of 'X' is
(a) 20 (b) 21 (c) 23 (d) 29

8. The king was freed from his people and they from their king.' On whose death did Badauni
comment thus ?
(a) Balban
(b) Ala-ud-din Khalji
(c) Muhammad-binTughlak


(d) Feroze Shah Tughlak

9. Match the hormones in List I with items in List II and select the correct answer
List I List II
A. Adrenalin 1. Anger, fear, danger
B. Estrogen 2. Attracting partners through sense of smell
C. Insulin 3. Females
D. Pheromones 4. Glucose
A B C D A B C D
(a) 3 1 4 2 (b) 1 3 2 4
(c) 1 3 4 2 (d) 3 1 2 4

10. In the new Panchayati Raj Bill enacted in 1993, there are several fresh provisions deviating
from the past. Which one of the following is not one such provisions ?
(a) A number of added responsibilities in the area of agriculture, rural development, primary
education and social forestry among others
(b) Elections being made mandatory for all posts at the time they are due
(c) A statutory representation for women in the panchayats, upto a third of the strength
(d) Regular remuneration to the panchayat members, so as to ensure their punctuality and
accountability
11. A ship sailing from the eastern extremity of the Aleutian Islands to Dutch crosses 180
meridian at 23.30 hrs on January 1, 1999. What time and date will be recorded by the captain of the
ship in his diary after one-hour journey from the point of crossing of the meridian ?
(a) January 1, 0030 hrs (b) January 2, 0030 hrs
(c) January 3, 0030 hrs (d) January 4, 0030 hrs

12. Neem tree has acquired industrial importance as a source of
(a) bio-pesticide and antifertility compound
(b) antifertility compound, bio-fertilizer and anti-cancer drug
(c) bio-fertilizer, bio-pesticide and antifertility compound
(d) anti-cancer drug, bio-pesticide and bio-fertilizer
13. On another planet, the local terminology for earth, water, light, air and sky, are 'sky', 'light', 'air',
'water' and 'earth' respectively. If someone is thirsty there, what would be drink ?
(a) Sky (b) Water
(c) Air (d) Light

14. Consider the following statements
An amendment to the Constitution of India can be initiated by the
1. Lok Sabha 2. Rajya Sabha
3. State Legislatures 4.President
Which of the above statements is/are correct ?
(a) 1 alone (b) 1, 2 and 3
(c) 2, 3 and 4 (d) 1 and 2

15. The first venture of Gandhi in all-India politics was the
(a) Non-Cooperation Movement
(b) Rowlatt Satyagraha
(c) Champaran Movement
(d) Dandi March


16. Match List I (Drugs/Chemicals) with list II (Their uses) and select the correct answer
List I List II
A. Atropine 1. Local anaesthesia
B. Ether 2. Heart trouble
C. Nitroglycerine 3. Dilation of pupil
D. Pyrethrin 4. Mosquito control
A B C D A B C D
(a) 1 3 2 4 (b) 1 3 4 2
(c) 3 1 4 2 (d) 3 1 2 4
17. Persons below the poverty line in India are classified as such based on whether
(a) they are entitled to a minimum prescribed food basket
(b) they get work for a prescribed minimum number of days in a year
(c) they belong to agricultural labourer household and the scheduled caste/tribe social group
(d) their dairy wages fall below the prescribed minimum wages
18. Which one of the following scholars suggests the earth's origin from gases and dust particles ?
(a) James Jeans (b) H. Alfven
(c) E Hoyle (d) O. Schmidt

19. In a business concern, there are four functions, namely,
Production (PR), Finance (FN), Personnel (PS) and Marketing (MK). The Customer (C) may also
play a role in the prosperity of the concern. Match List I with List II and select the correct answer
using the codes given below the lists
List I List II
( Pie Diagram ) (Statement)


A B C D A B C D


(a) 4 3 1 2 (b) 5 4 1 2
(c) 4 3 2 5 (d) 5 4 2 1
20. The term 'Aryan' denotes
(a) an ethnic group (b) a nomadic people
(c) a speech group (d) a superior race

21. Match List I (Naturally occurring substances) with List II (Elements) and select the correct
answer
List I List II
A. Diamond 1. Calcium
B. Marble 2. Silicon
C. Sand 3. Aluminium
D. Ruby 4. Carbon
A B C D A B C D
(a) 3 1 2 4 (b) 4 2 1 3
(c) 2 1 3 4 (d) 4 1 2 3

22. Consider the following statements
Regional disparities in India are high and have been rising in recent years because
1. there is persistent investment over time only in select locales.
2. some areas are agro-climatically less conducive to development.
3. some areas continue to face little or no agrarian transformation and the consequent lack of social
and economic opportunities.
4. some areas have faced continuous political instability.
Which of the above statements are correct ?
(a) 1, 2 and 3 (b) 1, 2 and 4
(c) 1, 3 and 4 (d) 2, 3 and 4

23. Match List with List II and select the correct answer
List I List II
(Volcanic Mountain) (Country)
A. Mt. Rainier 1. Italy
B. Etna 2. Mexico
C. Paricutin 3. Philippines
D. Taal 4. U.S.A.
A B C D A B C D
(a) 4 2 1 3 (b) 4 1 2 3
(c) 2 1 4 3 (d) 4 3 2 1
24. Indian farmers are unhappy over the introduction of 'Terminator Seed Technology" because the
seeds produced by this technology are expected to
(a) show poor germination
(b) form low-yielding plants despite the high quality
(c) give rise to sexually sterile plants
(d) give rise to plants incapable of forming viable seeds

25. Consider the following statements about the recent amendments to the Election Law by the
Representation of the People (Amendment) Act 1996
1. Any conviction for the offence of insulting the Indian National flag or the Constitution of India
shall entail disqualification for contesting elections to Parliament and State Legislatures for six


years from the date of conviction.
2. There is an increase in the security deposit which a candidate has to make to contest the election
to the Lok-Sabha.
3. A candidate cannot now stand for election from more than one Parliamentary constituency.
4. No election will now be countermanded on the death of a contesting candidate.
Which of the above statements are correct ?
(a) 2 and 3 (b) 1, 2 and-4
(c) 1 and 3 (d) 1,.2, 3 and 4
26. The average speed of a train in the onward journey is 25% less than that of the return journey.
The trains halts for one hour on reaching the destination. The total time taken for the complete to
and fro journey is 17 hours covering a distance of 800 km. The speed of the train in the onward
journey is
(a) 45 km per hour (b) 47.06 km per hour
(c) 50.00 km per hour (d) 56.25 km per hour

27. Consider the following statements
The striking feature of the Jama Masjid in Kashmir completed by Zain-ul-Abidin include (s)
1. turret
2. similarity with Buddhist pagodas
3. Persian style
Which of the above statements is/are correct ?
(a) 1 alone (b) 1, 2 and 3
(c) 2 and 3 (d) 1 and 3

28. The Ravva offshore block, with great potential for oil, is located in
(a) Krishna-Godavari basin
(b) Cauvery basin
(c) Mahanadi basin
(d) Pblar-Pennar basin

29. Consider the following statements regarding a motor car battery
1. The voltage is usually 12 V
2. Electrolyte used is hydrochloric acid.
3. Electrodes are lead and copper.
4. Capacity is expressed in ampere-hour.
Which of the above statements are correct ?
(a) 1 and 2 (b) 2 and 3
(c) 3 and 4 (d) land 4

30. In the given map, which one of the following map of Ocean current's is Shown ?




(a) Benguela and Falkland (b) canary and Humboldt
(c) Agulhas and Guinea (d) Benguela and Guinea
31. In a town 25% families own a phone and 15% own a car, 65% families own neither a phone
nor a car. 2000 families own both a car and a phone.
Consider the following statements in this regard
1. 10% families own both a car and a phone.
2. 35% families own either a car or a phone.
3. 40,000 families live in the town.
Which of the above statements are correct ?
(a) 1 and 2 (b) 1 and 3 (c) 2 and 3 (d) 1, 2 and 3
32. Which one of the following statements regarding the levying, collecting and distribution of
Income Tax is correct ?
(a) The Union levies, collects and distributes the proceeds of income tax between itself and the
states
(b) The Union levies, collects and keeps all the proceeds of income tax to itself
(c) The Union levies and collects the tax but all the proceeds are distributed among the states
(d) Only the surcharge levied on income tax is shared between the Union and the states
33. The Congress policy of pray and petition ultimately came to an end under the guidance of
(a) Aurobindo Ghosh (b) Bal Gangadhar Tilak
(c) Lala Lajpat Rai (d) Mahatma Gandhi
34. Match List I (Quantity) with List II (Units) and select the correct answer using the codes given
below
List I List II
A. High speed 1. Mach
B. Wavelength 2. Angstrom
C. Pressure 3. Pascal
D. Energy 4. Joule
A B C D A B C D
(a) 2 1 3 4 (b) 1 2 4 3
(c) 12 3 4 (d) 2 1 4 3
35. Consider the following statements
Industrial development in India, to an extent, is constrained by


1. lack of adequate entrepreneurship and leadership in business.
2. lack of savings to invest.
3. lack of technology, skills and infrastructure.
4. limited purchasing power among the larger masses.
Which of the above statements are correct ?
(a) 1, 2 and 3 (b) 1, 3 and 4 (c) 2, 3 and 4
(d) 1, 2 and 4
36. At which one of the following positions shown in the diagram will the height of the ocean tide
be maximum ?

37.Which one of the following genetic diseases is sex-linked?
(a) Royal haemophilia (b) Tay-Sachs disease (c) Cystic
fibrosis (d) Hypertension
38. The yield versus fertilizer input is shown in the
.
Consider the following statements based on this graph
1. Yield rate is zero at B and C. 2. There is no yield with no fertilizer input.
3. The yield is minimum at D. 4. The yield is neither minimum nor maximum at C.
Which of the above statements are correct ?
(a) 1, 2 and 4 (b) 3 and 4 (c) 2 and 3 (d) 1, 3 and 4
39. Among which one of the following sets of social/religious groups is the extent of poverty the
highest, as per Government statistics for the nineties ?
(a) Muslims in Kerala, Gujarat and A.P
(b) Tribals in Bihar, Orissa, M.P and Maharashtra
(c) Scheduled Castes in Punjab,. Western UP northern Rajasthan and Tamil Nadu
(d) Christians in Gujarat, Maharashtra and Assam
40. Which one of the following ports handled the north Indian trade during the Gupta period ?
(a) Tamralipti (b) Broach (c) Kalyan (d) Cambray


41. For which one of the following is capillarity not the only reason ?
(a) Blotting of ink (b) Rising of underground water
(c) Spread of water drop on a cotton cloth
(d) Rising of water from the roots of a plant to its foliage
42. Tourism industry in India is quite small compared to many other countries in terms of India's
potential and size. Which one of the following statements is correct in this regard ?
(a) Distances in India are too far apart and its luxury hotels are too expensive for western tourists
(b) For most of the months India is too hot for western tourists to feel comfortable
(c) Most of the picturesque resorts in India such as in the North-East and Kashmir are, for all
practical purposes, out of bounds
(d) In India, the infrastructure required for attracting tourists is inadequate
43. Which one of the areas marked as A, B, C and D in the given figure of the cyclone, witnesses
heavy torrential short duration rainfall accompanied by thunderstorms ?

(a) A (b) B (c) C (d) D
44. In a code language, 'SOLID' is written as `WPSLPIMFHA'. What does the code word
"ATEXXQIBVO' refer to ?
(a) EAGER (b) WAFER (c) WAGER (d) WATER
45. Which one of the following statements is correct ?
(a) Kacchativu and Tin Bigha were territories acquired by the Indian Republic from the French (b)
Kacchativu and Tin Bigha are territories handed over to Sri Lankan and Bangladesh is sovereignity
by the Government of India
(c) Kacchativu and Tin Bigha are areas that were annexed by the Chinese in the 1962 Sino-Indian
War
(d) Kacchativu and Tin Bigha are enclaves which were transfered to India by lease arrangements
with Sri Lanka and Pakistan respectively

46. Which one of the following pairs is not correctly matched ?
(a) Jahangir William Hawkins
(b) Akbar Sir Thomas Roe
(c) Shahjahan Travernier
(d) Aurangzeb Manucci

47. Consider the following statements
1.If a persons looks at a coin which is in a bucket of water, the coin will appear to be closer than it
really is.
2. If a person under water looks at a coin above, the water surface, the coin will appear to be


at a higher level than it really is.
Which of the above statements is/are correct ?
(a) 1 and 2 (b) 1 alone (c) 2 alone (d) Neither 1 nor 2

48. The Employment Assurance Scheme envisages financial assistance to rural areas for
guaranteeing employment to at least
(a) 50 per cent of the men and women seeking jobs in rural areas
(b) 50 per cent of the men seeking jobs in rural areas
(c) one man and one woman in a rural family living below the poverty line
(d) one person in a rural landless household living below the, poverty line

49. Consider the following temperature and rainfall data
Month Temperature
C
Rainfall cm
January 6.7 14.0
February 6.7 13.2
March 7.2 11.4
April 8.9 9.4
May 11.1 8.1
June 13.9 8.1
July 15.0 9.6
August 15.0 12.2
September 13.9 10.4
October 11.1 14.4
November 8.9 14.0
December 7.8 16.8
The climate to which this data pertains is
(a) St. Lawrence type (b) Chine type
(c) West European type (d) Mediterranean type

50. Lathyrism is caused by excessive consumption of
(a) Kesari Dal (b) Mustard oil (c) Polished rice (d) Mushrooms

51. Match List I with List II and select the correct answer
List I (Persons) List II (Journals)
A.Shyamji Krishna Varms 1. Bande Mataram
B. Madam Bhikaji Cama 2. Indian Sociologist
C. Annie Besant 3. The Talwar
D. Aurobindo Ghosh 4. Commonweal
A B C D A B C D


(a) 2 3 4 1 (b) 3 2 1 4
(c) 2 3 1 4 (d) 3 2 4 1

52. A company manufacturing air-conditioners has set a monthly target. The target and realised
values are shown in the bar chart.


Consider the following statements based on the chart
1.The targeted sales on a monthly basis have been achieved.
2.The overall target value has been exceeded by 7.5%.
3.The sales Department deserves a pat on the back.
Which of the above statements is/are correct ?
(a) 1 alone (b) 2 alone (c) 1 and 2 (d) 2 and 3

53. A British citizen staying in India cannot claim Fight to
(a) Freedom of trade and profession (b) Equality before the Law
(c) Protection of life and personal liberty (d) Freedom of religion

54. Match List I with List II and select the correct answer
List I (Timber) List II (Country)
A. Cedar 1.Myanmar
B. Douglas Fir 2. Canada
C. Mahogany 3. Mexico
D. Teak 4. Honduras
A B C D A B C D
(a) 3 2 1 4 (b) 3 2 4 1
(c) 2 3 4 1 (d) 2 3 1 4

55. In a group of five people, K, L and M are ambitious, M. N and R are honest, L, M and N are
intelligent and K, N and R are industrious. Among these, neither industrious nor ambitious person
(s) would include
(a) K alone (b) Land R (c) M and N (d) none in the group

56. The first writer to use Urdu as the medium of poetic expression was
(a) Amir Khusru (b) Mirza Ghalib
(c) Bahadur Shah Zafar (d) Faiz

57. Barium in a suitable form is administered to patients before an X-ray examination of the
stomach, because


(a) barium allows X-rays to pass through the stomach on account of its transparency to X-rays
(b) barium compound, like magnesium sulphate helps in cleaning the stomach before X-ray
examination
(c) barium is a good absorber of X-rays and this helps the stomach to appear clearly in contrast with
the other regions in the picture
(d) barium salts are white in colour and this helps the stomach to appear clearly in contrast with
other regions in the picture

58. Consider the following statements regarding the National Human Rights Commission of India
1.Its Chairman must be a retired Chief Justice of India.
2.It has formations in each stab?. as State Human Rights Commission.
3.Its powers are only recommendatory in nature.
4.It is mandatory to appoint a woman as a member of the Commission.
Which of the above statements are correct ?
(a) 1, 2, 3 and 4 (b) 2 and 4 (c) 2 and 3 (d) 1 and 3

59. Match List I with List II and select the correct answer
List I List II
A. Cotton 1. Rainfall 1000-1500mm; Temperature 40 - 60C
B. Flax 2. Rainfall 1500-2000mm; Temperature 25 - 35C
C. Sugar beet 3. Rainfall 600-800mm; Temperature 5 -18C
D. Jute 4. Rainfall 500-1000mm; Temperature 18 -22C
5. Rainfall 500 - 600mm; Temperature 18 - 22C
A B C D A B C D
(a) 1 3 4 2 (b) 2 3 5 4
(c) 4 5 2 1 (d) 4 3 5 2

60. The minimum land area recommended for forest cover to maintain proper ecological balance in
India is
(a) 25% (b) 33% (c) 43% (d) 53%

61. A man is standing on the 8m long shadow of a 6 m long pole. If the length of his shadow is
2.4m, what is the height of the man ?
(a) 1.4 m (b) 1.6 m (c) 1.8 m (d) 2.0 m
62. The paintings of Rabindranath Tagore are classified as
(a) realistic (b) socialistic (c) revivalist (d) impressionistic

63. If the angles of a triangle are in the ratio of 4 : 3 : 2 then the triangle
(a) is obtuse (b) has one angle greater than 80
o

(c) is a right triangle (d) is acute

64. From the third century AD when the Hun invasion ended the Roman Empire, the Indian
merchants relied more and more on the
(a) African trade (b) West-European trade
(c) South-East Asian trade (d) Middle-Eastern trade

65. Endoscopes, a technique used to explore the stomach or other inner parts of the body is based
on the phenomenon of
(a) total internal reflection (b) interference (c) diffraction (d) polarization



66. The planning process in the industrial sector in India has assumed a relatively less important
position in the nineties as compared to that in the earlier period. Which one of the following is not
true in this-regard ?
(a) With the advent of liberalization, industrial investments/ development have largely been placed
within the domain of private and multinational sectors
(b) With markets assuming a central place, the role of central planning in many sectors has been
rendered redundant
(c) The focus of planning has shifted to sectors like human resource development, infrastructure,
population control and welfare
(d) The nation's priorities have shifted away from industrial development to rural development

67. Which one of the following port cities in Venezuela has been developed as an oil port ?
(a) Caracas (b) Maraca Ibo (c) Mara cay (d) Carupano

68. Match List I with List II and select the correct answer
List I List II
A. Pandit Vishnu 1. Introduced the scheme of Raga
Digambar Paluskar classification in Indian music
B. Venkatamahi 2. Proponent of the Carnatic music
C. Shyama Shastri 3. Proponent of the Khayal form of Hindustani music
D. Amir Khusru 4. Wrote the music for the song Vande Mataram
A B C D A B C D
(a) 4 1 3 2 (b) 4 1 2 3
(c) 1 4 3 2 (d) 1 4 2 3

69. The surface area of a spherical dome-shaped roof of a cylindrical water tank shown in the figure
is


(a) 60m
2
(b) 109m
2
(c) 120m
2
(d) 300m
2


70. There was no independent development of industries in India during British rule because of the
(a) absence of heavy industries (b) scarcity of foreign capital
(c) scarcity of natural resources (d) preference of the rich to invest in land

71. Strips of two metals A and B are firmly joined together as shown in the figure


On heating, A expands more than B does. If this joined strip is heated, then it will appear as





72. Consider the following statements
Small-scale industries are in most cases, not as efficient and competitive as the large-scale ones.
Yet the Government provides preferential treatment and reservations in a range of products to the
small firms because small-scale industries
1. provide higher employment on a per unit capital development basis.
2. promote a regional dispersion of industries and economic activities.
3. have performed better in export of manufactured products than the large scale ones.
4. provide jobs to low-skill workers, who otherwise may not find employment avenues
elsewhere. Which of the above statements are correct ?
(a) 1 and 4 (b) 1 and 2 (c) 2 and 3 (d) 3 and 4

73. Match the cities labeled as A, B, C and D in the given map with the names of cities and select
the correct answer using the codes given below the names of cities
Names of Cities :
1.Darwin 2. Kuala Lumpur 3. Lagos 4. Nairobi 5. Singapore

A B C D A B C D
(a) 1 2 4 3 (b) 2 1 4 3
(c) 1 4 5 2 (d) 4 3 5 2

74. The first feature film (talkie) to be produced in India was
(a) Hatimtai (b) Alam Ara
(c) Pundalik (d) Raja Harishchandra

75. Which one of the following agricultural practices is eco-friendly ?
(a) Organic farming (b) Shifting cultivation
(c) Cultivation of high yielding varieties (d) Growing plants in glass-houses

76. If X + 2Y = 2X + Y, then X
2
/ Y
2
is equal to
(a) 0 (b) 1 (c) 2 (d) 4

77. A hemispherical bowl is filled to the brim with a beverage. The contents of the bowl are
transferred into a cylindrical vessel whose radius is 50% more than its height. If the diameter is
same for both bowl and cylinder, then the volume of the beverage in the cylindrical vessel will be
(a) 66 2 /3 % (b) 781/2 %
(c) 100% (d) more than 100% (that is, some liquid will still be left in the bowl)

78. A hunter aims his gun at a point between the eyebrows of a monkey sitting on a branch of a
tree. Just as he fires, the monkey jumps down. The bullet will
(a) hit the monkey at the point aimed (b) hit the monkey below the point aimed


(c) hit the monkey above the point aimed (d) miss the monkey altogether

79. The economic crisis in the latter half of 1990s most seriously affected Indonesia, Thailand,
Malaysia and South Korea. The cause of the crisis was
(a) mismanagement of the financial resources and financial sector, in general
(b) the prolonged over-valuation of local currencies vis-a-vis the western currencies
(c) the downswing and recession in the western economies which earlier provided export market to
these export oriented countries
(d) none of the above

80. A person of mixed European and Indian blood in Latin America is called a
(a) Mulatto (b) Mestizo
(c) Meiji (d) Mau Mau

81. Match List I with List II and select the correct answer
List I (Libraries) List II (Locations)
A. Saraswati Mahal Library 1. Patna
B. Library of Tibetan Work and Archives 2. Dharamsala
C. Raza Library 3. Thanjavur
D. Khuda Baksh Oriental Public Library 4. Rampur
A B C D A B C D
(a) 3 2 4 1(b) 3 2 1 4
(c) 2 3 1 4 (d) 2 3 4 1

82. In an office, the distribution of work hours is as shown in the following table
Consider the following inferences drawn from the table
No. of Staff
Members
No. of hours
worked
5
12
25
40
15
8
0 19
20 24
25 29
30 34
35 39
40 - 45
1.The average number of hours worked by a staff member is about 30.
2.The percentage of those who worked 35 or more hours is less than 25.
3.At least 5 staff members worked more than 44 hours.
Which of these inferences is/are valid ?
(a) 1 alone (b) 2 alone
(c) 1 and 2 (d) 1, 2 and 3

83. The following persons came to India at one time or another
1. Fa-Hien 2. I-Tsing
3. Megasthenese 4. Hieun Tsang
The correct chronological sequence of their visits is
(a) 3, 1, 2, 4 (b) 3, 1, 4, 2
(c) 1, 3, 2, 4 (d) 1, 3, 4, 2



84. When atns bite, they inject
(a) glacial acetic acid (b) methanol
(c) formic acid (d) stearic acid

85. The farmers are provided credit from a number of sources for their short- and long-term needs.
The main sources of credit to the farmers include
(a) the Primary Agricultural Cooperative Societies, commercial banks, RRBs and private money
lenders
(b) the NABARD, RBI, commercial banks and private money lenders
(c) the District Central Cooperative Banks (DCCB), the lead banks, IRDP and JRY
(d) the Large Scale Multi-purpose Adivasis Programme, DCCB, IFFCO and commercial banks

86. Which one of the countries labelled as 1, 2, 3 and 4 on the given map was granted membership
of G-15 in its VII Summit at Kuala Lumpur ?
(a) 1 (b) 2 (c) 3 (d) 4

87. In a factory, quality assurance test is conduced on various samples for a specifics characteristic
value of the product; The values and the number of samples are as given in the following table

Characteristic
value, X
No. of samples
10
11
12
13
14
15
16
17
18
19
20
3
7
10
15
28
33
24
11
10
6
3
Consider the following statements based on the table
1.The probability that X < 15 is 0.64
2.The probability that 13 < X _< 17 is greater than 0.64
3.The probability that X = 15 is less than 0.22


Which of the above statements is/are not true ?
(a) 1 alone (b) 1 and 2 (c) 2 and 3 (d) 1, 2 and 3

88. Transchart is the
(a) chartering wing of the Ministry of Surface Transport
(b) container service established by Indian Railways
(c) training institute of maritime studies and research
(d) passenger insurance scheme of Indian Railways

89. To which Lodi Sultan does the given map relate and what town does the side marked A on
the map represent ?
(a) Behlol Lodi, Jaunpur (b) Sikandar Lodi, Aligarh
(c) Ibrahim Lodi, Jaunpur (d) Ibrahim Lodi, Aligarh



90. "India has the largest population of the Asian X. Today, there are just about 20,000 to 25,000 X
in their natural habitat spreading across the evergreen forests, dry thorn forests, swamps and
grasslands. Their prime habitats are, however, the moist deciduous forests. The X population in
India ranges from North West India where they are found in the forest divisions of Dehradun,
Bijnor and Nainital districts of UP to the Western Ghats in the states of Karnataka and Kerala and
in Tamil Nadu.In Central India, their population is distributed in southern Bihar and Orissa. In the
East, they are seen in North Bengal, Assam and a few other states." The animal 'X' referred to in
this quotation is
(a) Lion (b) Elephant (c) Tiger (d) One-horned rhinoceros

91. Which one of the following statements is not true of the Konkan Railway ?
(a) The total length is about 760 km
(b) It runs through the states of Karnataka, Goa, Maharashtra and Kerala
(c) It is the only rail route that cuts across the Western Ghats
(d) The Konkan Railway Construction Company which came into being raised money through
Public Issues

92. The product life cycle from inception to demise is shown in the graph. Match List I with List II


and select the correct answer using the codes given below:

A B C D
(a) 1 4 3 2
(b) 1 4 3 2
(c) 4 1 2 3
(d) 4 1 3 2

93. Which one of the following is a useful functional association between fungi and the roots of
higher plants ?
(a) Bio-fertilizer
(b) Coralloid root
(c) Lichen
(d) Mycorrhiza

94. The physical regions marked as 1, 2, 3 and 4 on the given map are respectively

(a) Andes, Brazilian Shield, Guyana Highlands and Amazon Basin
(b) Andes, Guyana Highlands, Brazilian Shield and Amazon Basin
(c) Amazon Basin,. Guyana Highlands, Brazilian Shield and Andes
(d) Guyana Highlands, Brazilian Shield, Andes and Amazon Basin

95. The given figure shows a portion of Southern India. The proposed site (Koodankulam) for the
construction of two 1000 MW nuclear power plants has been labelled in the map as



(a) A (b) B
(c) C (d) D

96. At a given time, two players are standing on a playfield. The cartesian coordinates of their
locations are (20, 60) and (--40, -20) units. What is the distance between players ?
(a) 60 units (b) 80 units
(c) 100 units (d) 140 units

97. Abinava Bharat a secret society of revolutionaries was organised by
(a) Khudiram Bose
(b) VD. Savarkar
(c) Prafulla Chaki
(d) Bhagat Singh

98. Which one of the following types of coal contains a higher percentage of carbon than the rest ?
(a) Bituminous coal (b) Lignite
(c) Peat (d) Anthracite

99. Which one of the following is the objective of National Renewal Fund ?
(a) To safeguard the interests of workers who may be affected by technological up-gradation of
industry or closure of sick units
(b) To develop the core sector of the economy
(c) For the development of infrastructure such as energy, transport, communications and irrigation
(d) For human resource development such as full literacy, employment, population control, housing
and drinking water

100. Match List I with List II and select the correct answer
List I List II
(Resorts) (States)
A. Chakrata 1. Assam
B. Haflong 2. West Bengal
C. Kalimpong 3. Uttar Pradesh
D. Kufri 4. Himachal Pradesh
A B C D
(a) 1 3 2 4
(b) 3 1 4 2
(c) 3 1 2 4
(d) 1 3 4 2

101. Consider the following statements about the European Union
1. The European Union was known earlier as the European Community.
2. The Single European Act (1986) and the Masticate Treaty were miles ones in its formation.
3. Citizens of European Union countries enjoy dual cityZen ship.


4. Switzerland is a member of the European Union.
Which of the above statements are correct ?
(a) 2 and 4
(b) 1 and 3
(c) 3 and 4
(d) 1, 2 and 3

102. The area of an ellipse is twice that of a circle. The major diameter of the ellipse is twice that of
the minor diameter. The radius of the circle is
(a) 50% of minor diameter of the ellipse
(b) 50% of major diameter of the ellipse
(c) minor diameter of the ellipse
(d) major diameter of the ellipse

103. Which one of the following was initially the most powerful city state of India in the 6th
century B.C.?
(a) Gandhar (b) Kamboj
(c) Kashi (d) Magadh


104. Cobalt-60 is commonly used in radiation therapy because it emits
(a) alpha rays (b) beta rays
(c) gamma rays (d) X-rays

105. In the given figure, the site of the Tehri dam has been labelled as


(a) A (b) B
(c) C (d) D
106. In which one of the following areas in the given map was there a recent discovery of copper
deposits by the Atomic Minerals Division of Department of Atomic Energy ?



(a) 1 (b) 2
(c) 3 (d) 4
107. Which one of the following statements is not correct ?
Dinar/New Dinar is the currency of
(a) Sudan (b) Yugoslavia
(c) U.A.E. (d) Tunisia

108. Which one of the following pairs is not correctly matched ?
(a) Jim Laker Highest number of wickets in a Cricket Test Match
(b) Brian Lara Highest score in an innings in Test Cricket
(c) Sanath Jayasuriya Highest partnership in an innings in Test Cricket
& Roshan Mahanama
(d) Sunil M. Gavaskar Highest aggregate of runs in Test Cricket

109. The most short-lived of all of Britain's constitutional experiments in India was the
(a) Indian Councils Act of 1861
(b) Indian Councils Act of 1892
(c) Indian Councils Act of 1909
(d) Government of India Act of 1919

110. Match List I with List II and select the correct answer
List I List II
(Industries) (Industrial Centres)
A. Pearl fishing 1.Pune
B. Automobiles 2.Tuticorin
C. Ship building 3. Pinjore
D. Engineering goods 4. Marmagao
A B C D
(a) 2 1 4 3
(b) 2 1 3 4
(c) 12 4 3
(d) 1 2 3 4

111. The language spoken by the largest number of people in the world is widely accepted reason
for this ?
(a) Hindi (b) English
(c) Mandarin (d) Spanish

112. "It made its proposals in May. It still wanted a united India. There was to be a Federal Union


composed of British provinces." The above quotation is related to
(a) Simon Commission
(b) Gandhi-Irwin Pact
(c) Cripps Mission
(d) Cabinet Mission

113. The Governor-General who followed a spirited "Forward" policy towards Afghanistan was
(a) Minto (b) Dufferin
(c) Elgin (d) Lytton

114. The first marine sanctuary in India, having within its bounds coral reefs, mollusca, dolphins,
tortoises and various kinds of sea birds, has been established in
(a) Sundarbans (b) Chilka Lake
(c) Gulf of Kachchh (d) Lakshadweep

115. Which one of the following pairs of folk dance forms and states is not correctly matched ?
(a) Korku Maharashtra
(b) Jhummar Haryana
(c) Thali Himachal Fradesh
(d) Mukna Manipur

116. Low temperatures (Cryogenics) find application in
(a) space travel, surgery and magnetic levitation
(b) surgery, magnetic levitation and telemetry
(c) space travel, surgery and telemetry
(d) space travel, magnetic levitation and telemetry

117. The population growth rate in Kerala is the lowest among major Indian states. Which one of
the following is the most widely accepted reason for this ?
(a) Kerala has made the highest investment in family planning
(b) Kerala has the highest literacy rate in India
(c) Kerala has invested heavily in promoting literacy and public health and placed high priority on
social policies
(d) The population pyramid in Kerala has relatively fewer women in the reproductive age group

118. In order to win the Grand Slam in Tennis, a player must win which one of the following
groups of tournaments ?
(a) Australian Open, Wimbledon, French Open, U.S. open
(b) Wimbledon, French Open, U.S. Open
(c) Wimbledon, French Open, Paegas Czech Open, U.S. Open
(d) Davis Cup, Wimbledon, French Open

119. Which one of the following legislations does not deal with the protection of environment ?
(a) The Water (Cess) Act, 1977
(b) The Forest (Conservation) Act, 1980
(c) The Public Liability Insurance Act, 1991
(d) The Port Laws Amendment Act, 1997

120. The Indo-Greek kingdom set up in north Afghanistan in the beginning of the second century


B.C. was
(a) Bactria (b) Scythia (c) Zedrasia (d) Aria
121. Match List I with List II and select the correct answer
List I List II
A. WTO 1. Provides loans to address short-term
balance of payment problems
B. IDA 2. Multi-lateral trade negotiation body
C. IMF 3. Sanction of soft loans
D. IBRD 4. Facilitating lending and borrowing for reconstruction and
development
A B C D A B C D
(a) 2 3 4 1 (b) 2 3 1 4
(c) 3 2 4 1 (d) 3 2 1 4

122. At a time when empires in Europe were crumbling before the might of Napoleon which one of
the following Governors-General kept the British flag flying high in India ?
(a) Warren Hastings (b) Lord Cornwallis
(c) Lord Wellesley (d) Lord Hastings

123. Match the areas shown as A, B, C and D on the given map showing with the largest religious
minorities. Select the correct answer using the codes given below the list of minorities

Largest Religious Minorities
1. Buddhists 2. Christians 3. Jains
4. Muslims 5. Sikhs
A B C D A B C D
(a) 2 3 4 1 (b) 2 3 1 4
(c) 3 2 4 1 (d) 3 2 1 4

124. Which one of the following pairs of states and tribes is not correctly matched ?
(a) Assam Miri
(b) Nag land Konyak
(c) Arunachal Pradesh Apatani
(d) Madhya Pradesh Lambada

125. Amar, Akbar and Anthony are friends, being looked after by a matron Farah. Amar weighs


50% more than Akbar and Anthony weighs 25% less than 25%. Farah weighs a third of the
combined weight of the three boys. All four together weigh 232 kg. The correct arrangement of the
person in the ascending order of their weights is
(a) Anthony, Akbar, Farah, Amar
(b) Anthony, Akbar, Amar, Farah
(c) Akbar, Anthony, Amar, Farah
(d) Akbar, Anthony, Farah, Amar

126. Which Indian nationalist leader looked upon a war between Germany and Britain as a god sent
opportunity which would enable Indians to exploit the situation to their advantage?
(a) C. Rajagopalachari
(b) M.A. Jinnah
(c) Subhash Chandra Bose
(d) Jawaharlal Nehru

127. As per 1991 Census, which one of the following groups of Union Territories had the highest
literacy rate ?
(a) Chandigarh and Dadra & Nagar Haveli
(b) Delhi and Andaman & Nicobar Islands
(c) Andaman & Nicobar Islands and Pondicherry
(d) Pondicherry and Delhi

128. In the rough outline map of a part of Jammu and Kashmir shown in the figure, places marked
A, B, C and D represent respectively

(a) Anantnag, Baramula, Srinagar and Kargil
(b) Baramula, Srinagar, Kargil and Anantnag
(c) Baramula, Srinagar, Anantnag and Kargil
(d) Srinagar, Baramula, Kargil and Anantnag

129. Which one of the following leaders of the Congress was totally in favour of Cabinet Mission
plan ?
(a) Mahatma Gandhi
(b) Jawaharlal Nehru
(c) Sardar Patel
(d) Maulana Abul Kalam Azad

130. In the November 1998 Composite Dialogue Process between India and Pakistan three


contentious issues listed below as 1,2 and 3 were discussed.

Contentious Issues
1.Disengagement of troops
2.Settlement of boundary dispute
3.Sharing river water.
Match the issues with the areas marked in the map as A, B and C and select the correct answer
using the codes given below
A B C A B C
(a) 2 1 3 (b) 2 3 1
(c) 1 3 2 (d) 3 2 1

131. Which one of the following statements is not correct ?
(a) 'Neel Darpan' was a play based on the exploitation of the indigo farmers
(b) The author of the play 'Ghashiram Kotwal is Vjay Tendulkar
(c) The play 'Navanna' by Navind Chandra Das was based on the famine of Bengal
(d) Urdu theatre used to depend heavily on Parsi theatre

132. Which one of the following Indian leaders was dismissed by the British from the Indian Civil
Service ?
(a) Satyendranath Tagore
(b) Surendranath Banerjee
(c) R.C. Dutt
(d) Subhash Chandra Bose

133. Match List I with List II and select the correct answer :
List I List II
(Writers) (Books)
A. Sashi Tharoor 1. Clear light of Day
B. Amitav Ghosh 2. Circle of Reason
C. Anita Desai 3. Love and Longing in Bombay
D. Vikram Chandra 4. Show Business
A B C D A B C D
(a) 4 2 1 3 (b) 4 2 31
(c) 2 4 1 3 (d) 2 4 3 1

134. Match List I with List II and select the correct answer


List I List II
(Rivers) (Dams)
A. Cauvery 1. Alamatti
B. Krishna 2. Mettur
C. Narmada 3. Gandhi Sagar
D. Chambal 4. Sardar Sarovar
A B C D A B C D
(a) 1 4 2 3 (b) 2 1 4 3
(c) 2 1 3 4 (d) 1 3 4 2

135. Match List I with List II and select the correct answer
List I List II
(Books) (Authors)
A. The First Indian War 1. Rabindranath Tagore
of Independence
B. Anand Math 2. Sri Aurobindo
C. Life Divine 3. Bankim Chandra Chatterji
D. Sadhana 4. Vinayak Damodar Savarkar
A B C D A B C D
(a) 4 3 2 1 (b) 3 4 1 2
(c) 4 3 1 2 (d) 3 4 2 1

136. Consider the following events
1. Indigo Revolt
2. Santhal Rebellion
3. Deccan Riot
4. Mutiny of the Sepoys
The correct chronological sequence of these events is
(a) 4, 2, 1, 3 (b) 4, 2, 3, 1
(c) 2, 4, 3, 1 (d) 2, 4, 1, 3

137. Consider the following statements Glass can be etched or scratched by
1. diamond
2. hydrofluoric acid
3. aquaregia
4. conc. sulphuric acid
Which of these statements are correct ?
(a) 1 and 4 (b) 2 and 3
(c) 1 and 2 (d) 2 and 4

138. Match List I with List II and select the correct answer
List I List II
(Years) (Events)
A. 1775 1. First Anglo-Burmese War
B. 1780 2. First Anglo-Afghan War
C. 1824 3. First Anglo-Maratha War
D. 1838 4. Second Anglo-Mysore War
A B C D A B C D
(a) 4 3 2 1 (b) 4 3 1 2
(c) 3 4 1 2 (d) 3 4 2 1



139. Since 1980, the share of the tertiary sector in the total GDP of India has
(a) shown an increasing trend
(b) shown a decreasing trend
(c) remained constant
(d) been fluctuating

140. The term "imperial preference" was applied to the
(a) special privileges on British imports in India
(b) racial discrimination by the Britishers
(c) subordination of Indian interest to that of the British
(d) preference given to British political agents over Indian Princes

Directions : The following 10 (ten) items consist of two statements, one labelled as 'Assertion A'
and the other labelled as 'Reason R'. You are to examine these two statements carefully and decide
if the Assertion A and the Reason R are individually true and if so, whether the Reason is a correct
explanation of the Assertion. Select your answers to these items using the codes given below and
mark your answer sheet accordingly.
(a) Both A and R are true and R is the correct explanation of A
(b) Both A and R are true but R is NOT the correct explanation of A
(c) A is true but R is false
(d) A is false but R is true

141. Assertion (A) : During the time of Akbar, for every ten cavalrymen, the mansabdars had to
maintain twenty horses.
Reason (R) : Horses had to be rested while on march and replacements were necessary in
times of war.

142. Assertion (A) : Lord Linlithgow described the August Movement of 1942 as the most serious
rebellion since Sepoy Mutiny.
Reason (R) : There was massive upsurge of the peasantry in certain areas.

143. Assertion (A) : To dilute sulphuric acid, acid is added to water and not water to acid. Reason
(R) : Specific heat of water is quite large.

144. Assertion (A) : Devaluation of a currency may promote export.
Reason (R) : Price of the country's products in the international market may fall due to
devaluation.

145. Assertion (A) : Fiscal deficit is greater than budgetary deficit.
Reason (R) : Fiscal deficit is the borrowing from the Reserve Bank of India plus other
liabilities of Government to meet its expenditure.

146. Assertion (A) : According to statistics, more female children are born each year than male
children in India.
Reason (R) : In India, the death rate of a male child is higher than that of the female child.
147. Assertion (A) : Insect resistant transgenic cotton has been produced by inserting Bt gene.
Reason (R) : The Bt gene is derived from a bacterium.



148. Assertion (A) : Information technology is fast becoming a very important field of activity in
India.
Reason (R) : Software is one of the major exports of the country and India has a very strong
base in hardware.

149. Assertion (A) : Chile continues to be an important producer of copper in the world. Reason
(R) : Chile is endowed with the world's largest deposit of porphyry copper.

150. Assertion (A) : Dolly was the first cloned mammal.
Reason (R) : Dolly was produced by in vitro fertilization.










General Studies - 2000 (Prelims)
Time Allowed : Two Hours Maximum Marks : 300
1. Insect-resistant cotton plants have been genetically engineered by inserting a gene from a/an
(a) virus (b) bacterium
(c) insect (d) plant

2. The Swarna Jayanti Shahari Rozgar Yojana which came into operation from 1-12-1997 aims to
provide gainful employment to the urban unemployed or underemployed poor but does not include
(a) Nehru Rozgar Yojana
(b) Urban Basic Services Programme
(c) Prime Minister's Integrated Urban Poverty Eradication Programme
(d) Prime Minister's Rozgar Yojana

3. Soft drinks such as colas contain significant quantities of
(a) caffeine (b) nicotine
(c) tannin (d) renin

4. Consider the following statements regarding the Chakiarkoothu form of dance :
1. It is performed by Chakiar caste.
2. It cannot be traditionally witnessed by the higher caste Hindus.
3. Mizhavu is the accompanying instrument.
4. Its theatre form is called koothambalam.
Which of these statements are, correct ?
(a) 1, 3 and 4 (b) 1, 2 and 3
(c) 2, 3 and 4 (d) 1, 2 and 4

5. Consider the following pictures of a dice

What is the number opposite 3 ?
(a) 1 (b) 4 (c) 5 (d) Data insufficient

6. Match List I with List II and select the correct answer
List I (Minerals) List II (Major producer)
A. Mineral Oil 1. Zambia
B. Copper 2. Guyana
C. Manganese 3. Venezuela
D. Bauxite 4. Gabon


A B C D A B C D
(a) 3 1 4 2 (b) 3 1 2 4
(c) 1 3 2 4 (d) 1 3 4 2

7. Which one of the following does a TV remote control unit use to operate a TV set ?
(a) Light waves (b) Sound waves
(c) Microwaves (d) Radio waves

8. Consider the following statements :
The Indian rupee is fully convertible
1. in respect of Current Account of Balance of Payment
2. in respect of Capital Account of Balance of Payment
3. into gold
Which of these statements is/are correct ?
(a) 1 alone (b) 3 alone
(c) 1 and 2 (d) 1, 2 and 3

9. Who among the following streamlined the Maratha administration after Sambhaji ?
(a) Raja Ram (b) Balaji Viswanath
(c) Ganga Bai (d) Nanaji Deshmukh

10. Hybridoma technology is a new biotechnological approach for commercial production of
(a) monoclonal antibodies (b) interferon
(c) antibiotics (d) alcohol

11. Match List I (Books) with List II (Authors) and select the correct answer
List I List II
A. My Music, My Life 1. Laxman Gaikwad
B. Adha Gaon 2. Rahi Masoom Raza
C. Radha 3. Ramakanta Rath
D. The Pilferer 4. Ravi Shankar
A B C D A B C D
(a) 3 2 4 1 (b) 4 2 3 1
(c) 4 1 3 2 (d) 3 1 4 2

12. Examine the following relationships among members of a family of six persons A, B, C, D, E
and F
1. The number of males equals that of females.
2. A and E are sons of F
3. D is the. mother of two, one boy and one girl.
4. B is the son of A.
5. There is only one married couple in the family at present.
Which one of the following inferences can be drawn from the above ?
(a) A, B and C are all females (b) A is the husband of D
(c) E and F are children of D (d) D is the granddaughter of F

13. Which one of the following is knows as the "Coffee port" of the world ?
(a) Sao Paulo (b) Santos
(c) Rio de Janeiro (d) Buenos Aires



14. Resurgent India Bonds were issues in US dollar, Pound, Sterling and
(a) Japanese Yen (b) Deutsche Mark
(c) Euro (d) French Franc

15. The given diagram shows the number of students who failed in an examination comprising
papers in English, Hindi and Mathematics. The total number of students who took the test is 500.
What is the percentage of students who failed in atleast two subjects ?
(a) 6.8 (b) 7.8
(c) 34 (d) 39


16. At which one of the cities labelled as A, B, C and D on the given map of Europe was the
historic treaty between NATO and Warsaw Pact countries signed in 1998 ?

(a) A (b) B (c) C (d) D

17. Match List I (Power generation plant) with List II (Feed material) and select the correct answer
:
List I List II
A. Ws Gowthami Solvents Oil 1. Rice husk
Limited, Andhra Pradesh
B. M/s K.M. Sugar Mills, Uttar 2. Slaughter-house
Pradesh waste
C. M/s Satia Paper Mills, Punjab 3. Distillery spent wash
D. M/s Al Kabeer Exports 4. Black liquor
Limited, Andhra Pradesh
A B C D A B C D
(a) 3 1 2 4 (b) 3 1 4 2
(c) 1 3 4 2 (d) 1 3 2 4



18. In a certain code, MARCH is written as OCTEJ, how is RETURN written in that code ?
(a) TFUVSM (b) QGSTQM
(c) TGVWTP (d) TGRVSO

19. The given map refers to the kingdom of


(a) Akbar at the time of capture of Khandesh in 1601
(b) Akbar at the time of his death in 1605
(c) Aurangzeb at the time of capture of Hyderabad
(d) Aurangzeb at the time of his death in 1707

20. The sensation of fatigue in the muscles after prolonged strenuous physical work is caused by
(a) a decrease in the supply of oxygen
(b) minor wear and tear of muscle fibres
(c) the depletion of glucose
(d) the accumulation of lactic acid

21. Consider the following events
1. Reign of Krishna Deva Raya of Vijayanagara.
2. Construction of Qutab Minar.
3. Arrival of Portugese in India.
4. Death of Fruz Tughlaq.
The correct chronological sequence of these events is
(a) 2, 4, 3, 1 (b) 2, 4, 1, 3
(c) 4, 2, 1, 3 (d) 4, 2, 3, 1

22. A, B, C, D, E and F not necessarily in that order are sitting in six chairs regularly placed around
a round table. It is observed that A is between D and F C is opposite D D and E are not on
neighbouring chairs Which one of the following must be true ?
(a) A is opposite B
(b) D is opposite E
(c) C and B are neighbours
(d) B and E are neighbours

23. The given map shows locations of airports labelled as 1, 2, 3, 4 and 5. What is the correct
sequence of the airports in which the hijacked Indian Airlines place IC-814 landed after its initial
take off from Katmandu in December 1999 ?



(a) 3, 1, 2, 4 (b) 2, 4, 1, 3
(c) 5, 4, 2, 3 (d) 5, 1, 3, 2

24. Consider the following statements :
The Ministerial Meeting of WTO held in December 1999 was unsuccessful because it attempted to
link trade with
1. labour related issues. 2. environment related issues.
3. terrorism related issues. 4. debt related issues.
Which of these statements are correct ?
(a) 1, 3 and 4 (b) 1 and 2
(c) 2 and 3 (d) 2 and 4

25. Match List I with List II and select the correct answer
List I (Artist) List II (Medium of music delivery)
A. Balamurali Krishna 1. Hindustani vocal
B. Mita Pandit 2. Ghatam
C. Kanyakuman 3. Sitar
D. Nikhil Bannerjee 4. Violin
5. Carnatic vocal
A B C D A B C D
(a) 5 1 2 3 (b) 4 3 1 5
(c) 3 1 5 2 (d) 5 4 1 3

26. Aluminium surfaces are often 'anodized'. This means the deposition of a layer of
(a) chromium oxide (b) aluminium oxide
(c) nickel oxide (d) zinc oxide

27. Match List I with List II and select the correct answer
List I List II
A. Iqta 1. Marathas
B. Jagir 2. Delhi Sultans
C. Amaram 3. Mughals
D. Mokasa 4. Vijayanagara
A B C D A B C D
(a) 3 2 1 4 (b) 2 3 4 1
(c) 2 3 1 4 (d) 3 2 4 1

28. Consider the following statements about the 'Roaring Forties' :
1. They blow uninterrupted in the Northern and Southern Hemispheres.
2. They blow with great strength and constancy.
3. Their direction is generally from North-West to East in the Southern Hemisphere.


4. Overcast skies, rain and raw weather are generally associated with them.
Which of these statements are correct ?
(a) 1, 2 and 3 (b) 2, 3 and 4
(c) 1, 3 and 4 (d) 1, 2 and 4

29. In a class there are 18 boys who are over 160 cm tall. If these boys constitute three-fourths of
the boys and the total number of boys is two-third of the number of students in the class, then what
is the number of girls in the class ?
(a) 6 (b) 12 (c) 18 (d) 24

30. A college student desires to get elected to the Municipal Council of his city. The validity of his
nomination would depend on the important condition, among others, that
(a) he obtains permission from the Principal of his college
(b) he is a member of a political party
(c) his name figures in the Voters List
(d) he files a declaration owing allegiance to the Constitution of India
31. Consider the following provinces of former Yugoslavia
1. Bosnia
2. Croatia
3. Slovenia
4. Yugoslavia
The correct sequence of these provinces from the east to the west is:
(a) 4, 1, 3, 2
(b) 4, 1, 2, 3
(c) 1, 4, 3, 2
(d) 1, 4, 2, 3

32. Consider the following statements :
Hard water is not suitable for
1. drinking
2. washing clothes with soap
3. use in boilers
4. irrigating crops
Which of these statements are correct ?
(a) 1 and 3
(b) 2 and 3
(c) 1, 2 and 4
(d) 1, 2, 3 and 4

33. Which one of the following Muslim rulers was hailed as the 'Jagadguru' by his Muslim subjects
because of his belief in secularism ?
(a) Husain Shah
(b) Zain-ul-Abidin
(c) Ibrahim Adil Shah
(d) Mahmud II

34. Match List I with List II and select the correct answer
List I (Local bodies) List II (States as in 1999)
A. Zilla Parishads at the 1. Andhra Pradesh


sub-divisional level
B. Mandal Praja Parishad 2. Assam
C. Tribal Councils 3. Mizoram
D. Absence of Village Panchayats 4. Meghalaya
A B C D
(a) 2 1 4 3
(b) 1 2 4 3
(c) 3 2 1 4
(d) 2 1 3 4

35. A small pouch containing silica gel is often found in bottles of medicine in tablet or powder
form because silica gel
(a) kills bacteria
(b) kills germs and spores
(c) absorbs moisture
(d) absorbs all gases present inside the bottle

36. Which one of the following statements is incorrect?
(a) Goa attained full statehood in 1987
(b) Diu is an island in the Gulf of Khambhat
(c) Daman and Diu were separated from Goa by the 56th Amendement of the Constitution of India
(d) Dadra and Nagar Haweli were under French colonial rule till 1954

37. Canola refers to special type of oil seed mustard varieties bred for human consumption. The
main characteristic of these varieties is that the
(a) seeds have very high oil content
(b) oil is rich in unsaturated fatty acids
(c) oil has long shelf-life
(d) oil has very low Erucic acid content

38. Which one of the following lakes forms an international boundary between Tanzania and
Uganda ?
(a) Chad
(b) Malawi
(c) Victoria
(d) Zambezi

39. A rectangular piece of iron sheet measuring 50 cm by 100 cm is rolled into a cylinder of height
50 cm. If the cost of painting the cylinder is Rs. 50/- per square meter, then what will be the cost of
painting the outer surface of the cylinder ?
(a) Rs.25.00
(b) Rs.37.50
(c) Rs.75.00
(d) Rs.87.50

40. The growth rate of per capita income at current prices is higher than that of per capita income at
constant prices, because the latter takes into account the rate of
(a) growth of population
(b) increase in price level
(c) growth of money supply


(d) increase in the wage rate

41. "In this instance we could not play off the Mohammedans against the Hindus."
To which one of the following events did this remark of Aitchison relate ?
(a) Revolt of 1857
(b) Champaran Satyagraha (1917)
(c) Khilafat and Non-Cooperation Movement (1919-22)
(d) August Movement of 1942

42. A noise level of 100 decibels would correspond to
(a) just audible sound
(b) ordinary conversation
(c) sound from a noisy street
(d) noise from a machine-shop

43. The following news item appeared in a National daily dated 1 - 12 - 1999 :
".......... Parliament today rejected a Bill to grant women the right to vote and stand for office in
parliamentary elections, by a margin of 32 to 30.
The National Assembly was split between liberal, pro-government and Shiite Muslim
depties who were in favour of women's rights, while the opposition camp grouped Sunni Muslim
fundamentalists and tribal MPs.
A total of 64 MPs and Ministers were present, of whom two abstained."
The Parliament referred to in this quotation is that of
(a) Kuwait (b) Iran
(c) Bahrain (d) Saudi Arabia

44. Which one of the following statements is not true ?
(a) Ghaggar's water is utilised in the Indira Gandhi Canal
(b) Narmada rises from Amarkantak region
(c) Nizam Sagar is situated on the Manjra river
(d) Penganga is a tributary of the Godavari

45. The upper part of the given graph is a hypothetical movement in the BSE Sensex over a few
months and the lower part is the fluctuation in the average value of automobile shares in the same
period (actual values not given).



Which one of the following inferences can be drawn from the graphs ?
(a) The automobile-share market has been as unstable as BSE Sensex in that period
(b) There has been a major political change in June/July
(c) Automobile shares have shows a steady improvement in price, unaffected by large fluctuations
in BSE Sensex
(d) None of the above

46. A black hole' is a body in space which does not allow any radiation to come out. This property
is due to its
(a) very small size
(b) very large size
(c) very high density
(d) very low density

47. The first Indian state to have its Human Development Report prepared and released by Amartya
Kumar Sen in Delhi is
(a) West Bengal
(b) Kerala
(c) Madhya Pradesh
(d) Andhra Pradesh

48. In an examination, every candidate took physics or mathematics or both 65.8% took physics
and 59.2% took mathematics. The total number of candidates was 2000. How many candidates took
both physics and mathematics ?
(a) 750 (b) 500
(c) 250 (d) 125

49. Consider the following statements about the megacities of India :
1. Population of each megacity is more than 5 million.
2. All the megacities are importance sea ports.
3. Megacities are either national or state capitals.
Which of these statements are correct ?
(a) 1, 2 and 3 (b) 1 and 2
(c) 2 and 3 (d) 1 and 3



50. The memory of a computer is commonly expressed in terms of kilobytes or Megabyes. A byte
is made up of
(a) eight binary digits
(b) eight decimal digits
(c) two binary digits
(d) two decimal digits

51. The Parliament can make any law for the whole or any part of India for implementing
International treaties
(a) with the consent of all the States
(b) with the consent of the majority of States
(c) with the consent of the States concerned
(d) without the consent of any State

52. A bag contains 20 balls. 8 balls are green, 7 are white and 5 are red. What is the minimum
number of balls that must be picked up from the bag blind-folded (without replacing &.y of it) to be
assured of picking atleast one ball of each colour ?
(a) 4 (b) 7
(c) 11 (d) 16

53. In an open economy, the national income (Y) of the economy is :
(C, I, G, X, M stand for Consumption, Investment, Govt. Expenditure, total exports and total
imports respectively)
(a) Y= C + I + G + X
(b) Y= C + I + G - X + M
(c) Y= C + I + G + (X - M)
(d) Y= C + I - G + X - M

54. Which one of the following is NOT a feature of the Government of India Act of 1935 ?
(a) Diarchy at the Centre as well as in the provinces
(b) A bicameral legislature
(c) Provincial autonomy
(d) An All-India Federation

55. The correct sequence in decreasing order of the four sugarcane producing States in India is
(a) Maharashtra, UP, Tamil Nadu, Andhra Pradesh
(b) UP, Maharashtra, Tamil Nadu, Andhra Pradesh
(c) Maharashtra, U.P, Andhra Pradesh, Tamil Nadu
(d) UP, Maharashtra, Andhra Pradesh, Tamil Nadu

56. Which one of the following statements about a Money Bill is not correct ?
(a) A Money Bill can be tabled in either House of Parliament
(b) The Speaker of Lok Sabha is the final authority to decide whether a Bill is a .Money Bill or not
(c) The Rajya Sabha must return a Money Bill passed by the Lok Sabha and send it for
consideration within 14 days
(d) The President cannot return a Money Bill to the Lok Sabha for reconsideration

57. Consider the following features of newer models of motor cars :
1. Radial tyres


2. Streamline body
3. Multipoint fuel injection
4. Catalytic converter with exhaust
Which of these features make the newer models of motor cars more fueld efficient ?
(a) 1 and 2 (b) 2 and 3
(c) 2, 3 and 4 (d) 1, 3 and 4

58. The Indian National Army (I.N.A.) came into existence in 1943 in
(a) Japan (b) then Burma
(c) Singapore (d) then Malaya

59. Which one of the following has a greater perimeter than the rest ?
(a) A square with an area of 36 sq. cm.
(b) An equilateral triangle with a side of 9 cm.
(c) A rectangle with 10 cm as length and 40 sq. cm. as area
(d) A circle with a radius of 4 cm.

60. Match the cities labelled as 1, 2, 3, 4 and 5 in the given map with the names of the institutes
located in these cities and select the correct answer using the codes given below the names of
the institutes

Name of institutes
A. Central Marine Fisheries Research Institute
B. Central Sheep Breeding Farm
C. National Dairy Research Institute
D. National Institute of Agricultural Extension Management
A B C D
(a) 5 1 3 2
(b) 5 2 1 4
(c) 4 2 1 3
(d) 1 2 3 4

61. A rise in ' SENSEX' means
(a) a rise in prices of shares of all companies registered with Bombay Stock Exchange
(b) arise in prices of shares of all companies registered with National Stock Exchange
(c) an overall rise in prices of shares of group of companies registered with Bombay Stock
Exchange
(d) a rise in prices of shares of all companies belonging to a group of companies registered with


Bombay Stock Exchange

62. The new G D P series released by the C S O in February, 1999 is with reference to base price of
(a) 1991-92 (b) 1992-93
(c) 1993-94 (d) 1994-95

63.

In the given figure OQP = 30 and ORP = 20, QOR is equal to
(a) 1001 (b) 1200 (c) 1301 (d) 1400

64. Consider the following statements :
1. Maharashtra has the highest acreage under jawar in India.
2. Gujarat is the largest producer of groundnut in India.
3. Rajasthan has the largest area of cultivable wasteland in India.
4. Andhra Pradesh has the highest per hectare yield of maize in India.
Which of these statements are correct ?
(a) 1 and 4 (b) 2 and 3 (c) 1 and 3 (d) 2 and 4

65. A club has 1,08 members. Two-thirds of them are men and the rest are women. All members
are married except for 9 women members. How many married women are there in the club?
(a) 20 (b) 24 (c) 27 (d) 30

66. The last major extension of British Indian territory took place during the time of
(a) Dufferin (b) Dalhousie (c) Lytton (d) Curzon

67. Fluorescent tubes are fitted with a choke. The choke coil
(a) steps up the line voltage (b) steps down the line voltage
(c) reduces current in the circuit (d) chokes low frequency currents

68. Which one of the following pairs is not correctly matched?
(a) Baki Itihas Badal Sarkar (b) Sita Swayamvar : Vishnu Das Bhave
(c) Yayati Girish Kamad (d) Giddha Jabbar Patel

69. The following figure represents times vs. learning curves of two students. Q and R for learning
a mathematics lesson

Which one of the following inferences can be drawn from the graph


(a) R started slowly in the beginning but got ahead of Q. to complete learning the lesson
(b) Q started slowly and finished learning the lesson earlier then R
(c) R was always faster than Q in learning mathematics
(d) Q was always faster than R in learning mathematics

70. Match List-I with List-II and select the correct answer
List I (Industrial Unit) List II (Centre)
A. Atlas Cycle Company Ltd. 1. Bangalore
B. Bharat Earth Movers Ltd. 2. Bhubaneswar
C. Indian Farmers Fertilizers Co-operative Ltd. 3. Kalol
D. National Aluminium Company Ltd 4. Sonepat
A B C D A B C D
(a) 1 4 2 3 (b) 1 4 3 2
(c) 4 1 2 3 (d) 4 1 3 2

71. For reproducing sound, a CD (Compact Disc) audio player uses a
(a) quartz crystal (b) titanium needle
(c) laser beam (d) barium titanate ceramic

77. The 73
rd
Constitution Amendment Act 1992 refers to the
(a) generation of gainful employment for the unemployed and the under-employed men and women
in rural area
(b) generation of employment for the able bodies adults who are in need ant desirous of work
during the lean agricultural season
(c) laying the foundation for strong and vibrant Panchayati Raj Institutions in the country
(d) guarantee of right to life, liberty and security of person, equality before law and equal protection
without discrimination.

73. A goat is tied to two poles P and Q with ropes that are 15 meters long. P and Q are 20 meters
apart as shown in the given diagram

Which one of the following shaded portions indicates the total area over which the goat can
graze ?




74. Match the locations of ports labelled as A, B, C and D in the given map with the names of those
ports and select the correct

A B C D A B C D
(a) 4 2 3 5 (b) 5 2 4 1
(c) 1 3 4 2 (d) 5 3 2 1

75. When a CD (compact disc used in audio and video systems) is seen in sunlight, rainbow like
colours are seen. This can be explained on the basis of the phenomenon of
(a) reflection and diffraction (b) reflection and transmission
(c) diffraction and transmission (d) refraction, diffraction and transmission

76. AS an alternative to the partition of India, Gandhiji suggested to Mountbatten that he
(a) postpone granting of independence
(b) invite Jinnah to form the government
(c) invite Nehru and Jinnah to form the government together
(d) invite the army to take over for some time

77. A person starts from a point A and travels 3 km eastwards to B and then turns left and travels
thrice that distance to reach C. He again turns left and travels five times the distance he covered
between A and B and reaches his destination D. The shortest distance between the starting point
and destination is
(a) 18 km (b) 16 km (c) 15 km (d) 12 km

78. Which one of the following ports of India handles the highest tonnage of import cargo ?
(a) Calcutta (b) Kandla
(c) Mumbai (d) Visakhapatnam

79. The native state of Tripura became involved in the Freedom movement early in the 20
th
century
because
(a) the kings of Tripura were always anti-British
(b) the Bengal revolutionaries took shelter in Tripura
(c) the tribes of the state were fiercely freedom loving
(d) there were already some groups fighting against the kingship and its protector, the British

80. Match List I (Industrial processes) with List II (Industry with which associated) and select the
correct answer :
List I List II


A. Cracking 1. Rubber
B. Smelting 2. Petroleum
C. Hydrogenation 3. Copper
D. Vulcanization 4. Edible Fats
A B C D A B C D
(a) 3 2 1 4 (b) 2 3 4 1
(c) 2 3 1 4 (d) 3 2 4 1

81. Within biological communities, some species are important in determining the ability of a large
number of other species to persist in the community. Such species are called
(a) Keystone species (b) Allopatric species
(c) Sympatric species (d) Threatened species

82. Match List I with List II and select the correct answer
List I List II
A. Boom 1. Business activity at high level with increasing income, output and
employment at macro level
B. Recession 2. Gradual fall-of income, output and employment with business activity in a
low gear
C. Depression 3. Unprecedented level of under employment and unemployment, drastic fall
in income, output and employment
D. Recovery 4. Steady rise in the general level of prices, income, output and employment
A B C D A B C D
(a) 1 2 3 4 (b) 1 2 4 3
(c) 2 1 4 3 (d) 2 1 3 4

83. The distribution of 1,00,000 tourists who visited India during a particular year is shown in the
given charts. Based on this, the number of Japanese tourists below the age of 39 who visited
India in the year concerned is

(a) 10,000 (b) 8,000 (c) 6,000 (d) 4,000

84. Match the different ports of Union Territory of Pondicherry labelled as A, B, C and D in the
given map with their respective names and select the correct answer using the codes given below
the list of pots



List (Ports of Pondicherry)
1. Karaikal 2. Mahe
3. Pondicherry 4.Yanam
A B C D A B C D
(a) 2 1 3 4 (b) 1 2 3 4
(c) 2 1 4 3 (d) 1 2 4 3

85. That the per capital income in India was Rs. 20/- in 1867-68, was ascertained for the first time
by
(a) M. G. Ranade (b) Sir W. Hunter
(c) R. C. Dutta (d) Dadabhai Naoroji

86. Which one of the following pairs of primitive tribes and places of their inhabitation is NOT
correctly matched ?
(a) Buksa......................... Pauri-Garhwal
(b) Kol............................. Jabalpur
(c) Munda........................ Chhotanagpur
(d) Korba......................... Kodagu

87. What is maximum number of pieces of 5 cm x 5 cm x 10 cm cake that can be cut from a big
cake of 5 cm x30 cm x 30 cm size?
(a) 10 (b) 15 (c) 18 (d)30

88. Which one of the following statements is true according to 1991 census data ?
(a) UP has the highest density of population in India
(b) Himachal Pradesh has the highest female to male sex ratio in India
(c) West Bengal has the highest growth rate of population in India
(d) Bihar has the lowest literacy rate in India

Directions :- The next two items are based on the following table. Study the same carefully and
attempt the two items that follow it :
Table
Indicators of development for some Asian countries
Country Life Infant Adult
expectancy at birth mortality rate literacy rate
(Years) (Per 1000 live births) (Percent)
1995 1996 1995
India 62.4 72 52
China 69.2 38 82
Indonesia 64.0 47 84
Malaysia 71.4 11 84


Thailand 69.5 31 94
Korea 71.7 6 98
Philippines 67.4 32 95

89. Which one of the following statements is false?
(a) All countries other than India have over 80% literacy
(b) Malaysia and Korea have life expectancy higher than all other countries
(c) Higher the adult literacy lower is the infant mortality
(d) The life expectancy at birth in India is almost the same as that of Indonesia

90. The best performance in terms of Human Development among the Asian countries is by
(a) China (b) Malaysia (c) Korea (d) Philippines

91. After returning from South Africa, Gandhiji launched his first successful Satyagraha in
(a) Chauri-Chaura
(b) Dandi
(c) Champaran
(d) Bardoli

92. Match List I with List II and select the correct answer
List I List II
A. Chittagong Armoury Raid 1. Kalpana Dutt
B. Abhinav Bharat 2. Guru Ram Singh
C. AnushilanSamiti 3. Vikram Damodar Savarkar
D. Kuka Movement 4. Aurobindo Ghosh
Codes :
A B C D
(a) 1 3 4 2
(b) 1 3 2 4
(c) 3 1 2 4
(d) 3 1 4 2

93. Match List I with List II and select the correct answer
List I List II
A. Land allotted to big feudal landlords 1. Jagirdari System
B. Land allotted to revenue farmers or 2. Ryotwari System
rent collectors
C. Land allotted to each peasant with 3. Mahalwari
the right to sublet, mortgage, gift or sell
D. Revenue settlements made at village 4. Zamindari System level
Codes :
A B C D
(a) 1 3 2 4
(b) 1 4 2 3
(c) 3 4 1 2
(d) 2 1 3 4

94. Which one of the following countries occupies the first place in the "Global Competitive
Report" of World Economic Forum ?


(a) U.S.A.
(b) Singapore
(c) Hong Kong
(d) France
Directions :- The following 16 (sixteen) items consist of two statements, one labelled as 'Assertion
A' and the other labelled as 'Reason R'. You are to examine these two statements carefully and
decide if the Assertion A and the Reason R are individually true and if so, whether the Reason is a
correct explanation of the Assertion. Select your answers to these items using the codes given
below and mark your answer sheet accordingly.
Codes :
(a) Both A and Rare true and R is the correct explanation of A
(b) Both A and R are true but R is NOT the correct explanation of A
(c) A is true but R is false
(d) A is false but R is true

95. Assertion (A) : Lord Linlithgo described the August Movement of 1942 as the most serious
revolt after the Sepoy mutiny.
Reason (R) : Peasants joined the movement in large number in some places.
96. Assertion (A) : A man standing on a completely frictionless surface can propel himself by
whistling.
Reason (R) : If no external force acts on a system, its momentum cannot change.
97. Assertion (A) : The Aham and Puram poems of the Padinen Kilukanakku group formed a
continuation of the Sangam composition.
Reason (R) : They were included under the Post-Sangam works as against the Sangam
works proper.
98. Assertion (A) : In Australia, cattle rearing is done more for meat than for milk.
Reason (R) : Australians are traditionally non-vegetarians.

99. Assertion (A) : In human beings, the females play a major role in determining the sex of the
offspring.
Reason (R) : Women have two 'X' chromosomes.

100. Assertion (A) : Large cold storage plants use ammonia as refrigerant while domestic
refrigerators use chlorofluorocarbons.
Reason (R) : Ammonia can be liquefied at ambient temperatures at low pressures.
101. Assertion (A) : The frequency of floods in North Indian plains has increased during the last
couple of decades.
Reason (R) : There has been reduction in the depth of river valleys due to deposition of silt.
102. Assertion (A) : The rate of growth of India's exports has shown an appreciable increase after
1991.
Reason (R) : The Govt. of India has resorted to devaluation.
103. Assertion (A) : "DNA Finger printing" has become a powerful tool to establish paternity and
identity of criminals in rape and assault cases.


Reason (R) : Trace evidences such as hairs, saliva and dried semen are adequate for DNA
analysis.
104. Assertion (A) : Ganga Plain is the most densely populated part of India.
Reason (R) : Ganga is the most harnessed river of India.

105. Assertion (A) : The emphasis of Jainism on non-violence (ahimsa) prevented agriculturalists
from embracing Jainism.
Reason (R) : Cultivation involved killing of insects and pests.

106. Assertion (A) : The origin of feudal system in ancient India can be traced to military
campaigns.
Reason (R) : There was considerable expansion of the feudal system during the Gupta
period.
107. Assertion (A) : Ashoka annexed Kalinga to the Mauryan Empire.
Reason (R) : Kalinga controlled the 'land and sea routes to South India.
108. Assertion (A) : The basic weakness of the early nationalist movement lay in its narrow social
base.
Reason (R) : It fought for the narrow interests of the social groups which joined it.

109. Assertion (A) : In a motion picture, usually 24 frames are projected every second over the
whole length of the film.
Reason (R) : An image formed on the retina of eye persists for about 0.1 s after the removal
of stimulus.

110. Assertion (A) : Small glass beads fixed on traffic signals glow brightly when light falls upon
them.
Reason (R) : Light is totally reflected when the angle of incidence exceeds a certain critical
value and light traveling in a denser medium is reflected from a rarer medium.

111. Match List I with List II and select the correct answer
List I List II
A. Development Programme 1. U N India Human Development Report
B. National Council of Applied 2. India Development Economic Research Report
C. Indira Gandhi Institute of 3. World Development Development Research Report
D. World Bank 4. Human Development Report
Codes :
A B C D
(a) 4 1 2 3
(b) 4 2 1 3
(c) 2 3 4 1
(d) 2 1 4 3

112. "........... instil into the vast millions of workers, men and women, who actually do the job, a
sense of partnership and of cooperative performance " The above passage related to
(a) Planned Development
(b) Community Development
(c) Panchayati Raj System
(d) Integrated Development Programme



113. The 'stones' formed in human kidney consist mostly of
(a) calcium oxalate
(b) sodium acetate
(c) magnesium sulphate
(d) calcium

114. The Speaker can ask a member of the House to stop speaking and let another member speak.
This phenomenon is known as
(a) decorum
(b) crossing the floor
(c) interpellation
(d) yielding the floor

115. While delivering the presidential address, the Congress President who advocated the
introduction of Roman script for Hindi language was
(a) Mahatma Gandhi
(b) Jawaharlal Nehru
(c) Abul Kalam Azad
(d) Subhash Chandra Bose

116. Most of the explosions in mines occur due to the mixing of
(a) hydrogen with oxygen
(b) oxygen with acetylene
(c) methane with air
(d) carbon dioxide with ethane

117. Consider the following statements about the Attorney General of India
1. He is appointed by the President of India.
2. He must have the same qualifications as are required for a Judge of the Supreme Court.
3. He must be a member of either House of Parliament.
4. He can be removed by impeachment by Parliament.
Which of these statements are correct ?
(a) 1 and 2
(b) 1 and 3
(c) 2, 3 and 4
(d) 3 and 4

118. A rectangular water tank measures 15 m x 6 m at top and is 10 m deep. It is full of water. If
water is drawn out lowering the level by 1 meter, how much of water has been drawn out ?
(a) 90,000 litres
(b) 45,000 litres
(c) 4,500 litres
(d) 900 litres

119. Along which one of the following meridians did India experience the first light of the sunrise
of the new millennium ?
(a) 20
o
30
'
W
(b) 82
o
30' E
(c) 92
o
30' W


(d) 92
o
30' E

120. Which one of the following organisms can serve as a biofertilizer for rice crop ?
(a) Blue-green algae
(b) Rhizobium sp
(c) Mycorrhizal fungi
(d) Azotobacter sp

121. The Standing Committee of State Finance Ministers recommended in January, 2000 uniform
rates across the states in respect of
(a) value-added tax
(b) sales tax
(c) stamp duty and registration fees
(d) agricultural income tax

122. At the time of partition of India, which one of the following provinces of British India came
forward with a plan for a united and independent existence ?
(a) Punjab
(b) Assam
(c) Bengal
(d) Bihar

123. Consider the following functionaries
1. Cabinet Secretary
2. Chief Election Commissioner
3. Union Cabinet Ministers
4. Chief Justice of India
Their correct sequence, in the Order of Precedence is
(a) 3, 4, 2, 1
(b) 4, 3, 1, 2
(c) 4, 3, 2, 1
(d) 3, 4, 1, 2

124. The primary function of the Finance Commission in India is to
(a) distribute revenue between the Centre and the States
(b) prepare the Annual Budget
(c) advise the President on financial matters
(d) allocate funds to various ministries of the Union and. State Governments

125. The state which has the largest number of seats reserved for the Scheduled Tribes in the Lok
Sabha is
(a) Bihar
(b) Gujarat
(c) Uttar Pradesh
(d) Madhya Pradesh

126. An accurate clock shows 8 O' clock in the morning. Through how many degrees will the hour
hand rotate when the dock shows 2 O'clock in the afternoon ?
(a) 150
o
(b) 144
o


(c) 168 (d) 180

127. Consider the following statements :
1. Tides are of great help in navigation and fishing.
2. High tide enables big ships to enter or leave the harbour safely.
3. Tide prevents siltation in the harbours.
4. Kandla and Diamond Harbour are tidal ports.
Which of these statements are correct ?
(a) 1 and 4
(b) 2, 3 and 4
(c) 1, 2 and 3
(d) 1, 2, 3 and 4

128. At which stage in its life cycle does the silk worm yield the fibre of commerce ?
(a) Egg
(b) Larva
(c) Pupa
(d) Imago

129. Indian Human Development Report does not give for each sample village
(a) Infrastructure and Amenities Index
(b) Education Related Index
(c) Health Related Index
(d) Unemployment Related Index

130. The monthly income of Komal and Asha are in the ratio of 4 : 3. Their monthly expenses are
in the ratio of 3 : 2. However, both save Rs. 600/- per month. What is their total monthly income?
(a) Rs. 8,400 /-
(b) Rs. 5,600 /-
(c) Rs. 4,200 /-
(d) Rs. 2,800 /-

131. Match List I with List II and select the correct answer
List I (Oceanic Trench) List II (Location)
A. Aleutian 1. Indian Ocean
B. Kermadec 2. North Pacific Ocean
C. Sunda 3. South Pacific Ocean
D. S. Sandwich 4. South Atlantic Ocean
A B C D
(a) 2 4 1 3
(b) 2 3 1 4
(c) 1 3 2 4
(d) 1 4 2 3

132. The Balkan Plan for fragmentation of India was the brainchild of
(a) W. Churchil
(b) M. A. Jinnah
(c) Lord Mountbatten
(d) V P Menon



133. Match List I with list II and select the correct answer
List I List II
A. German Silver 1. Tin
B. Solder 2. Nickel
C. Bleaching Powder 3. Sodium
D. Hypo 4. Chlorine
A B C D
(a) 1 2 4 3
(b) 2 1 3 4
(c) 1 2 3 4
(d) 2 1 4 3

134. Match List I with list II and select the correct answer
List I List II
(Institute) (Location)
A. Central Institute of Higher Tibetan 1. Hyderabad
Studies
B. Indira Gandhi Institute of Development 2. Mumbai
Research
C. National Institute of Mental Health 3. Bangalore
and Neuro-sciences
D. Central Institute of English and Foreign 4. Dharamshala
Languages 5. Varanasi
A B C D A B C D
(a) 5 3 4 1 (b) 5 2 3 1
(c) 3 2 4 5 (d) 4 5 1 2

135. Match List I (Diseases) with List II (Types of disease) and select the correct answer
List I List II
A. Haemophilia 1. Deficiency disease
B. Diabetes 2. Genetic disease
C. Rickets 3. Hormonal disorder
D. Ringworm 4. Fungal infection
A B C D A B C D
(a) 2 3 4 1 (b) 2 3 1 4
(c) 3 2 1 4 (d) 3 2 4 1

134. Economic liberalisation in India started with
(a) substantial changes in industrial licensing policy
(b) the convertibility of Indian rupee
(c) doing away with procedural formalities for foreign direct investment
(d) significant reduction in tax rates

137. The blood glucose level is commonly expressed as
(a) mm. of Hg
(b) milligram per decilitre
(c) parts per million
(d) grammes per litre

138. The practice of military governorship was first introduced in India by the


(a) Greeks (b) Shakas
(c) Parthians (d) Mughals

139. If X = - 2, then
X
3
- X
2
- X - 1 is equal to
(a) 1 (b) - 3
(c) -11 (d) -15

140. Consider the following statements about the Indian National Congress
1. Sarojini Naidu was the first woman to be the President of the Congress
2. C. R. Das was in prison when he functioned as the President of the Congress
3. The first Britisher to become the President of the Congress was Alan Octavian Hume
4. Alfred Webb was the President of the Congress in 1894
Which of these statements are correct ?
(a) 1 and 3
(b) 2 and 4
(c) 2, 3 and 4
(d) 1, 2, 3 and 4

141. Match the drainage basins labelled as A; B, C and D with the names listed below and select
the correct answer using the codes given below the drainage basins
Names of drainage basins :
1. Ganga-Brahmaputra 2. Indus
3. Parana 4. Zambeszi
DRAINAGE BASINS

A B C D A B C D
(a) 3 1 2 4 (b) 1 3 4 2
(c) 1 3 2 4 (d) 3 1 4 2

142. Phytotron is a facility to
(a) grow plants under disease-free conditions
(b) conserve endangered species of plants
(c) grow plants under controlled conditions
(d) induce mutations

143. The Raga which is sung early in the morning is
(a) Todi (b) Darbari
(c) Bhopali (d) Bhimpalasi

144. In the given figure, all line segments of the shaded portions are of the same length and at right
angles to each other. The same can be cut out of a board of side 10 cm. What is the area of


the shaded portion ?

(a) 36 cm
2

(b) 48 cm
2

(c) 52 cm
2

(d) 64 cm
2


145. Gilt-edged market means
(a) bullion market
(b) market of Government securities
(c) market of guns
(d) market of pure metals

146. Match List I (Endocrine glands) with List II (Hormones secreted) and select the correct answer
List I List II
A. Gonads 1. Insulin
B. Pituitary 2. Progesterone
C. Pancreas 3. Growth hormones
D. Adrenal 4. Cortisone
A B C D A B C D
(a) 3 2 4 1 (b) 2 3 4 1
(c) 2 3 1 4 (d) 3 2 1 4

147. Which one of the following dynasties was ruling over North India at the time of Alexanders
invasion ?
(a) Nanda (b) Maurya (c) Sunga (d) Kanva

148. Match the international events listed below with their respective places labelled as 1, 2, 3, 4
and 5 in the given map and select the correct answer using the codes given below the lists



International Events :
A. Venue of Commonwealth Conference held in 1999
B. Venue of World Trade Organisation meeting held in 1999
C. Place of Israel-Syria Peace talks held in January, 2000
D. Place of military action by Russian troops in January, 2000
A B C D A B C D
(a) 2 1 5 3 (b) 3 4 2 1
(c) 4 1 2 3 (d) 4 3 5 2

149. Which one of the following materials is very hard and very ductile ?
(a) Carborundum (b) Tungsten
(c) Cast iron (d) Nichrome

150. To reach the final of first grand slam of the year 2000, Martina Hingis defeated
(a) Lindsay Davenport (b) Jennifer Capriati
(c) Sarena William (d) Conchita Martinez

Potrebbero piacerti anche